You are on page 1of 183

1.

Basic Science and Structure of Skin


# Question MCQ A
1 Pick the correctly paired keratin 1. K1/K10 - basal cells E
with its structure: 2. K3/K12 - esophagus
3. K4/K13 - cornea
4. K5/K14 - suprabasal cells
5. K16/K6 - palms and soles
2 Which of the following statements 1. The DIF is negative in the vast majority of C
about the direct cases
immunofluorescence pattern in 2. Deposition of IgG is within cytoid bodies in
lichen planus is correct? the superficial dermis
3. The DEJ deposition is granular
4. Deposition of fibrinogen is within cytoid
bodies in the deep dermis
5. There is prominent deposition of IgM within
the spinous layer of the epidermis
3 Krause end bulbs are: 1. Adapting mechanoreceptors found on B
weight-bearing sites that respond to vibrational
stimul
2. Found on the vermillion border of the lips
3. Located in the dermal papillae of digital skin
4. Located in the deep dermis and within the
subcutis in weight-bearing sites of the body
5. Found at the orifice of the hair follicle and
particularly sensitive to cold
4 On electron microscopy, which cell 1. Langerhans cell C
demonstrates cytoplasmic 2. Keratinocyte
projections and secretory 3. Mast cell
granules? 4. Melanocyte
5. Macrophage
5 When do melanocytes begin to 1. 2nd month of gestation B
synthesize melanin? 2. 3rd month of gestation
3. 4th month of gestation
4. 5th month of gestation
5. 6th month of gestation
6 Epidermolysis bullosa simplex 1. Collagen VII D
(EBS), Weber Cockayne type, is 2. Alpha-6-beta-4 integrin
caused by what defect? 3. Keratins 1 & 10
4. Keratins 5
5. Plectin
7 Mutations in which of the following 1. Uncein B
proteins results in epidermolysis 2. Plectin
bullosa simplex associated with 3. a6b4 integrin
muscular dystrophy: 4. Laminin 5
5. Collagen type IV
8 S100 staining would be negative 1. langerhans cells E
in? 2. eccrine cells
3. schwann cells
4. adipocytes
5. keratinocytes
9 Direct immunofluorescence is of no 1. Neonatal LE A
value in the diagnosis of: 2. Lichen planus
3. Mixed connective tissue disease
4. SLE
5. Erythema multiforme
10 The major component of the 1. Uncein D
anchoring filaments is: 2. Plectin
3. a6b4 integrin

1
4. Laminin 5
5. Collagen type IV
11 Hair grows at: 1. 0.004 mm/day C
2. 0.04 mm/day
3. 0.4 mm/day
4. 4 mm/day
5. 10 mm/day
12 Fragmentation and/or loss of 1. Cutis laxa E
elastic fibers in not seen in: 2. Marfan's syndrome
3. Anetoderma
4. Psuedoxanthoma elasticum
5. Buschke-Ollendorf Syndrome
13 Sebaceous glands secrete sebum 1. Holocrine A
through which of the following 2. Merocrine
secretory mechanisms? 3. Apocrine
4. Holocrine and Merocrine
5. Holocrine and Apocrine
14 Which of the following is true 1. The first primordial hair follicles form at 15 C
regarding the development of hair weeks gestation
follicles? 2. The first hair follicles form on the scalp and
eyelashes
3. Follicles develop in a cephalad to caudal
direction
4. New follicles develop during the first 3
months postpartum
5. The eyebrows develop late in gestation
15 The proteins that make up the 1. Stratum corneum D
cornified cell envelope are 2. Stratum basale
synthesized in the: 3. Melanocytes
4. Stratum granulosum
5. Langerhans cells
16 Keratin filaments in basal cells 1. Desmosomes A
insert into: 2. Adherens junctions
3. Connexins
4. Lamellar granules
5. Odland bodies
17 Which of the following statements 1. Postganglionic sympathetic fibers with A
is true about eccrine glands? acetylcholine as the principal neurotransmitter
2. Postganglionic sympathetic fibers with
norepinephrine as the principal neurotransmitter
3. Postganglionic parasympathetic fibers with
acetylcholine as the principal neurotransmitter
4. Postganglionic parasympathetic fibers with
norepinephrine as the principal neurotransmitter
5. Postganglionic sympathetic fibers with both
norepinephrine and acetylcholine as the principal
neurotransmitters
18 Red or blonde hair pigmentation 1. The presence of eumelanin C
primarily results from: 2. The absence of melanin
3. The presence of pheomelanin
4. The reduced activity of tyrosinase
5. The reduced activity of DOPA dehydroxylase
19 Each of the following is true about 1. Most characteristic organelle of the D
melanosomes except: melanocyte
2. Tyrosinase activity decreases as
melanosomoes mature
3. Are transferred to keratinocytes via
phagocytosis
4. Are singly dispersed in the basal layer in
white skin

2
5. Are larger in size in black skin compared to
white skin
20 The microflora of pilosebaceous 1. Pityrosporum ovale A
unit consist of which of the 2. Staphylococcus aureus
following: 3. Escherichia coli
4. Pseudomonas aeruginosa
5. Corynebacterium diphtheriae
21 Which protein is the largest 1. Keratin D
component of the cornified cell 2. Involucrin
envelope? 3. Profilaggrin
4. Loricrin
5. Ceramide
22 Apocrine glands are found in all of 1. Axillae D
the following areas of the body 2. Breasts
except: 3. Eyelid
4. Palms
5. Perineum
23 Which type of collagen in mutated 1. Collagen I A
in osteogenesis imperfecta? 2. Collagen II
3. Collagen III
4. Collagen IV
5. Collagen VII
24 The finding on DIF that reflects 1. Granular fluorescence throughout the A
binding of the Ro and La antigens cytoplasm and nucleus of basal keratinocytes
in subacute cutaneous lupus 2. Cytoid bodies
erythematosus is: 3. Immune deposits along the DE junction
4. Granular deposits along the basement
membrane
5. A "chicken-wire" pattern within the epidermis
25 Cutaneous warts in renal 1. K2 D
transplant recipients show 2. K5
increased expression of which 3. K9
keratin? 4. K13
5. K16
26 The best substrate for 1. Rat bladder C
immunofluoresence in the 2. Monkey esophagus
dermatologic condition transmitted 3. Guinea pig esophagus
by the black fly is: 4. Armadillo foot pads
5. White mouse tail
27 Which of the following options is 1. Defects in the alpha-6 domain of integrin E
characteristic of integrins? result in epidermolysis bullosa simplex with
muscular dystrophy
2. Alpha-6-Beta-4 integrin is found at sites
where desmogleins attach
3. Expression is seen in all layers of the
epidermis
4. The extracellular alpha6 domain binds
collagen 7
5. These proteins coordinate linkage between
intermediate filaments and extracellular matrix
of the basement membrane
28 Which of the following amino acids 1. Desmosine A
are typically found in elastic fibers? 2. Isoleucine
3. Proline
4. Hydroxyproline
5. Glycine
29 The cutaneous 1. Intercellular IgG and C3 D
immunofluorescence pattern in 2. Linear IgG and C3 along the basement
patients with Senear-Usher membrane zone
syndrome is: 3. Intercellular IgG on guinea pig esophagus

3
4. Intercellular IgG and C3 and linear IgG and
C3 along the basement membrane zone
5. Linear IgG and C3 along the basement
membrane zone and intercellular IgG on guinea
pig esophagus
30 Which of the following statements 1. Tumors of glomus cells are most commonly C
about glomus cells is correct? found on the tongue
2. Are of neural origin
3. Allow rapid shunting of blood from the
arterioles to venules, bypassing capillaries
4. Tumors composed of glomus cells are
asymptomatic
5. Tumors composed of glomus cells are often
malignant
31 Keratinocytes are derived from 1. Endoderm C
which of the following: 2. Mesoderm
3. Ectoderm
4. Neural Crest
5. Bone marrow precursors
32 Which mechanoreceptor found in 1. Merkel cell C
hair bearing areas sense deep 2. Meissner corpuscle
touch and vibration? 3. Vater-Pacini corpuscle
4. Krase end-bulb
5. Free nerve ending
33 Which of the following medications 1. Cyclophosphamide E
is concentrated in the eccrine 2. Cytarabine
glands? 3. Ciprofloxacin
4. Cephalexin
5. All of the answers are correct
34 Dermal dendrocytes: 1. Are responsible for immediate-type C
hypersensitivity reactions.
2. Actively synthesize and release IgG.
3. Present antigen to na�ve T cells in the
lymph node.
4. Are the primary cell found in a glomangioma.
5. Are the primary cell found in an
angiosarcoma.
35 Which element is necessary for 1. Iron E
function of matrix 2. Nitrogen
metalloproteinases? 3. Manganese
4. Magnesium
5. Zinc
36 Which of the following diseases has 1. Flegel's A
decreased or absent lamellar 2. Psoriasis
granules? 3. Lamellar ichthyosis
4. Epidermolytic hyperkeratosis
5. Pemphigus vulgaris
37 Which of the following cells are 1. Neutrophil B
required for wound healing? 2. Macrophage
3. Eosinophil
4. Langerhans cell
5. Lymphocyte
38 Glomus cells are: 1. Modified fibroblasts C
2. Modified skeletal muscle cells
3. Vascular smooth muscle cells
4. Monocytes
5. Neuronal cells
39 Apocrine glands: 1. Demonstrate holocrine secretion B
2. Demonstrate decapitation secretion
3. Are fully functional at birth

4
4. Are diffusely distributed on the body
5. Are thermoregulatory
40 The greatest density of mast cells 1. Stratum spinosum C
is found in the: 2. Stratum basale
3. Papillary dermis
4. Reticular dermis
5. Subcutaneous fat
41 Which signaling molecule mediates 1. Foxn1 C
the transition of hair cycling from 2. Fgf5
telogen to anagen phase? 3. Sonic hedgehog
4. Dihydotestosterone
5. 5a-reductase
42 Which one of the following is 1. Involucrin E
responsible for maintaining a 2. Filaggrin
barrier to water loss in the stratum 3. Loricrin
corneum? 4. Transglutaminase
5. Odland bodies
43 Tyrosinase is the enzyme that 1. Zinc B
catalyzes the conversion of 2. Copper
tyrosine to DOPA and DOPA to 3. Selenium
DOPAquinone. The enzyme 4. Iron
contains which of the following 5. Magnesium
ions?
44 The major protein component of 1. Envoplakin D
the cornified envelope is: 2. Desmoplakin
3. Plectin
4. Loricrin
5. Transglutaminase
45 The embryonic periderm becomes 1. vernix caseosa A
part of the 2. stratum corneum
3. stratum basale
4. dermis
5. hair follicle
46 The strength of a scar: 1. Is 5% at 1 week D
2. Is 20% at 3 weeks
3. Is 70% at 1 year
4. All of these options are correct
5. None of these options are correct
47 Homocystinuria has abnormal 1. Cystathione synthase A
crosslinking of collagen because of 2. Type I collagen N-peptidase gene
a mutation in: 3. Lysyl hydroxylase
4. Tenascin X
5. Lysyl oxidase
48 Which of the following protein 1. Uncein B
plays a major role in wound 2. Fibronectin
healing? 3. Nidogen
4. Entactin
5. Band-6 protein
49 In the epidermis, the cell most 1. Keratinocytes D
responsible for antigen detection 2. Merkel Cell
and processing is the: 3. Melanocyte
4. Langerhans cell
5. CD4+ T cell
50 How soon does epithelialization 1. Minutes B
begin after a skin wound occurs? 2. Hours
3. 2 days
4. 4 days
5. 6 days
51 Desmosine and isodesmosine are 1. Collagen fibers C
typical amino acids found in: 2. Anchoring fibril

5
3. Elastic fibers
4. Heparan sulfate
5. Anchoring plaques
52 Retinoids upregulate transcription 1. 1 and 3 C
of which types of collagen? 2. 1 and 4
3. 1 and 7
4. 3 and 7
5. 4 and 7
53 Mutations in which of the following 1. MC1-R A
genes will produce red hair? 2. Agouti
3. Hairless
4. c-kit
5. Tyrosinase
54 Keratinocytes have been shown to 1. IL-1 E
secrete all of the following 2. IL-6
cytokines except: 3. IL-8
4. TNF-alpha
5. IL-2
55 Direct immunofluorescence 1. Paraneoplastic pemphigus A
staining of intercellular spaces and 2. Anti-epiligrin pemphigoid
the basement membrane zone, in 3. Pemphigus vegetans
combination, is seen in: 4. Pemphigus foliaceous
5. IgA pemphigus
56 The desmosomal connections of 1. Iron D
the epidermis are dependent on 2. Zinc
which of the following ions? 3. Selenium
4. Calcium
5. Sodium
57 Which of the following is the most 1. GNAQ D
common genetic alteration seen in 2. Cyclin Dependant Kinase 4/6
mucosal melanomas? 3. BRAF
4. KIT
5. HDM2
58 People with darker skin show: 1. Smaller, more concentrated melanosomes D
2. A more rapid degradation of melanosomes
3. A lessened production of melanosomes
within melanocytes
4. A higher degree of dispersion of
melanosomes in keratinocytes
5. A grouping of melanosomes with a low
degree of melanization
59 A salt split skin DIF is performed on 1. Epidermal side A
a biopsy taken adjacent to the skin 2. Dermal side
lesions shown. Where would you 3. Epidermal and Dermal sides equally
expect staining to be seen? 4. In the lamina densa
5. In the anchoring plaques
60 Which eponym describes vestigial 1. Fuchter lines A
lines of pigmentary demarcation? 2. Wallace's lines
3. Langer's lines
4. Lines of Blaschko
5. Dermatome
61 Each of the following is true about 1. Anchoring filaments attach the basal cell A
the basement membrane zone membrane to the lamin lucida
except: 2. Can be visualized on light microscopy with
PAS staining
3. Lamina densa is composed of type IV
collagen
4. Contains laminin 1 and laminin 5
5. Anchoring fibrils are composed of type VII
collagen

6
62 During hair follicle development, 1. Smoothened B
the WNT signaling pathway is one 2. Beta-catenin
of the earliest molecular pathways 3. Keratin 16
involved in hair follicle initiation. 4. p53
What is the downstream mediator 5. HLA-B27
of WNT signaling?
63 Type VII collagen in found in 1. Fetal skin C
anchoring fibrils and also in: 2. Bone
3. Amnion
4. Aorta
5. Blood vessels
64 Itch is most commonly transmitted 1. C-polymodal nociceptor class nerves A
by: 2. A-delta class nerves
3. A-beta class nerves
4. Parasympathetic postganglionic fibers
5. A-beta and A-delta fibers
65 What component is the major 1. Sebum D
barrier in the stratum corneum? 2. Squalene
3. Collagen
4. Ceramide
5. Triglycerides
66 Sneddon Wilkinson disease is 1. Desmocollin 1 A
caused by a defect in what 2. Desmoglein 1
antigen? 3. Desmogelin 3
4. BPAg1
5. BPAg2
67 Which of the following statements 1. Elastic fibers form 35% of the dry weight of B
about elastic fibers is true? the skin
2. Elastic fibers are 90% elastin wrapped in
fibrillin
3. Collagen 1 is mutated in Marfan syndrome
4. Oxytalan fibers run parallel within the
superficial papillary dermis
5. Elaunin fibers run perpendicular in thin
bands within the reticular dermis.
68 Where are glomus cells derived 1. Mesenchyme C
from? 2. Bone marrow
3. Susquet-Hoyer canal
4. Skeletal muscle
5. Dermis
69 Which of the following cytokines 1. IL-2 E
are secreted from keratinocytes? 2. IL-3
3. IL-17
4. IL-22
5. IL-23
70 Desmoglein 1 is the antigen in 1. Pemphigus foliaceus A
which of the following autoimmune 2. Bullous impetigo
diseases of the skin: 3. Dermatitis herpetiformis
4. Bullous pemphigoid
5. Pemphigoid gestationis
71 Which cytokeratin would you 1. K1 E
expect to be preferentially 2. K5
expressed in the keratinocytes of a 3. K12
psoriatic plaque: 4. K15
5. K16
72 Elaunin fibers: 1. Run parallel in bands within the superficial C
papillary dermis
2. Run perpendicular from the dermo-epidermal
junction within the superficial papillary dermis
3. Run parallel in bands within the reticular

7
dermis
4. Run perpendicular in bands within the deep
dermis
5. Run perpendicular in bands within the
reticular dermis
73 Regarding dermal-epidermal 1. There are no anchoring filaments in lamina C
junction, which of the following lucida
statements is true: 2. Lamina fibroreticularis lies above lamina
densa
3. Lamina fibroreticularis comprises of
anchoring fibrils and the elastic microfibrils
4. Blood vessels cross the dermal-epidermal
junction to reach the epidermis
5. Lamina lucida is an electron-dense layer
74 Which of the following enzymes 1. Lysyl oxidase E
does not require copper for 2. ATP7a
functioning? 3. Tyrosinase
4. Cystathione beta-synthase
5. Ferrochelatase
75 What structure delineates the 1. Nail matrix E
anatomic region between the nail 2. Proximal nail fold
bed and the distal groove, where 3. Lunula
the nail plate detaches of the distal 4. Eponychium
portion of the digit? 5. Hyponychium
76 Which keratins are expressed in 1. K4, 13 E
the stratum germinativum and are 2. K1,10
present but not made de novo in 3. K2e, 10
the stratum spinosum? 4. K3,12
5. K5,14
77 Which of the following dermal cells 1. Mononuclear phagocytic cells A
always express CD11c and CD6? 2. Fibroblasts
3. Mast cells
4. Erythrocytes
5. Glomus cells
78 Which of the following statements 1. The number of melanosomes in melanocytes D
about darkly pigmented races are the same
versus lighter pigmented races is 2. The individual melanosomes have the same
correct? degree of melanization
3. The melanosomes are equal in size
melanosomes
4. There are equal numbers of melanocytes
5. There is a faster rate of melanosome
degradation
79 Which of the following domains is 1. NC16A of BP180 A
targeted by the autoantibodies in 2. Laminin 5
bullous pemphigoid? 3. Plectin
4. alpha-6 integrin
5. NC16A of BP230
80 Acid keratins are expressed on 1. 12 B
which of the following 2. 17
chromosomes? 3. 9
4. 16
5. 3
81 Which of the following cytokines is 1. IL-2 C
secreted by adipocytes? 2. Il-4
3. IL-6
4. IL-10
5. IL-12
82 Tissue contraction begins: 1. At 3rd day of wound healing B
2. During the 2nd week of wound healing

8
3. After the first month of wound healing
4. After the 3rd month of wound healing
5. After the 9th month of wound healing
83 The hair follicle is the site of 1. All the options are correct A
production or conversion of which 2. dihydrotestosterone (DHT)
of the following hormones? 3. prolactin
4. adrenocorticotropin hormone (ACTH)
5. alpha-melanocyte stimulating hormone
(MSH)
84 What is the major function of 1. Bacteriocidal acid produced by stratum C
urocanic acid? corneum
2. Primarily a UVB filter
3. Primarily a UVA filter
4. Helps degrade free fatty acids
5. Aids in protecting the skin from
dermatophytes
85 A specific marker of Merkel cells is: 1. Cytokeratin 10 C
2. Cytokeratin 15
3. Cytokeratin 20
4. Loricrin
5. Envoplakin
86 Which keratins are upregulated in 1. Keratins 1 and 10 D
hyperproliferative disease such as 2. Keratins 2e and 10
psoriasis? 3. Keratins 5 and 14
4. Keratins 6 and 16
5. Keratins 8 and 18
87 A 78 year-old man is diagnosed 1. BPAg2 B
with cicatricial pemphigoid. Which 2. Laminin 5
of the following antigens being 3. Laminin 6
implicated in his disease should 4. Beta-4 integrin
trigger a work-up for malignancy? 5. Type VII collagen
88 The main permeability barrier in 1. heparan sulfate proteoglycan A
the lamina densa is: 2. collagen IV
3. laminin 5
4. nidogen
5. alpha-6-beta-4 integrin
89 Moving from internally to 1. Inner root sheath cuticle - Huxley's layer - D
externally choose the correct Henle's layer - medulla - cortex - hair shaft
description of the hair follicle: cuticle
2. Hair shaft cuticle - cortex - medulla - Henle's
layer - Huxley's layer _ inner root sheath cuticle
3. Henle's layer - Huxley's layer - inner root
sheath cuticle - hair shaft cuticle - cortex -
medulla
4. Medulla _ cortex _ hair shaft cuticle _ inner
root sheath cuticle _ Huxley's layer _ Henle's
layer
5. Inner root sheath cuticle _ outer root sheath
cuticle _ hair shaft cuticle _ cortex _ Huxley's
layer _ Henle's layer
90 Platelets release which of the 1. Neutrophil chemotactic factor D
following factors to promote new 2. IL-1
tissue growth? 3. ADP
4. TGF-alpha
5. FGF
91 Which component of hair is 1. Outer root sheath B
positive for citrulline? 2. Inner root sheath
3. Cortex
4. Glassy vitreous layer
5. Medulla

9
92 What percentage of the dry weight 1. 2 B
of skin in elastin? 2. 4
3. 6
4. 8
5. 10
93 Apocrine glands: 1. Are coiled glands E
2. Have a two segment ducts that empties onto
the skin
3. Are present everywhere on the skin except
on the palms and soles
4. Function from birth
5. Secretions are initially odorless
94 All mononuclear phagocytic cells in 1. CD3 B
the dermis express: 2. CD6
3. CD34
4. CD68
5. CD20
95 The epidermis is comprised of what 1. Keratinocytes, Melanocytes, Merkel cells, A
type of cells? Langerhan cells
2. Keratinocytes, Endothelial cells, Merkel cells,
Langerhan cells
3. Keratinocytes, Melanocytes, Neutrophils,
Langerhan cells
4. Keratinocytes, Melanocytes, Merkel cells,
Goblet cells
5. Keratinocytes, Endothelial cells, Merkel cells,
Goblet
96 Defects in what kind of structural 1. Collagen D
protein lead to pyloric atresia 2. Elastin
associated with junctional 3. Loricrin
epidermolysis bullosa: 4. Integrin
5. Plectin
97 What is the most important cell for 1. Fibroblasts C
wound healing? 2. Neutrophils
3. Macrophages
4. Lymphocytes
5. Mast cells
98 All of the following are true 1. The inner root sheath keratinizes by means D
regarding the formation of hair of trichohyalin granules
except: 2. Henle�s layer is outside of Huxley�s layer
3. The outer root sheath is a downward
extension of the epidermis
4. Huxley�s layer contains melanin
5. The hair matrix becomes the hair and the
inner root sheath
99 At what estimated gestational age 1. 8 weeks E
are all layers of the keratinized 2. 12 weeks
epidermis identifiable? 3. 16 weeks
4. 20 weeks
5. 24 weeks
100 Which cadherin is responsible for 1. E-cadherin A
adhesion of Langerhan cells to the 2. P-cadherin
epidermis? 3. N-cadherin
4. Desmoglein
5. Desmocollin
101 Homocystinuria is an autosomal 1. Abnormal crosslinking of collagen A
recessive condition with findings 2. Abnormal development of elastin fibers
including a marfanoid habitus, 3. Melanocyte death
downward dislocation of the lens, 4. Pigmentation of cartilage
cardiovascular disease and mental 5. Black urine

10
retardation. It is caused by a
mutation in cystathionine beta-
synthetase. What does this
mutation in cystathionine beta-
synthetase cause other than an
accumulation of homocystine?
102 Hair follicle development in the 1. 1st trimester A
human embryo begins during: 2. 2nd trimester
3. 3rd trimester
4. At the blastocoele stage
5. Within 2 weeks of fertiization
103 Upon presentation of an antigen in 1. B cells B
the skin surface, a hapten forms. 2. Langerhans cells
The first cell to take up the hapten 3. keratinocytes
is: 4. T cells
5. Mast cell
104 Which of the following keratins 1. K6a/16 B
would most likely be expressed in 2. K6b/17
the nail bed? 3. K1/9
4. K2e/10
5. K4/13
105 Keratohyalin granules contain: 1. Desmoplakin C
2. Envoplakin and Keratin 6
3. Profilaggrin and loricrin
4. Numerous Golgi apparati
5. Involucrin
106 Elastic fibers contain the specific 1. Lysine and proline D
amino acids: 2. Leucine and isoleucine
3. Alanine and phenylalanine
4. Desmosine and isodesmosine
5. Glycine and proline
107 Which of the following 1. plakoglobin E
polypeptides is found in the lamina 2. desmoplakin
lucida? 3. keratocalmin
4. demoyokin
5. laminin 5
108 Which of the following is not a 1. Moll's gland A
specialized type of sebaceous 2. Meibomian gland
gland? 3. Gland of Zeis
4. Montgomery's tubercle
5. Fordyce spot
109 The antibody target in ocular 1. Junctional epidermolysis bullosa, Herlitz type E
cicatricial pemphigoid is also 2. Recessive dystrophic epidermolysis bullosa
mutated in: 3. Junctional epidermolysis bullosa with
myotonic dystrophy
4. Dominant dystrophic epidermolysis bullosa
5. Junctional epidermolysis bullosa with pyloric
atresia
110 During embryogenesis, periderm 1. Ceramide B
cells of the fetus contain which of 2. Glycogen
the following substances? 3. Free fatty acids
4. Porphyrins
5. Sebum
111 Anchoring fibrils are primarily 1. Type I collagen D
composed of: 2. Type III collagen
3. Type IV collagen
4. Type VII collagen
5. Type II collagen
112 Which of the following is true about 1. Spheroid melanosomes have concentric E
melanosomes? lamellae

11
2. Spheroid melanosomes synthesize brown-
black eumelanin
3. Elliptical melanosomes have microvesicular
structure
4. Elliptical melanosomes synthesize yellow or
red pheomelanin.
5. The melanosomes are positioned after the
Golgi apparatus in the secretory pathway.
113 At any one time, the approximate 1. 40% C
proportion of hair follicles in 2. 60%
anagen is: 3. 85%
4. 95%
5. 15%
114 The first primordial hair follicles 1. 7 weeks B
form on the eyebrows, upper lip 2. 9 weeks
and chin at which gestational age? 3. 12 weeks
4. 16 weeks
5. 20 weeks
115 Anchoring filaments originate at 1. Desmosome E
the hemidesmosomes and insert 2. Sub basal dense plate
into the: 3. Lamina lucida
4. BPAG 180
5. Lamina densa
116 Odland bodies: 1. Contain keratins E
2. Are found intercellularly in the basal cell
layer
3. Are exclusively intracellular
4. Crosslink with keratins 5 and 14
5. Are secretory granules with features of
lysosomes
117 Telogen effluvium: 1. Involves a diffuse alopecia affecting more E
than 50% of the scalp
2. Often results from antimetabolites used
during cancer chemotherapy
3. Is an inflammatory alopecia devoid of
scarring
4. Is a patchy alopecia affecting less than 50%
of the scalp
5. Results from sudden illness or surgery
118 What is the major component of 1. Type I Collagen D
the anchoring fibril? 2. Type III Collagen
3. Type IV Collagen
4. Type VII Collagen
5. Laminin 5
119 Anagen effluvium is best described 1. An abrubt transition from anagen to catagen B
as: in rapidly dividing hair matrix cells
2. A cessation of mitotic activity in rapidly
dividing hair matrix cells
3. An abrupt transition of telogen to anagen in
resting hair matrix cells
4. A cessation of mitotic activity in resting hair
matrix cells
5. A scarring alopecia affecting only anagen
stage follicles
120 What is the average duration of the 1. 2-6 years C
telogen cycle in terminal scalp 2. 2-3 weeks
hair? 3. 3 months
4. 6 months
5. 9 months
121 Regarding sebaceous glands: 1. These glands are present at birth at their D

12
adult size
2. Size of the gland is proportional to the size of
the associated hair follicle
3. Are always associated with a hair follicle
4. Are found everywhere on the skin except
palms and soles
5. Are unilobular glands
122 Which hormone is homologous to 1. Insulin E
alpha-MSH (melanocyte 2. Human growth factor
stimulating hormone)? 3. Prolactin
4. Thyroid stimulating hormone
5. Adrenocorticotropic hormone
123 Anagen effluvium is 1. Rarely seen following administration of B
cancer chemotherapeutic agents
2. Stimulus induces the abrupt cessation of
miotic activity in the rapidly dividing hair matrix
cells
3. Occurs within 24 to 48 hours of exposure
4. Entirely inreversible
5. Patients being treated with nitrosurea agents
are usually spared
124 Embryologically, epidermal 1. 4 weeks B
stratification occurs at 2. 8 weeks
approximately what estimated 3. 12 weeks
gestational age? 4. 16 weeks
5. 20 weeks
125 Regarding the stratum 1. Intermediate filaments in basal cells insert C
germinativum (basale): into only hemidesmosomes
2. Keratins 1 and 10 are expressed
3. Not all basal cells have the potential to
divide
4. Microfilaments assist in downward
movement of cells
5. Plectins regulate adhesion and initiation of
differentiation.
126 Regarding the stratum spinosum, 1. No keratin 1/10 is present C
which of the following is correct? 2. New synthesis of K5/14 occurs in this layer
3. The "spines" seen on pathology are due to
desmosomal connections between keratinocytes
4. This layer contains melanocytes
5. This layer contains the cornified cell
membrane
127 Elastic fibers are present in the 1. Fibrillins and fibulins A
dermis and are responsible for 2. Hyaluronic acid
providing tissue resiliency. They 3. Collagen II
are comprised of elastin as well as 4. Laminin 5 and BPAG2
microfibrillar proteins including: 5. Desmoplakin and plakoglobin
128 What is the location of the 1. Cytoplasm A
unbound corticosteroid receptor? 2. Nucleus
3. Mitochondria
4. Plasma membrane
5. golgi apparatus
129 Granulation tissue primarily 1. Collagen I B
contains: 2. Collagen III
3. Collagen IV
4. Fibrin
5. Collagen VII
130 Sebaceous glands form a lipid-rich 1. Vacuolar exocytosis E
substance called sebum and are 2. Passive diffusion
usually associated with a hair 3. Decapitation secretion

13
follicle. They secrete sebum by 4. Autocrine secretion
what mechanism? 5. Holocrine secretion
131 1 melanocyte has contact with: 1. 6 keratinocytes E
2. 12 keratinocytes
3. 18 keratinocytes
4. 30 keratinocytes
5. 36 keratinocytes
132 All of the following pertain to 1. Contain squalene A
Odland bodies EXCEPT: 2. Are found intracellularly in upper level
keratinocytes
3. Discharge their contents into the
extracellular space at the junction of the granular
and cornified layers
4. Establish a barrier to water loss
5. Mediate stratum corneum adhesion in
conjunction with filaggrin.
133 Which of the following is a member 1. E-cadherin E
of the armadillo family of linking 2. Periplakin
proteins? 3. Envoplakin
4. Desmocollin
5. Plakoglobin
134 Numerous neuromediators are 1. Noradernaline B
involved in cutaneous neurobiology 2. Substance P
and many play a role in the 3. Neurokinin A
development of inflammation in 4. Acetylcholine
the skin. One such mediator can be 5. Pro-opiomelanocortin
induced by application of capsaicin
to the skin. Which of the following
is the correct neuromediator?
135 Type 1 collagen is the most 1. Type II collagen B
prevalent collagen in skin, 2. Type III collagen
accounting for 80% or more of the 3. Type IV collagen
total collagen in the adult dermis. 4. Type VII collagen
The next most predominant 5. Type XVII collagen
collagen in adult human dermis is:
136 Melanocytes are derived from: 1. Bone marrow B
2. Neural crest
3. Mesodermal precursors
4. Endodermal precursors
5. Yolk sac derived
137 Which of the following make up the 1. Loricrin A
major protein of the cornified cell 2. Involucrin
envelope? 3. Envoplakin
4. Filaggrin
5. Laminin V
138 Mast cells are derived from bone 1. CD3 D
marrow ____+ cells? 2. CD6
3. CD20
4. CD34
5. CD68
139 Which of the following 1. lamellar granules are uniformly absent A
abnormalities would be present in 2. keratohyaline granules are normal in all
a patient with Harlequin types of harlequin fetus
ichthyosis? 3. demoyokin mutation
4. band 6 protein is absent
5. plakoglobin is abnormal
140 Which cell type is required for 1. Macrophage A
wound healing? 2. Lymphocyte
3. Mast cell
4. Dermal dendrocyte

14
5. Neutrophil
141 Acid keratins (K10-20) are 1. 17 A
expressed on which of the 2. 18
following chromosomes? 3. 19
4. 10
5. 12
142 Apocrine chromhidrosis results 1. Lipofuschin A
from which of the following 2. Squalene
contents of apocrine sweat? 3. Cholesterol
4. Fatty acids
5. Ammonia
143 Meibomian glands are: 1. Eccrine glands localized to the vermillion C
border of the lips
2. Sebaceous glands found on the areola of the
breast
3. Sebaceous glands found on the eyelids
4. Apocrine glands found in the anogenital
regions
5. Apocrine glands found on the eylelids
144 The triple helix of the collagen 1. Alanine and asparagine E
molecule is largely maintained due 2. Tyrosine and threonine
to its amino acid composition. The 3. Histidine and ornithine
polypeptide chains of collagen are 4. Leucine and isoleucine
repeating triplets of Glycine-X-Y. 5. Proline and hydroxyproline
The X and Y positions can be
occupied by multiple amino acids,
but are most often:
145 Human sebum is distinguished 1. Cholestrol D
from lipids of internal organs by 2. Cholestrol esters
the presence of: 3. Squalene
4. Wax esters
5. Glycerides
146 The first cell type to migrate into a 1. Neutrophil A
new wound in great numbers is 2. Monocyte
the: 3. Macrophage
4. Lymphocyte
5. Mast cell
147 Dystrophic epidermolysis bullosa 1. Collagen type I E
results from mutations in: 2. Collagen type II
3. Collagen type III
4. Collagen type V
5. Collagen type VII
148 Which of the following statements 1. Laminins span from the plasma membrane C
about Laminins is correct? of basal keratinocytes to the lamina lucida
2. Laminins provide little structural support in
the basement membrane
3. Laminins provide signaling molecules that
interact with other proteins to transmit
morphogenetic information to the cell's interior
4. Laminin 5 is also called plectin
5. Laminin 5 is the only laminin found in the
basement membrane
149 Which of the following skin 1. Rothmund-Thompson C
conditions involves elastin? 2. Epidermolytic Hyperkeratosis
3. Pseudoxanthoma elasticum
4. Progeria
5. Classic type Ehlers-Danlos syndrome
150 Which of the following elements is 1. Copper A
necessary for melanin production? 2. Selenium
3. Iron

15
4. Zinc
5. Calcium
151 Keratinocytes in the basal layer of 1. Keratins 1 and 10 B
the epidermis attach to the 2. Keratins 5 and 14
basement membrane zone at 3. Desmogleins 1 and 3
hemidesmosomes via what 4. Desmoplakin and Desmoglobin
intermediate filament molecules? 5. BPAG1 and BPAG2
152 Which of the following stains would 1. S-100 A
you expect to be positive in a 2. Prussian blue
normal eccrine unit? 3. Giemsa
4. Verhoeff von Gieson
5. Steiner
153 Which of the following statements 1. Links fillagrin to the plasma membrane D
about plectin is correct? 2. Crosslinks proteins in adherens junction
3. Mutations result in junctional epidermolysis
bullosa with pyloric atresia
4. Plectin is a member of the plakin family
5. Plectin is a constituent protein of the
desmosomal plaque
154 Aprocrine glands are adnexal 1. Vacuolar exocytosis C
glands that are quiescent until 2. Passive diffusion
puberty. Once active, they secrete 3. Decapitation secretion
their contents by what 4. Autocrine secretion
mechanism? 5. Holocrine secretion
155 What is the most abundant 1. Type I Collagen C
collagen found on fetal skin? 2. Type II Collagen
3. Type III Collagen
4. Type IV Collagen
5. Type VII Collagen
156 Once a keratinocyte leaves the 1. 7 days B
basal cell layer, the normal transit 2. 14 days
time to stratum corneum is at 3. 21 days
least: 4. 28 days
5. 35 days
157 Sebaceous glands are located in 1. Nipple C
each of the following locations 2. Labia minora
except: 3. Palms
4. Eyelids
5. Buccal mucosa
158 In which of the following locations 1. Stria vascularis of the ear E
would you be least likely to identify 2. Iris
melanocytes? 3. Leptomeninges
4. Retina
5. Pericardium
159 Which of the following is true 1. It is pathogenic in cicatricial pemphigoid B
regarding BPAg1? 2. It is a member of the plakin family
3. It is pathogenic in pemphigoid gestationis
4. It is not pathogenic in paraneoplastic
pemphigus
5. It coprecipitates with plakoglobin
160 Surgery, Parturition, Fever, 1. Telogen effluvium A
Kwashiokor and Hypervitaminosis 2. Anagen effluvium
A are all causes of: 3. Alopecia areata
4. Androgenetic alopecia
5. Both telogen and anagen effluvium
161 Which of the following glands is not 1. Sebaceous glands A
under neural control? 2. Apocrine glands
3. Eccrine glands
4. Salivary glands
5. Ceruminous glands

16
162 The formation of granulation tissue 1. Neutrophils B
depends on the presence of: 2. Fibronectin
3. Collagen type I
4. Platelets
5. Collagen type IV
163 The function of 1. Regulate water-binding capacity A
glycosaminoglycans/proteoglycans 2. Interact with dermal dendrocytes
in the dermis is: 3. Facilitate COLVII binding to the anchoring
plaques
4. Facilitate mast cell degranulation
5. Regulate lymphocyte trafficking
164 The main collagen component of 1. Collagen IV A
the basement membrane is: 2. Collagen III
3. Collagen I
4. Tenascin-X
5. Collagen VII
165 What is the significance of the 1. It is the location of the insertion of the D
critical line of Auber? erector pili muscle
2. the bulk of the mitotic activity in the hair
occurs above this line
3. the inner root sheath is formed above this
line
4. It is the widest diameter of the hair bulb
5. It is where keratinization first occurs in the
hair
166 Sebaceous glands secrete sebum 1. Holocrine mechanism A
via: 2. Exocrine mechanism
3. Endocrine mechanism
4. Exostosis
5. Mecrocrine mechanism
167 Sebaceous glands: 1. Respond to chemical stimuli such as A
hormones
2. Respond to cholinergic neural activity,
exclusively
3. Respond to adrenergic neural activity,
exclusively
4. Respond to both adrenergic and cholinergic
stimuli
5. Respond to the local release of cytokines
from inflammatory cells
168 The classical types of Ehlers- 1. Elastin D
Danlos Syndromes (Type 1 and 2) 2. Fibrillin 1
lead to varying degrees of 3. ABCC6
hyperextensible skin, easy 4. Collagen V
bruising, wide, atrophic scars, and 5. Cystathionine beta-synthase
hypermobile joints. The underlying
defect in this disorder is a mutation
in:
169 Choose the correct answer 1. In black and brown skin the melanosomes C
regarding melanin and skin color: are smaller in diameter and length
2. Facultative skin color is the amount of
cutaneous melanin pigment generated according
to cellular genetics
3. In white skin the melanosomes form groups
within the secondary lysosomes
4. Eumelanin produces a yellow chromophore
5. The number of melanocytes increases with
one exposure to UVA/visible light
170 Which of the following markers are 1. CD20 E
specific and reliable for Merkel 2. HMB-45

17
cells? 3. CD34
4. CD3
5. CK20
171 Meibomian glands are: 1. Modified sebaceous glands A
2. Found everywhere except on the palms and
soles
3. Sebaceous lobules that feed into a
lactiferous duct
4. Modified ceruminous glands
5. Modified apocrine glands
172 Glomus cells are primarily found: 1. on hands/feet A
2. on the trunk
3. on the lateral thighs
4. on the genital skin
5. on the face
173 Eccrine glands are found in all the 1. Axillae C
following areas of the body except: 2. Palms
3. Labia minora
4. Scalp
5. Cutaneous lip
174 Merkel cells are mechanoreceptors 1. Keratin 7 B
found in areas of high-tactile 2. Keratin 20
sensitivity. This 3. S-100
immunohistochemical marker is 4. Factor XIIIa
restricted to Merkel cells in the skin 5. LYVE-1
and is thus a reliable marker for
these cells:
175 LEMD3 is mutated in which of the 1. Buschke Ollendorf A
following syndromes? 2. Albright Hereditary Osteodystrophy
3. Goltz
4. McCune-Albright
5. Menkes
176 BP230 is a member of of which of 1. Plakin A
the following families? 2. Integrin
3. Laminin
4. Collagen
5. Elastin
2.Immunodermatology
# Question Answers Corre
ct
Answ
er
1 The human leukocyte antigen that has the 1. HLA-B51 D
closet association with psoriasis is: 2. HLA-DQ3
3. HLA-B27
4. HLA-Cw6
5. HLA-DR1
2 Herpes simplex virus-related erythema 1. HLA-B7 D
multiforme has been associated with an 2. HLA-B8
increased frequency of: 3. HLA-B13
4. HLA-B15
5. HLA-B27
3 The most common autoimmune disorder in 1. Autoimmune thyroiditis A
patients with chronic hepatitis C infection is: 2. Idiopathic thrombocytopenic
purpura
3. Rheumatoid arthritis
4. Sj�gren�s syndrome
5. Systemic lupus erythematosus
4 The best screen for classical pathway 1. CD4/CD8 ratio B
complement deficiency or dysfunction is: 2. CH50
3. C2 esterase levels
4. Total C4 levels

18
5. Serum electophoresis

5 Which virus is most closely associated with 1. Human herpes virus 2 D


Kaposi�s sarcoma in HIV-infected patients? 2. Cytomegalovirus
3. Human herpes virus 6
4. Human herpes virus 8
5. Epstein-Barr virus
6 Which of the following diseases occur with an 1. Psoriasis D
increased frequency in persons deficient in C2? 2. Dermatitis Herpetiformis
3. Androgenetic Alopecia
4. DLE
5. Leiner's disease
7 Which T-cell subset is commonly found in 1. CD4+/CD7+ D
S�zary syndrome? 2. CD8+/CD7-
3. CD4-/CD7+
4. CD4+/CD7-
5. CD8+/CD7+
8 Which of the following is not true about the 1. UV radiation causes an increase in A
effects of ultraviolet radiation on the immune number of Langerhans cells in the
system? epidermis
2. UV radiation causes nuclear DNA
damage
3. Effects can be demonstrated by the
example of reactivation of latent herpes
simplex infection after sun exposure
4. UV radiation acts to suppress the
immune system both locally and
systematically
5. Effects can be demonstrated by the
ability of an antigen to induce an allergic
hypersensitivity reaction when applied to
skin which has been exposed to low
doses of UV radiation
9 Major histocompatibility complex (MHC) Class I 1. Are inducible on keratinocytes B
molecules: 2. Complexed with antigen trigger
cytotoxic T cells
3. Are recognized by receptors on
CD4+ T cells
4. Bear peptides derived from
pathogens taken up into vesicles
5. All of the above
10 Which cytokine is the main macrophage- 1. TNF B
activating cytokine? 2. IFN-gamma
3. IL-4
4. IL-10
5. lymphotoxin
11 Immunocytomas are: 1. Low grade B-cell lymphomas A
2. Aggressive B-cell lymphomas
3. Low grade T-cell lymphomas
4. Aggressive T-cell lymphomas
5. NK cell lymphomas
12 A previously healthy child presents with 1. Perivascular IgA A
palpable purpura, arthritis, and vomiting. You 2. P-ANCA autoantibodies
suspect a hypersensitivity vasculitis 3. Granulomas and eosinophilia
characterized by: 4. Infiltration of destruction of vessels
by atypical lymphocytoid and
plasmacytoid cells
5. Nectrotizing granulomatous
vasculitis
13 Efalizumab is an antibody which is directed 1. CD40 D
against LFA1 on the T-cell, blocking this 2. B7
molecule's interaction with: 3. LFA3
4. ICAM-1
5. P-selectin
14 What cytokine is most critical for the 1. Interleukin-2 C

19
development and maturation of eosinophils. 2. Interleukin-4
3. Interleukin-5
4. Interferon-alpha
5. Interferon-gamma
15 Which of the follwing is a chemotactic factor 1. TNF C
for eosinophils? 2. IL2
3. C5a
4. Plasminogen activator
5. IL8
16 When attempting to identify Langerhans cells 1. CD1 A
in a specimen, which marker is most helpful? 2. CD4
3. CD7
4. CD8
5. CD20
17 Subacute cutaneous lupus erythematosus has 1. Phenytoin C
been associated with the ingestion of which of 2. Allopurinol
the following drugs? 3. Terbinafine
4. Trimethoprim/sulfamethoxazole
5. Auranofin
18 Anti-Ro (SS-A) antibodies are most commonly 1. Mixed connective tissue disease E
found in: 2. Eosinophilic fasciitis
3. Drug-induced systemic lupus
erythematosus
4. Homozygous C2 deficiency
5. Neonatal lupus erythematosus
19 Of the following complement components, the 1. C3 B
most powerful neutrophil chemoattractant is: 2. C5a
3. C3a
4. C5b, C6, C7, C8, C9
5. C4a
20 Which of the following suture materials 1. Surgical gut D
induces the least inflammation? 2. Polyglycolic acid (Dexon)
3. Polyglycan 910 (Vicryl)
4. Polypropylene (Prolene)
5. Silk
21 Which of the following TH2 cytokines is a B cell 1. IL-4 A
growth factor? 2. IL-5
3. IL-10
4. IL-13
5. IFN-gamma
6.
22 This skin disease has been shown to be 1. Atopic dermatitis A
associated with reduced Beta 2 defensin. The 2. Psoriasis
diagnosis is: 3. Lepromatous leprosy
4. Tuberculoid leprosy
5. Subacute cutaneous lupus
erythematosus
6.
23 Linear IgA disease is most closely associated 1. Erythromycin B
with which of the following medications? 2. Vancomycin
3. Streptomycin
4. Azithromycin
5. Clarithromycin
6.
24 Eosinophils are typically seen in the cutaneous 1. Krabbe�s disease D
infiltrate of: 2. Kaposiform hemangioendothelioma
3. Kawasaki�s disease
4. Kimura�s disease
5. Ki-1 lymphoma
25 This patient says the rash is spreading and not 1. 1 week C
controlled with topical therapy. You give him a 2. 2 weeks
course of oral treatment that lasts: 3. 3 weeks
4. 4 weeks
5. 5 weeks
26 Which of the following diseases does NOT 1. Leishmaniasis which self-resolves B
respond with a Th1-type responses? 2. Lepromatous Leprosy

20
3. Tuberculoid Leprosy
4. Allergic contact dermatitis
5. Psoriasis
27 Angiocentric NK/T-cell lymphoma in children 1. Papular acrodermatitis of childhood D
may present as: 2. Acropustulosis of infancy
3. Childhood dermatomyositis
4. Hydroa vacciniforme
5. En coup de sabre
28 The target antigen of chronic bullous disease 1. BPAG 1 9230 kd BPAG) B
of childhood is: 2. 97 kd LAD-1 (a component of
BPAG2)
3. Collagen type VII
4. Alpha 6 beta 4 integrin
5. Plectin
29 Which of the following immunoglobulin crosses 1. IgA D
the placenta? 2. IgD
3. IgE
4. IgG
5. IgM
30 Anti-centromeric antibodies are associated 1. CREST A
with which rheumatologic disease? 2. Mixed connective tissue disease
3. SLE
4. Dermatomyositis/polymyositis
5. Progressive systemic sclerosis
31 In adult patients with Henoch-Sch�nlein 1. Pulmonary hemorrhage D
purpura with IgA vasculitis, which of the 2. Hemorrhagic cystitis
following complications is most likely to occur? 3. Peripheral neuropathy
4. Mesangial nephropathy
5. Facial edema
32 Which monoclonal gammopathy is most 1. IgA A
commonly associated with erythema elevatum 2. IgD
diutinum? 3. IgE
4. IgG
5. IgM
33 Psoriasis affects over 2% of the world�s 1. HLA-DR4 C
population and has a strong association with 2. HLA-DR1
which HLA class I haplotype? 3. HLA-CW6
4. HLA-B27
5. HLA-DQ6
34 Imiquimod induces which of the following 1. Interleukin-10 B
cytokines? 2. Interferon-alpha
3. Interleukin-2
4. Interleukin-4
5. Interleukin-5
35 Histamine is a biologic amine produced by 1. Monocytes E
which of the following cells? 2. Eosinophils
3. Basophils
4. Platelets
5. Basophils and Platelets
36 With which HLA type is psoriasis most 1. HLA-B51 E
definitively linked? 2. HLA-B8
3. HLA-DR4
4. HLA-DR3
5. HLA-Cw6
37 Which of the following HLA alleles is most 1. HLA-B27 B
strongly associated with Behcet�s disease? 2. HLA-B51
3. HLA-DQw2
4. HLA-DR1
5. HLA-DR4
38 All of the following statements regarding major 1. MHC class II molecules bind stably A
histocompatibility complex molecules are true to peptides derived from proteins
EXCEPT: sythesized and degraded in the cytosol
2. MHC class I molecules bearing viral
peptides are recognized by cytotoxic T-
cells that subsequently kill the infected
cell
3. Class I molecules such as HLA-A, B,

21
and C, are present on all nucleated cells
4. The MHC is located on chromosome
6 in humans
5. MHC Class II molecules bearing
peptides are recognized by TH1 or TH2
cells.
39 The gene for NEMO (NF-kappa bets essential 1. Papillon-Lef�vre syndrome E
modulator) is mutated in: 2. Pachyonychia congenital type II
3. Dyskeratosis congenital
4. Noonan�s syndrome
5. Incontinentia pigmenti
40 Topical tacrolimus and pimecrolimus are used 1. Sodium C
to treat atopic dermatitis and other 2. Potassium
inflammatory skin conditions. On which of the 3. Calcium
following ions is the inflammatory pathway 4. Selenium
blocked by these medications dependent? 5. Zinc
41 Which of the following is a criterion for the 1. Inflammatory bowel disease B
diagnosis of Behcet'sdisease? 2. Uveitis
3. Conjunctivitis
4. Nasal septal perforation
5. Lobular panniculitis
42 All of the following statements regarding 1. NK cells have properties of innate B
Natural Killer (NK) cells are true EXCEPT: and acquired immunity
2. NK cells express CD 2 molecules
3. NK cells are large granular
lymphocytes
4. NK cells mediate tumor lysis
5. NK cells mediate lysis of viral-
infected cells
43 Major histocompatibility complex class I 1. Peptides derived from proteins A
molecules bind to: synthesized and degraded in the cytosol
2. Peptides derived from proteins
degraded in endocytic vesicles
3. Peptides external to the cell
membrane
4. Immunoglobulin E
5. None of these answers are correct
44 Which cytokine is most important in recruiting 1. Interleukin-1 D
neutrophils? 2. Interleukin-2
3. Interleukin-4
4. Interleukin-8
5. Interleukin-10
45 Which of the following dietary supplements 1. Vitamin A D
may inhibit platelet function? 2. Vitamin C
3. Vitamin D
4. Vitamin E
5. Vitamin K
46 Sensation is intact in this lesion, but a Fite 1. IL-4 D
stain is positive. This lesions is associated with 2. IL-5
which of the following: 3. IL-10
4. All of these answers are correct
5. None of these answers are correct
47 Which cytokine is predominantly responsible 1. TNF-alpha A
for the Jarish-Herxheimer reaction? 2. IL-2
3. IL-5
4. IL-6
5. IL-12
48 All the following conditions exhibit a T-helper 1. Psoriasis B
cell 1 (Th1) cytokine secretion profile except: 2. Systemic lupus erythematosus
3. Granulomatous leprosy
4. Rheumatoid arthritis
5. Multiple sclerosis
49 All of the following statements regarding 1. They are highly phagocytic A
Langerhans cells are true EXCEPT: 2. They express CD1 on their surface
3. They are found in some areas of
lymph nodes and spleen
4. They have a high density of Class II

22
molecules on their surface
5. None of these answers are correct
50 Which of the following cytokines is a general 1. IL-4 C
down-regulator of TH1 immunity? 2. IL-5
3. IL-10
4. TNF
5. IFN-gamma
51 All of the following stains can be reactive in 1. CDXIIIa A
this condition except 2. CD31
3. CD34
4. Ulex europaeus
5. Factor VIII-related antigen
52 Which of the following is true of Langerhan 1. They are the primary antigen A
cells? presenting cell in the epidermis
2. They are primarily involved in the
innate immune response
3. They do not express the CD1a
marker
4. They contain intranuclear birbeck
granules
5. They are increased on the palms,
soles, genitalia, and buccal mucosa
53 This woman should have a workup for: 1. Lymphoma D
2. Nephrolithiasis
3. Pancreatic cancer
4. Hemochromatosis
5. Thalassemia
54 Which of the following is associated with 1. Essential mixed cryoglobulinemia A
hepatitis C infection? 2. Rheumatoid arthritis
3. Relapsing polychondritis
4. Wegener�s granulomatosis
5. Dermatomyositis
55 The pharmacologic activity of tacrolimus 1. Phosphorylation of NFAT (nuclear E
includes: factor of transcription)
2. Binding and inhibition of NF kappa B
3. Inhibition of interleukin-1 gene
transcription
4. Activation of calcineurin
5. Inhibition of interleukin-2 gene
transcription
56 Proteins in the alternate complement pathway 1. Factor B E
include: 2. properdin
3. C3
4. Factor B and properdin
5. Factor B, properdin and C3
57 All of the following statements regarding mast 1. Mast cells reside near small blood E
cells are true EXCEPT: vessels
2. Mast cells protect mucosal surfaces
against pathogens
3. Mast cells release substances that
affect vascular permeability
4. Mast cells have receptors for certain
fragments of complement on their
surface
5. None of the above (all are true)
58 Which of the following immune-mediated 1. Clonal expansion of CD8+ T cells A
events has been demonstrated in psoriasis 2. Decrease dermal Langerhans cells
vulgaris? 3. Downregulation of keratin 16
4. Increase Th2 CD4+ T cells
5. Decreased production of interferon-
gamma
59 Which cytokine is chemotactic for neutrophils? 1. IL-2 E
2. IL-3
3. IL-5
4. IL-6
5. IL-8
60 All of the following statements regarding 1. Neutrophils' major function is A

23
neutrophils are true EXCEPT: antigen presentation
2. Neutrophils have receptors for IgG
and complement
3. Neutrophils are granulocytes
4. Neutrophils are the most abundant
leukocytes
5. None of these answers are correct
61 Calcipotriene-induced improvement in 1. Interleukin-2 D
psoriasis is associated with increased lesional 2. Interleukin-8
levels of which cytokine? 3. Tumor necrosis factor
4. Interleukin-10
5. Interluekin-12
62 Pemphigus is associated with which HLA 1. HLA-DRw6 A
type(s)? 2. HLA-B8
3. HLA-B51
4. All of these answers are correct
5. None of these answers are correct
63 Which cytokine is not upregulated in atopic 1. IL-13 E
dermatitis patients? 2. IL-4
3. IL-5
4. IL-10
5. IFN-gamma
64 Purpuric contact dermatitis is most likely to be 1. Nickel B
associated with: 2. Formaldehyde
3. P-phenylenediamine
4. Propylene glycol
5. Sorbic acid
65 Serum IgA antibodies to tissue 1. Bullous pemphigoid E
transglutaminase occur in: 2. Linear IgA disease
3. Pemphigus foliaceus
4. Bullous lupus erythematosus
5. Dermatitis herpetiformis
66 Dermatitis herpetiformis is associated with 1. HLA-B8 D
which HLA type(s)? 2. HLA-DR3
3. HLA-DQw2
4. All of the above
5. None of the above
67 Which of the following best describes the 1. Inhibition of calcineurin-mediated C
mechanism of action for infliximab? dephosphorylation of transcription
factors
2. Inhibition of retinoic acid 4-
hydroxylase activity
3. Inhibition of tumor necrosis factor
alpha activity
4. Selective elimination of activated T
cells via binding to high affinity IL-2
receptor
5. Selective T-cell up-regulation of IL-4
and IL-5 production
68 Which common contact allergen is detected 1. Benzocaine D
via the dimethylglyoxime test? 2. Chromates
3. Formaldehyde
4. Nickel
5. Rhus
69 This disease is most associated with: 1. Gamma interferon A
2. IL-4
3. IL-5
4. IL-13
5. None of these answers are correct
70 Toll-like receptors (TLRs) have been found to 1. Clobetasol E
play an important role in innate immunity. This 2. Tacrolimus
has been utilized in the development of 3. Cyclosporine
medications frequently used in dermatology. 4. 5-Fluorouracil (5-FU)
The mechanism of what medication involves 5. Imiquimod
activation of TLR7.
71 MHC Class II molecules are present on which of 1. B cells A
the following cell types: 2. T cells

24
3. NK cells
4. Mast cells
5. All of the above
72 Which cytokine is upregulated in lesions of 1. IL 2 A
tuberculoid leprosy? 2. IL 4
3. IL 5
4. IL 10
5. None of the answers are correct
73 This patient recently developed this rash. You 1. Chamomile E
decide to patch test her, but in the meanwhile 2. Primin
you tell her to avoid: 3. Abietic acid
4. Benzocaine
5. Cinnamon
74 An 8 month-old baby with diffuse purpura is 1. CD4 + T cells D
admitted to the hospital for her third episode 2. Natural killer cell activation
of bacterial meningitis. Which component of 3. CD8 + T cells
her immune system is impaired? 4. Complement activation
5. Antibody production
75 The most useful pair of immunohistochemical 1. Synaptophysin, chromogranin B
stains to distinguish between Dermatofibroma 2. CD34, factor XIIIa
protuberans (DFSP) and a fibrous histiocytoma 3. CD34, CD31
would be: 4. CD31, CD3
5. Vimentin, synaptophysin
76 Naive T cells express which of the following 1. CD19 E
surface molecules: 2. CD20
3. CD79
4. CD45RO
5. CD45RA
77 Which cytokine is up-regulated in this 1. IL-2 A
geometric, eczematous dermatitis? 2. IL-4
3. IL-5
4. IL-7
5. IL-10
78 Which of the following is an example of a 1. Allergic contact dermatitis A
delayed hypersensitivity reaction? 2. Anaphylaxis
3. Latex allergy
4. Transfusion reaction
5. Serum sickness
79 Which systemic anti-inflammatory agent 1. Etanercept C
specifically blocks the ability of T cells to leave 2. Infliximab
the vasculature and enter the skin? 3. Efalizumab
4. Alefacept
5. None of the above
80 Relapsing polychondritis is an autoimmune 1. I B
disease associated with immunity to which 2. II
type of collagen? 3. III
4. IV
5. VII
81 A 26 year-old man presents with a history of 1. Parvovirus B19 B
recurrent episodes of targetoid, erythematous, 2. Herpes simplex virus
edematous macules, patches, and plaques on 3. Coxsackievirus
the arms, legs, palms, and soles. The most 4. Cytomegalovirus
likely etiologic agent is: 5. Human immunodeficiency virus
82 The endothelial ligand for cutaneous 1. Intercellular adhesion molecule 1 C
lymphocyte antigen (CLA) is: (ICAM-1)
2. L-selectin
3. E-selectin
4. Vascular cell adhesion molecule
(VCAM-1)
5. Leukocyte functional antigen (LFA 3)
83 Gene rearrangement analysis is usedful for 1. Lymphocyte clonality in mycosis A
determining: fungoides
2. Lymphocyte activity
3. Gene Function
4. Gene Mutations
5. T cell receptor status
84 The most definitive HLA association with 1. HLA-Cw6 A

25
psoriasis is: 2. HLA-B27
3. HLA-B13
4. HLA-B17
5. HLA-B37
85 Allergic contact dermatitis is caused by T-cell 1. Melanocyte B
response to topical exposure to compounds 2. Langerhans Cell
that form complexes with host proteins 3. B-cell
(haptens). Which cell is responsible for initial 4. Mast Cell
sensitization of the T-cells? 5. Macrophage
86 Which of the following causes a photoallergic 1. Ascorbic acid C
contact dermatitis that is exacerbated by UVA 2. Titanium dioxide
radiation? 3. Oxybenzone
4. Zinc oxide
5. Dihydroxyacetone
87 This patient has a lichenified plaque in the 1. Patch testing A
lower mid abdomen as well as these two 2. A steroid
exczematous plaques. This patient needs: 3. An antiviral
4. A KOH scraping
5. An antifungal
88 Which cytokine is responsible for activating 1. Interleukin 4 B
natural killer cells? 2. Interleukin 2
3. Interferon-alpha
4. Terferon-gamma
5. Tumor necrosis factor-alpha
89 The classical complement pathway: 1. Can be activated in the absence of C
antibody
2. Can be activated by IgG4
3. Can be activated by IgM
4. Includes C3 and factor B
5. Does not cause membrane damage
90 Psoriatic arthritis is most commonly associated 1. HLA-B27 A
with which HLA? 2. HLA-Cw6
3. HLA-Aw19
4. HLA-Bw35
5. None of these options are correct
91 Which of the following substances is located in 1. Eosinophilic cationic protein D
the core of an eosinophil? 2. Eosinophil-derived neurotoxin
3. Eosinophil peroxidase
4. Major basic protein
5. Chymase
6.
92 Which component of the T cell receptor is 1. D-beta E
associated with superantigen recognition? 2. J-alpha
3. J-beta
4. V-alpha
5. V-beta
93 Which of the following is the target antigen in 1. Desmoglein 3 A
pemphigus vulgaris? 2. Desmoplakin
3. Cadherin
4. Type XVII collagen
5. Desmoglein 1
94 A male infant presents with thrombocytopenia, 1. Ataxia telangiectasia E
eczema, and recurrent infections. You suspect 2. Di-George anomaly
which of the following immunodeficiency 3. Hyper-IgM syndrome
disorders? 4. Leiner’s disease
5. Wiskott-Aldrich syndrome
95 Anti-epiligrin (laminin 5) antibodies may be 1. Pemphigoid gestationis D
seen in: 2. Pemphigus vegetans
3. Fogo selvagem
4. Cicatricial pemphigoid
5. Paraneoplastic pemphigus
96 All of the following statements regarding Toll 1. Toll receptors are present on B
receptors are true EXCEPT: macrophages and dendritic cells
2. Toll 2 receptors are typically
activated by lipopolysaccharide
3. Toll 4 receptors are typically
activated by gram negative bacteria

26
4. Nuclear factor kappa B (NFKB) is the
final common pathway of toll receptors
5. None of the above (all are true)
97 For this patient, you request that the lab 1. Monkey esophagus C
perform indirect immunofluorescence using 2. Rat bladder
what substrate? 3. Guinea pig esophagus
4. Mouse epithelium
5. Hep-2 cells
98 Which of the following cytokines, together with 1. IL-5 C
IL-4, promotes isotype switching from IgM to 2. IL-10
IgE? 3. IL-13
4. TNF
5. IFN-gamma
99 The immunoglobulin most commonly found in 1. IgA A
mucous secretions is: 2. IgD
3. IgE
4. IgG
5. IgM
10 T-cell anergy occurs if: 1. Stimulation by a MHC Class III B
0 molecule is involved
2. MHC/TCR engagement occurs
without costimulatory molecules
3. FasL is bound on the T-cell
4. MHC Class I or II is bound in the
presence of IL-2
5. A HLA-DM facilitator is not involved
with the binding
10 Which is not a feature of mast cells? 1. Expresses c-kit B
1 2. Responds to RANTES
3. Produces IL-8
4. Produces prostaglandin D2
5. Stains with napththol chloro-acetate
esterase
10 Elaboration of which of the following cytokines 1. Interferon-8 (gamma) D
2 is characteristic of TH2 response? 2. Interleukin-1
3. Interleukin-2
4. Interleukin-4
5. Interleukin-12
10 Dermatitis Herpetiformis is most commonly 1. HLA-DR3 E
3 associated with which HLA? 2. HLA-B27
3. HLA-B8
4. HLA-Bw35
5. HLA-DQ(A1*0501, B1*02)
10 Keratinocytes express what class of major 1. MHC Class I A
4 histocompatibility complex under normal 2. MHC Class II
conditions? 3. MHC Class III
4. MHC Class IV
5. MHC Class V
10 A deficiency of this complement component 1. C1 Esterase Inhibitor B
5 may result clinically in susceptibility to 2. C3
pyogenic infections, glomerulonephritis, and 3. C4
partial lipodystrophy: 4. C50
5. Properidin
10 Which antibody is most commonly found in 1. IgA C
6 circulation of patients with atopic dermatitis? 2. IgD
3. IgE
4. IgG
5. IgM
10 Which of the following immunoglobulins 1. IgM E
7 cannot activate the complement pathway? 2. IgG1
3. IgG2
4. IgG3
5. IgG4
10 The major histocompatibility complex 1. Class II molecules are present on all D
8 (MHC)consists of a linked set of genes nucleated cells
encoding for MHC Class I, Class II, Class III, and 2. Class I molecules are present on
Class IB. Which of the following is/are true: erythrocytes

27
3. Class I molecules are expressed on
B8 cells, monocytes and dendritic cells
4. The level of Class I and II expression
can be modulated by cytokines
5. The MHC region is located on
chromosome 17 in humans
10 All of the following statements are true 1. Neutrophils have receptors for IgG C
9 regarding cells of the innate immune system 2. Basophils are a type of granulocyte,
EXCEPT: as are neutrophils
3. IL-5 downregulates the functions of
eosinophils
4. Langerhans cells are poorly
phagocytic
5. Langerhans cells express CD1 on
their surface
11 This patient developed an acute vesicular rash 1. Ginkgo fruit A
0 after eating a mango. She has returned for a 2. Croton
routine follow-up. She needs to be careful of 3. Ragweed
exposure to: 4. Tea tree oil
5. All of these answers are correct
11 Which antibody can bind the FcER1 portion of 1. IgA C
1 mast cells, basophils, Langerhans cells, dermal 2. IgD
dendritic cells? 3. IgE
4. IgG
5. IgM
11 Which paraprotein is found most commonly in 1. IgG B
2 patients with pyoderma gangrenosum? 2. IgA
3. IgM
4. IgE
5. IgD
11 The human major histocompatibility complex 1. 2 B
3 (MHC) is located on chromosome: 2. 6
3. 9
4. 11
5. 17
11 Chronic idiopathic urticaria is associated with 1. HLA-DR4 D
4 which HLA type(s)? 2. HLA-DRB4 53
3. HLA-DQ8
4. all of the above
5. none of the above
11 A very common pentadecacatechol sensitizer 1. Gingko fruit pulp C
5 is found in all of the following plants/plant 2. Poison sumac
components except: 3. Mango fruit pulp
4. Cashew oil
5. Indian marking nut oleoresin
11 Which of the following features of IgG is true? 1. IgG is not an opsonizing antibody B
6 2. IgG is the only class of
immunoglobulin that can pass through
the placenta
3. IgG cannot activate the complement
cascade
4. IgG represents 15% of the total
protein in serum
5. IgG is the second immunoglobulin
synthesized by the fetus
11 Patients with a type I reaction to latex may 1. Avocado A
7 have cross-reactions with which of the 2. Horseradish
following foods? 3. Cashews
4. Parsnips
5. Garlic
11 Natural killer (NK) cells eliminate infected cells 1. NK cells adhere to and kill target D
8 in all of the following ways except: cells coated with IgG
2. NK cells secrete perforins
3. NK cells secrete granzyme
4. NK cells secrete myeloperoxidase
5. NK cells do not target cells
expressing major histocompatibility

28
(MHC) class I molecules
11 The most likely target for exfoliative toxin A in 1. Desmoglein 3 C
9 bullous impetigo is: 2. Laminin 5
3. Desmoglein 1
4. Collagen VII
5. Cesmocollin
12 Which of the following B cell receptors is 1. CD40 A
0 involved in immunoglobulin isotype switching? 2. CD19
3. CD20
4. CD154
5. CD22
12 What is the best screening test for hereditary 1. C1 esterase B
1 angioedema? 2. C4
3. CH50
4. C2
5. C3
12 The main cytokine secreted by Th1 CD8+ 1. IL-1 C
2 effector T-cells is: 2. IL-2
3. IFN-gamma
4. IL-4
5. IL-5
12 Which cytokine is primarily responsible for 1. IL-1 E
3 stimulation of neutrophils? 2. IL-4
3. IL-5
4. IL-6
5. IL-8
12 The antibody produced in the early stages of 1. IgA E
4 antibody responses is: 2. IgD
3. IgE
4. IgG
5. IgM
12 This lesion is hypoesthetic and is associated 1. Gamma interferon D
5 with which of the following: 2. Interleukin 2
3. Interleukin 12
4. All of these answers are correct
5. None of these answers are correct
12 Mononuclear phagocytes residing in tissues: 1. Are called macrophages E
6 2. Phagocytose foreign antigens and
degrade them into peptides
3. Present antigen to T-cells
4. Produce cytokines, which recruit
other inflammatory cells
5. All of the above
12 Which systemic anti-inflammatory agent 1. Etanercept D
7 targets CD2+ activated T cells for apoptosis? 2. Infliximab
3. Efalizumab
4. Alefacept
5. None of the above
12 Which cytokine is responsible for fever in 1. IL-1 A
8 patients with sunburn? 2. IL-5
3. IL-10
4. IL-11
5. TNF-beta
12 This patient had antibodies to 180 kd antigen. 1. Bullous pemphigoid A
9 The likely diagnosis is: 2. Pemphigus vulgaris
3. Pemphigus foliaceous
4. Erythema multiforme
5. Bullous lichen planus
13 Anti Jo-1 antibodies are directed against which 1. Topoisomerase D
0 of the following? 2. Lysyl oxidase
3. Gyrase
4. Histidyl transfer RNA synthetase
5. Telomerase
13 Herpes gestationis is most commonly 1. HLA-DR3 A
1 associated with which HLA? 2. HLA-B27
3. HLA-B51
4. HLA-DR9

29
5. HLA-DQ8
13 The putative mechanism of action of topical 1. Lymphokine production A
2 macrolide immunomodulators is inhibition of: 2. Prostaglandin secretion
3. Antigen presentation
4. Neutrophil migration
5. Lymphocyte migration
13 A patient had antibodies to desmoglein 3, but 1. Pemphigus vulgaris A
3 no antibodies to desmoglein 1 or desmplakin. 2. Pemphigus foliaceous
The likely diagnosis is: 3. Bullous pemphigoid
4. Dermatitis herpetiformis
5. Erythema multiforme

3. Genodermatoses
# Question Answers A
1 Mucosal neuromas, 1. Menin B
pheochromocytoma and medullary 2. RET proto-oncogene
thyroid carcinoma in a patient with a 3. PTEN
marfanoid body habitus is associated 4. BHD
with which of the following gene 5. STK11
defects?
2 Which of the following syndromes is 1. Sturge Weber disease C
associated with hematologic 2. Klippel-Trenaunay-Parks-Weber
abnormalities? 3. Kasabach-Merritt syndrome
4. Blue rubber bleb nevus
syndrome
5. Bloom�s syndrome
3 The finding of 'maltese crosses' in the 1. Alkaptonuria B

30
urine is characteristic of which of the 2. Fabry disease
following conditions? 3. Gaucher disease
4. Neimann-Pick disease
5. Hunter syndrome
4 Which of the following 1. IgA D
immunoglobulins is commonly 2. IgD
decreased in Wiskott-Aldrich 3. IgE
syndrome? 4. IgM
5. IgG
5 The nucleotide excision DNA repair 1. Bourneville's disease D
pathway is defective in which disease: 2. Severe combined deficiency
syndrome
3. Griscelli syndrome
4. Xeroderma pigmentosa
5. Sjogren-Larssen syndrome
6 Which type of epidermolysis bullosa 1. Weber-Cockayne C
simplex is associated with early death? 2. Generalized (Koebner)
3. Dowling-Maera
4. Ogna variant
5. Non-Herlitz variant
6.
7 Mosaic mutations in PTEN are seen in 1. Gardner syndrome B
which of the following conditions? 2. Proteus syndrome
3. Incontinentia pigmenti
4. Noonan syndrome
5. Beckwith-Wiederman syndrome
6.
8 Which of the following is a feature of 1. Congenital hypertrophy of the C
Neurofibromatosis type II? retinal pigment epithelium
2. Lisch nodules
3. Juvenile posterior subcapsular
lenticular opacities
4. Lester iris
5. Optic gliomas
9 A patient with colon cancer is 1. Keratoacanthomas A
diagnosed with Muir-Torre syndrome. 2. Seborrheic keratoses
Which of the following cutaneous 3. Tricholemmomas
lesions might the patient have? 4. Arsenical keratoses
5. Basal cell carcinomas
6.
10 An infant with doughy, redundant skin 1. Metaphyseal widening in the A
and short sparse hairs is likely to show long bones
which features on x-ray? 2. Sphenoid wing dysplasia
3. Periosteal thickening
4. Osteopoikilosis
5. Stippled epiphyses
11 Which ocular finding may be seen in a 1. Comma-shaped corneal D
patient with this skin condition? opacities
2. Retinitis pigmentosa
3. Congenital hypertrophy of the
retinal pigmented epithelium
4. Angioid streaks
5. Pingueculae
12 Homocystinuria is caused by a defect 1. Phenylalanine hydroxylase D
in: 2. Biotinidase

31
3. Holocarboxylase synthetase
4. Cystathione beta-synthetase
5. Gp91-phox
13 A patient with melanoma and a 1. Rhabdomyosarcoma A
malignant glioma is diagnosed with Li- 2. Adrenocortical carcinoma
Fraumeni syndrome. Which of the 3. Lung carcinoma
following tumors occurs most 4. Breast carcinoma
frequently in this disease? 5. Leukemia
14 Which keratins are expressed in the 1. Keratins 1 and 9 A
suprabasal palmoplantar epidermis? 2. Keratins 1 and 10
3. Keratins 4 and 13
4. Keratins 5 and 14
5. Keratins 8 and 18
15 The main cause of death in patients 1. Oral squamous cell carcinoma D
with dyskeratosis congenita is which of 2. Leukemia
the following? 3. Renal cell carcinoma
4. Pancytopenia
5. Atherosclerotic heart disease
16 What is the classic radiologic findings 1. Dural calcifications C
associated with this disorder? 2. Calcifications of the falx-cerebri
3. Tram-track calcifications of the
temporal and occipital cortex
4. Osteopatha striata
5. Osteopoikilosis
17 A 3 year-old boy presents with the 1. Chronic Granulomatous disease B
findings seen in the image. He also has 2. Wiskott-Aldrich syndrome
thrombocytopenia with purpura and a 3. Hyper-IgE syndrome
history of recurrent pyogenic bacterial 4. Severe combined
infections. What is the most likely immunodeficiency
diagnosis in this child? 5. Leiner syndrome
18 Which of the following condition is NOT 1. Connective tissue nevi A
found in Von-Hippel Lindau syndrome? 2. Bilateral retinal
hemangioblastomas
3. Cerebellar/CNS
hemangioblastomas
4. Renal cell carcinoma
5. Pheochromocytoma
19 What is the most common ocular 1. Angioid streaks A
findings seen in this condition? 2. Phakomas
3. Lisch nodules
4. Bitot’s spots
5. Coloboma
20 A patient with Crowe�s sign and an 1. Neurofibromatosis I A
optic glioma has which of the following 2. Neurofibromatosis II
disorders? 3. Watson syndrome
4. Tuberous sclerosis
5. Lester iris syndrome
21 Maffucci syndrome is has 1. Sporadic A
characteristic venous malformations of 2. Autosomal dominant
the distal extremities and benign 3. Autosomal recessive
endochondromas which can 4. X-linked dominant
compromise bone strength and lead to 5. X-linked recessive
chondrosarcomas. The defect causing
this believed to be the PTH/PTHrP type
I receptor which is inherited in which

32
manner?
22 Which cutaneous finding is seen in 1. Angular stomatitis C
patients with phenylketonuria? 2. Ichthyosis
3. Pigment dilution of hair and
skin
4. Phyrnoderma
5. Erosive diaper dermatitis
23 A patient with renal cell carcinoma 1. Von-Hippel-Lindau syndrome D
caused by mutations in fumarate 2. Cowden syndrome
hydratase deficiency likely suffers 3. Birt-Hogg-Dube syndrome
which of the following conditions? 4. Familial multiple cutaneous
leiomyomatosis
5. Multiple endocrine neoplasia
24 A patient presents with focal 1. Jadassohn-Lewandowsky A
symmetric palmoplantar keratoderma, syndrome
thickened, hyperkeratotic fingernails 2. Jackson-Lawler syndrome
and toenails with a "pincer" 3. Schafer-Branauer syndrome
appearance and frequent staph and 4. Pachyonychia congenita tarda
candida paronychial infections, 5. None of the options are correct
follicular hyperkeratosis of the knees
and elbows and oral leukokeratosis.
The patients mother and grandfather
both have similar skin findings. Which
syndrome is described?
25 An infant girl of short stature and 1. Webbed neck A
shortened 4th and 5th metacarpals is 2. Alopecia
being evaluated for coarctation of the 3. Hemangioma
aorta and horseshoe kidneys. Physical 4. Giant congenital melanocytic
examination most likely reveals: nevus
5. Arachnodactyly
26 A patient with 20 nail dystrophy, 1. Keratins 5 D
steatocystoma multiplex and natal 2. Laminin 5
teeth likely has a mutation in the 3. Plakophilin 1
genes coding for: 4. Keratins 6b & 17
5. Keratins 6 &16
27 Low-cystine content in hair and nails 1. Wilson�s disease C
may contribute to the phenotype seen 2. Menke�s Kinky Hair syndrome
in: 3. Tay Syndrome
4. Nethertons
5. Bjornstad
28 Epidermolysis bullosa with muscular 1. Keratins 5 and 14 B
dystrophy is caused by mutations in 2. Plectin
which of the following? 3. Loricrin
4. Collagen 7
5. Collagen 17
29 Which of the following hereditary skin 1. Costello syndrome A
disorders is associated with the RAS- 2. Rothmund-Thompson Syndrome
ERK-MAPK pathway? 3. Carney complex
4. Tuberous Sclerosis
5. Griscelli syndrome
30 What gene defect would you expect to 1. Pax3 A
find in a child with white forelock, 2. MITF
dystopia canthorum, and upper limb 3. SOX10
abnormalities? 4. Endothelin-3
5. C-kit proto-oncogene

33
31 Underlying defect for the disease 1. ATP2A2 B
shown in picture is 2. ATP2C1
3. BPAG1
4. BPAG2
5. Collagen type 17
32 Dental enamel pits are seen in which 1. Hypomelanosis of Ito C
of the following conditions? 2. Letterer-Siwe disease
3. Tuberous sclerosis
4. Jackson Sertoli syndrome
5. Hyper-IgE syndrome
33 Refsum syndrome is due to a 1. Diet low in green vegetables, A
deficiency in phytanyl coenzyme A dairy and ruminant fats
hydroxylase. Treatment for this 2. Diet high in green vegetables,
condition is: dairy and ruminant fats
3. Enzyme replacement
4. No treatment is available at this
time
5. Avoid phenylalanine
34 A 16 year-old girl presents with a 1. Lisch nodules C
family history of Gardner syndrome. 2. Lester iris
Her mother is very concerned that her 3. Congenital Hypertrophy of the
daughter may have the syndrome as it Retinal Pigment Epithelium
runs in her family and she has many 4. Angioid streaks
skin complaints. Gardner syndrome 5. Retinal detachment
has been linked to defects in beta-
catenin mediated transcription. Which
of the following ocular finding is
diagnostic for Gardner syndrome?
35 A teenage female presents with the 1. Urinalysis A
complaint of "nail fungus". On exam, 2. CBC
she has triangular lunulae, 3. Fasting lipids
palmoplantar hyperhidrosis, 4. Renal ultrasound
micronychia and an absent patella. 5. X-ray of the knees, elbows and
Which of the following screening tests pelvis
should you order first?
36 Mutation in lamin A (nuclear envelope 1. Peutz-Jeghers syndrome C
protein) has been found in: 2. Buschek-Ollendorf syndrome
3. Progeria (Hutchinson-Gilford)
4. Albright�s syndrome
5. Marfan syndrome
37 In chronic granulomatous disease, the 1. Assay for fumarate hydratase B
diagnosis is made by which of the 2. Nitroblue tetrazolium reduction
following tests? assay
3. Assay for sphingomyelinase
4. Skin biopsy
5. Assay for glucocerebrosidase
38 A 3-year old boy has frequent 1. Stat 3 gene A
abscesses, chronic ear infections, and 2. FBLN 5 gene
severe eczema. He has an older 3. IL2 receptor gene
brother with similar problems. What is 4. LEMD3 gene
the gene mutation? 5. WAS gene
39 Retinal hemangioblastomas are found 1. Osler-Weber-Rendu disease B
in which syndrome: 2. Von-Hippel Lindau disease
3. Kasabach-Merritt syndrome
4. Klippel-Trenaunay Weber

34
syndrome
5. Sturge-Weber syndrome
40 A 11 year-old female patient with 1. XAP101 gene B
hypoparathyroidism is referred to your 2. AIRE gene
clinic secondary to chronic 3. CGI-58 gene
mucocutanous candidiasis which is 4. ALOX gene
refractory to standard treatments. The 5. PAHX gene
patient also has malabsorption and
severe chronic diarrhea. You determine
that she has autoimmune
polyendocrinopathy-candiasis-
ectodermal dystrophy
syndrome(APECED). What is the gene
defect for APECED?
41 The x-linked recessive type of 1. Dyskerin A
dyskeratosis congenita is: 2. TERC
3. CDKN2A
4. PTEN
5. Menin
6.
42 Which genetic defect could explain 1. RecQL3 A
cutaneous findings in addition to 2. ERCC6
abnormal immunoglobulin levels, 3. WAS gene
recurrent respiratory infections, 4. NADPH oxidase
hypogonadism, and an increased risk 5. Adenosine deaminase
of leukemia and lymphoma? 6.
43 Most common malignancy to develop 1. Thyroid cancer B
in a patient with tricholemmomas, 2. Breast cancer
acral verrucous papules and cobble- 3. Colon cancer
stoning of buccal and gingival mucosa? 4. Lymphoma
5. Melanoma
44 The most common cutaneous 1. Sebaceous carcinoma B
neoplasm associated with Muir Torre 2. Sebaceous adenoma
Syndrome is: 3. Keratoacanthoma
4. Basal cell carcinoma with
sebaceous differentiation
5. Squamous cell carcinomas
45 The most common cutaneous 1. Eczema D
association with monilethrix is: 2. Hypopigmentation
3. Hyperpigmentation
4. Keratosis Pilaris
5. Atrophy
46 A 20-year old male with a history of 1. Cushingoid features B
pheochromocytoma and medullary 2. Marfanoid features
thyroid cancer presents with mucosal 3. Short stature
papules. His overall body appearance 4. Lipodystrophy
is most likely to demonstrate: 5. Unilateral limb shortening
47 An infant presents with poikiloderma 1. Glaucoma B
on his face, buttocks, arms and legs. 2. Cataracts
He is also noted to have a hypoplastic 3. Subcapsular lens displacement
thumb and no radius. Yearly 4. Copper deposition
ophthalmologic examination is 5. Macular degeneration
indicated because of the infant is at
risk for developing:
48 The combination of gastrointestinal 1. Nicolau-Balus syndrome C

35
polyposis, nail atrophy, alopecia, 2. Peutz-Jeghers syndrome
generalized pigmentation of skin, and 3. Cronkhite-Canada syndrome
melanotic macules of the fingers is 4. Cowden syndrome
characteristic of which of the following 5. Bannayan-Riley-Ruvalcaba
syndromes? syndrome
49 What phenotype results from a low 1. Waardenberg's syndrome type D
activity of double stranded RNA 2
adenosine deaminase? 2. Piebaldism
3. Tietz syndrome
4. dyschromatosis symmetrica
hereditaria
5. oculocutaneous albinism type 4
50 Anodontia is a bone finding seen in 1. Hypomelanosis of Ito A
which of the following conditions: 2. Letterer-Siwe disease
3. Tuberous sclerosis
4. Jackson Sertoli syndrome
5. Hyper-IgE syndrome
51 Dystrophic epidermolysis bullosa is 1. Sublamina densa A
associated with mutations in collagen 2. Stratum spinosum
VII. Trauma or friction induced 3. Lamina lucida
blistering in these patients have a 4. Stratum basale
plane a splitting in the: 5. None of these answers are
correct
52 A patient with multiple sebaceous 1. Retinal examination C
adenomas should be screened with 2. Laryngoscopy
which of the following examinations? 3. Colonoscopy
4. MRI of the spine
5. Renal ultrasound
53 A 16 year-old girl presents with a 1. 50% A
family history of Gardner syndrome. 2. 25%
Her mother is very concerned that her 3. 10%
daughter may have the syndrome as it 4. 75%
runs in her family and she has many 5. 90%
skin complaints. What is likelihood that
this girl has Gardner syndrome based
on what you now about the inheritance
pattern and the fact that her father is
unaffected and her mother is a
heterozygote for this condition?
54 Which malignancy is associated with 1. Colon cancer A
Cowden syndrome? 2. Renal cancer
3. Lung cancer
4. Ovarian cancer
5. Basal cell cancer
55 Which of the following disorders is 1. Leukocyte adhesion deficiency A
associated with delayed separation of type 1 (LAD-1)
the umbilical cord? 2. Immunedysregulation,
polyendocrinopathy, enteropathy,
x-linked (IPEX)
3. Severe combined
immunodeficiency disorder(SCID)
4. X-linked agammaglobulinemia
5. Myeloperoxidase deficiency
56 Which of the following is true regarding 1. Confetti-like macules are D
tuberous sclerosis? typically present at birth

36
2. Facial angiofibromas are the
most common cutaneous
manifestation
3. Hypomelanotic macules (ash
leaf spots) have a decreased
number of melanocytes
4. Periungual fibromas are
considered a major feature in the
diagnosis of tuberous sclerosis
5. 6 or more hypomelanotic
macules (ash leaf spots) are
considered a major feature in the
diagnosis of tuberous sclerosis
57 Cutaneous osteomas are seen in which 1. Waardenburg syndrome D
syndrome? 2. LEOPARD syndrome
3. Carney complex
4. Albright hereditary
osteodystrophy
5. Gaucher�s syndrome
58 The gene PTEN (phosphatase and 1. Cowden D
tensin homolog deleted on 2. Banayan-Riley-Ruvalcaba
chromosome ten) is implicated in 3. Proteus
which of the following syndromes? 4. All of the options are correct
5. None of the options are correct
59 A sporadic syndrome affecting 1. Rubinstein-Taybi syndrome A
transcriptional coactivator CREB- 2. Cornelia de Lange syndrome
binding protein is: 3. Nonne-Milroy disease
4. Maffucci syndrome
5. Blue rubber bleb nevus
syndrome
60 A double row of eyelashes is 1. Lymphedema-distichiasis A
associated with: syndrome
2. Cornelia de Lange syndrome
3. Rubinstein-Taybi syndrome
4. Russell-Silver syndrome
5. Hunters syndrome
61 Comma shaped corneal opacities are 1. Refsum Syndrome D
seen in what disease? 2. Sjogren-Larson Syndrome
3. Pseudoxanthoma elasticum
4. X-linked ichthyosis
5. Proteus syndrome
62 Xeroderma pigmentosum (XP) variant 1. Defective DNA nucleotide B
is different than classic XP in which of excision repair of the global
the following way? genome
2. Defective post-replication
repair
3. Increased chromosomal
breakage and sister chromatid
exchanges
4. Defective DNA nucleotide
excision repair of actively
transcribing genes
5. Low IgM
63 A 16 month-old girl presents with 1. Brain MRI D
patchy alopecia, whorled 2. Alkaline phosphatase

37
erythematous scaly eruption, and 3. Chest radiograph
asymmetric limb shortening. What 4. Bone films
laboratory or radiologic test may aid in 5. Complete blood count
diagnosis?
64 What is the inheritance pattern of 1. Autosomal dominant A
dermatosis with acantholysis? 2. Autosomal recessive
3. X-linked dominant
4. X-linked recessive
5. Sporadic
65 A patient presents with focal 1. Keratin 6a/16 A
symmetric palmoplantar keratoderma, 2. Keratin 6b/17
thickened, hyperkeratotic fingernails 3. Keratin 1/10
and toenails with a "pincer" 4. Keratin 2e/10
appearance and frequent staph and 5. None of these options are
candida paronychial infections, correct
follicular hyperkeratosis of the knees
and elbows and oral leukokeratosis.
The patients mother and grandfather
both have similar skin findings. What is
the defect in PC type I?
66 A 2 year old girl presents with sunken 1. Transcription factor D
eyes, large ears, microcephaly and a 2. Surface glycoprotein
photodistributed eruption on her face. 3. Lysosomal protease
Eye exam reveals �salt and pepper� 4. DNA helicase
retina. The gene responsible for this 5. Mismatch repair gene
syndrome codes for a:
67 Junctional epidermolysis bullosa with 1. The alpha-6 subunit of integrin C
pyloric atresia is associated with 2. The beta-4 subunit of integrin
mutations in: 3. Both subunits of integrin can
have mutations causing this type of
junctional epidermolysis bullosa
4. Plectin
5. Laminin 5
68 Which of the following is caused by a 1. Epidermolysis bullosa simplex C
defect in a gap junction protein? 2. Hailey-hailey
3. Erythrokeratoderma variabilis
4. Dyskeratosis congenita
5. Bullous ichthyosis of siemens
69 Which of the following elastic tissue 1. Cutis laxa D
diseases demonstrates calcified elastic 2. Marfan syndrome
fibers? 3. Anetoderma
4. Pseudoxanthoma elasticum
5. Buschke-ollendorf syndrome
70 Christ-Siemens-Touraine Syndrome is 1. NEMO B
most commonly linked with defects in 2. Ectodysplasin (EDA)
which of the following genes? 3. ERCC2
4. ATP7A
5. None of these options are
correct
71 Which of the following conditions is 1. Darier�s Disease A
worsened by ingestion of lithium? 2. Hailey-Hailey Disease
3. Haim-Munk syndrome
4. Hereditary lymphedema
(Nonne-Milroy disease)
5. Epidermolytic hyperkeratosis

38
72 On cutaneous exam, angiokeratoma 1. Sialodosis D
corporis diffusum is characteristic of 2. Fucosidosis
which of the following conditions? 3. Fabry disease
4. All of these options are correct
5. None of these options are
correct
73 What is the inheritance pattern of 1. Autosomal recessive C
chronic granulomatous disease? 2. Autosomal dominant
3. X-linked recessive
4. X-linked dominant
5. Sporadic
74 Premalignant leukoplakia of the oral 1. Bloom syndrome D
mucosa is associated with: 2. Werner Syndrome
3. Xeroderma Pigmentosum
4. Dyskeratosis Congenita
5. Rothmund-Thomson syndrome
75 Electron microscopic examination of a 1. Netherton’s syndrome C
hair shaft reveals a canal-like groove 2. Menke’s Kinky Hair syndrome
along the shaft of a triangular-shaped 3. Spun-glass hair
hair. This patient has: 4. Trichothiodystrophy
5. Bjornstad syndrome
76 A mentally retarded patient is found to 1. Hypertrichosis E
also have a malar rash and downward 2. Short stature
lens displacement. Another clinical 3. Multiple nevi
feature might include: 4. Periodontitis
5. Deep venous thromboses
77 Which of the following signs is not a 1. Bilateral vestibular A
criteria for the diagnosis of schwannomas
Neurofibromatosis type I? 2. Axillary freckling
3. Optic gliomas
4. Greater than 5 caf�-au-lait
macules
5. Sphenoid dysplasia
78 The development of which malignancy 1. Non-Hodgkin's lymphoma B
is most commonly associated with 2. Mycosis fungoides
lymphomatoid papulosis? 3. Multiple myeloma
4. Immunoblastic lymphoma
5. Waldenstrom's
macroglobulinemia
79 Crumpled ears are associated with 1. Ehlers-Danlos Syndrome C
which disorder? 2. Marfan syndrome
3. Congenital contractural
arachnodactyly
4. Cutis laxa
5. Buschke-Ollendorf syndrome
80 Which of the following is caused by a 1. MEN IIa E
mutation in a gene that leads to 2. Piebaldism
defective NF-KB activation? 3. Chediak-Higashi
4. Vohwinkel�s
5. Incontinentia pigmenti
81 Odontogenic cysts and palmoplantar 1. Gardner�s syndrome B
pits are seen in: 2. Gorlin Syndrome
3. Bloom�s Syndrome
4. Goltz Syndrome
5. Refsum syndrome

39
82 A 45 year old woman with history of 1. Telomere maintenance B
uterine fibroids presents with painful 2. Involved in the Kreb's cycle
firm papules of the left upper arm. 3. Inhibits mTOR signalling
Biopsy of one of these lesions reveals 4. Impairs folliculin function
smooth muscle fascicles. She notes a 5. Encodes a deubiquitinating
history of renal cell cancer in her enzyme
family. The gene implicated in this
syndrome has which of the following
functions?
83 Which malignancy is seen in 1. Osteosarcoma C
approximately 15-20% of people with 2. Angiosarcoma
the disease characterized by a defect 3. Chondrosarcoma
in a parathyroid hormone receptor 4. Rhabdomyosarcoma
protein? 5. Epitheliod sarcoma
84 A patient with pseudoherpetic keratitis 1. Tyrosine/Phenylalanine A
and a painful PPK would improve with 2. Glycine
a diet low in: 3. Cytosine
4. Biotin
5. Zinc
85 Collagen III is mutated in which type(s) 1. All of the answers are correct A
of Ehlers-Danlos syndrome (EDS)? 2. None of the answers are correct
3. Benign Hypermobile (type III)
4. Vascular (type IV)
5. Periodontitis (type VIII)
6.
86 A child presents with pretibial 1. Hearing test B
hyperpigmentation, ataxia, decreased 2. Slit-lamp eye exam
motor coordination, cirrhosis, and 3. EKG
decreased motor coordination. The 4. Colonoscopy
physical exam which would reveal the 5. Renal ultrasound
most specific finding for this disease is: 6.
87 Which of the following diseases is 1. Striated PPK D
caused by a mutation in a gap junction 2. Schopf-Schulz-Passarge
protein? syndrome
3. Mal de Meleda
4. Vohwinkel syndrome (classic)
5. Vohwinkel syndrome
(ichthyotic)
6.
88 The following enzyme defect is most 1. 3-beta-hydroxysteroid A
commonly seen in CHILD Syndrome. dehydrogenase
2. 3-beta-hydroxysteroid
isomerase
3. Aryl sulfatase E
4. NAD oxido reductase
5. DNA helicase
6.
89 Ehlers-Danlos Syndrome with 1. Lysyl hydroxylase E
congenital adrenal hyperplasia is 2. Collagen 5
caused by mutations affecting which of 3. Fibronectin
the following? 4. Lysyl oxidase
5. Tenascin-X
6.
90 A 3 year old boy has white hair of the 1. It is an autosomal recessive B
central frontal scalp and depigmented disorder of melanocyte

40
symmetrical patches on the knees development
since birth. No ocular abnormalities or 2. There is typically no
deafness are noted. What is true of progression of depigmented
this disorder? patches
3. Regression of the white forelock
has been noted
4. One form of this condition is
associated with iris pigmentary
abnormalities
5. Results from a mutation in a
tumor suppressor gene
6.
91 Hereditary Hemorrhagic Telangiectasia 1. Type I families have an increase A
syndrome is transmitted in an incidence of pulmonary
autosomal dominant fashion and can arteriovenous fistulas
have two variants. Type I is linked to 2. Type II families have an
defects in HHT1, the endoglin gene. increased incidence of pulmonary
Type II is linked to defects in HHT2, the arteriovenous fistulas
ALK1 gene. What feature that 3. Type I families have an
differentiates type I from type II increased incidence of hepatic
clinically? arteriovenous malformations
4. Type II families have a
decreased incidence of hepatic
arteriovenous malformations
5. None of the answers are correct
6.
92 A teenage female presents with the 1. LMX1B A
complaint of "nail fungus". On exam, 2. COL5A1
she has triangular lunulae, 3. EEC1
palmoplantar hyperhidrosis, 4. LKB1
micronychia and an absent patella. 5. NEMO
Which of the following gene defects is 6.
most likely in this patient?
93 Reticulate pigmentation of skin, 1. Dyskeratosis congenita B
poikiloderma, alopecia, nail atrophy, 2. Hoyeraal-Hreidarsson syndrome
premalignant oral leukoplakia, and a 3. Bloom syndrome
Fanconi-type pancytopenia resulting in 4. Cockayne syndrome
early death in addition to posterior 5. Wiskott-Aldrich syndrome
fossa malformations is characteristic of 6.
which of the following syndromes.
94 Findings of dysplastic nevi and 1. Breast malignancy C
melanoma inherited in an autosomal 2. Thyroid malignancy
dominant fashion is linked with which 3. Pancreatic malignancy
of the following? 4. Renal cell carcinoma
5. Colon carcinoma
6.
95 Which type of epidermolysis bullosa is 1. Dominant dystrophic B
associated with mitten deformities of 2. Recessive dystrophic
the hands? 3. Weber-Cockayne
4. Herlitz type
5. Generalized atrophic benign
epidermolysis bullosa (GABEB)
6.
96 Premature aging seen in Cockayne�s 1. Transglutaminase E
syndrome is caused by a mutation in 2. ATM gene

41
which gene? 3. Patched gene
4. NEMO gene
5. DNA helicase gene
6.
97 In one variant of epidermolysis bullosa 1. Plakoglobin C
simplex, those affected have muscular 2. Plakophilin
dystrophy in addition to the skin 3. Plectin
findings. Which protein mutation has 4. Desmoglein
been linked to this finding? 5. Desmocollin
6.
98 A patient with a port wine stain 1. Lymphatic and deep venous A
covering one enlarged leg likely has insufficiency
which of the following associated 2. Visceromegaly with
symptoms? omphalocele
3. Bilateral retinal
hemangioblastomas
4. Enchondromas
5. Distichiasis
6.
99 Patients with Darier�s disease are at 1. Kaposi�s varicelliform eruption A
increased risk for: 2. Melanoma
3. Decreased life span
4. Basal cell carcinoma
5. Lipid abnormalities
6.
10 Defects in Fibrillin 2 are linked with: 1. Congenital contractural A
0 arachnodactyly
2. Cutis Laxa
3. Arthrochalasis multiplex
congenita
4. Occipital horn syndrome
5. Lipoid proteinosis
6.
10 The best screening test for 1. Ferritin A
1 hemochromatosis is: 2. Hematocrit
3. Complete Blood Count
4. Transferrin
5. Copper levels
6.
10 A 7 year old boy presents to your office 1. Eyebrows and eyelashes are B
2 with short and sparse hair. He is also not affected
short for his age. A hair mount reveals 2. There is no increased risk of
alternating light and dark bands under skin cancer
polarized light. Which of the following 3. Amino acid analysis of hair
statements is correct regarding this reveals high sulfur content
condition? 4. There is no increased hair
fragility.
5. Dark bands represent air
6.
10 Which of the following conditions is 1. Chodrodysplasia punctata D
3 inherited in an X-linked dominant (XD) 2. CHILD Syndrome
manner? 3. Focal dermal hypoplasia
4. All of the answers are correct
5. None of the answers are correct
6.

42
10 What medication may exacerbate this 1. Phenytoin B
4 autosomally dominant, acnatholytic 2. Lithium
disorder? 3. Oral contraceptives
4. Anti-malarials
5. Corticosteroids
6.
10 Pruritus is Sjogren Larsson syndrome is 1. Bile salts C
5 attributed to accumulation of what 2. Histamine
molecule(s)? 3. Leukotriene
4. All of these answers are correct
5. None of these answers are
correct
6.
10 Menkes kinky hair syndrome is 1. Pili torti A
6 associated most commonly with which 2. Trichorrhexis nodosa
of the following hair abnormalities? 3. Trichorrhexis invaginata
4. Pili triangulati et canaliculati
5. None of these options are
correct
6.
10 The presence of natal teeth and pincer 1. Congenital syphillis D
7 nails suggests which disease entity? 2. Thalidomide exposure in utero
3. Incontinentia pigmenti
4. Pachyonychia congenita
5. Anhidrotic ectodermal dysplasia
6.
10 Papillon-Lefevre and Haim-Munk 1. Right-ventricular E
8 syndromes have which of the following cardiomyopathy
symptoms? 2. Pseudoainhum
3. Esophageal cancer
4. Eccrine syringofibradenoma
5. Periodontitis with tooth loss
6.
10 Painful crises and 'whorled' corneal 1. Homogentisic acid oxidase B
9 opacities are seen with which of the 2. Alpha-galactosidase A
following enzyme abnormalities? 3. Glucocerebrosidase
4. Iduronate sulfatase
5. Glucoronidase
6.
11 Lamellar ichthyosis is caused by a 1. 25% A
0 defect in transglutaminase 1. It can 2. 75%
present as a colloidion baby at birth 3. Only male offspring are
with subsequent large thick plates of affected
scale especially on flexures, ectropion 4. Only female offspring are
and eclabium. If two unaffected carrier affected
parents have a child, how likely is their 5. None of these answers are
child to have this condition? correct
6.
11 Epistaxis in early childhood to teens 1. Hereditary Hemorrhagic A
1 followed by multiple cutaneous and Telangiectasia syndrome
gastrointestinal telangectasias 2. Maffucci syndrome
describes which of the following 3. CREST syndrome
syndromes? 4. Ataxia telangectasia
5. Fabry disease
6.

43
11 A thirty-year-old woman presents with 1. colon cancer C
2 new progressively worsening 2. ovarian cancer
headaches. Upon exam, you notice 3. breast cancer
multiple acral keratotic papules and 4. basal cell carcinoma
papillomas of the tongue. What 5. melanoma
malignancy does this patient need to 6.
be screened for?
11 What nail change is seen in patients 1. Onycholysis C
3 with Mal de Meleda Syndrome? 2. Longitudinal ridging
3. Koilonychia
4. leukonychia
5. pterygium
6.
11 Which of the following is caused by a 1. White sponge nevus A
4 defect in keratins 4 & 13? 2. Epidermolysis bullosa simplex
3. Epidermolysis bullosa simplex
with myotonic dystrophy
4. Junctional EB with pyloric
atresia
5. Clouston’s syndrome
6.
11 The syndrome characterized by 1. KID syndrome A
5 generalized mild hyperkeratosis, 2. Vohwinkel syndrome
erythematous keratotic plaques, 3. Erythrokeratoderma variabilis
palmoplantar keratoderma, non- 4. CHILD syndrome
progressive sensorineural deafness, 5. Refsum syndrome
progressive bilateral keratitis with 6.
secondary blindness is:
11 Which of the following is caused by a 1. oculocutaneous albinism I D
6 defect in lysosomal transport: 2. oculocutaneous albinism II
3. Piebaldism
4. Chediak-higashi
5. Bloom�s syndrome
6.
11 The most common ocular association 1. Cataracts B
7 with cutis marmorata telangiectatica 2. Glaucoma
congenital is: 3. Retinoblastoma
4. Corneal opacity
5. Angioid streaks
6.
11 Painful crises and 'whorled' corneal 1. Homogentisic acid oxidase B
8 opacities are seen with which of the 2. Alpha-galactosidase A
following enzyme abnormalities? 3. Glucocerebrosidase
4. Iduronate sulfatase
5. Glucoronidase
6.
11 Which of the following medications is a 1. Propranolol B
9 teratogen associated with a aplasia 2. Methimazole
cutis congenita? 3. Lithium
4. Alcohol
5. Warfarin
6.
12 What cutaneous manifestation is 1. Verrucous hemangioms C
0 associated with familial cerebral 2. Glomeruloid hemangiomas
cavernomas? 3. hyperkeratotic cutaneous

44
capillary-venous
malformations(HCCVM)
4. segmental facial hemangiomas
5. Tufted angiomas
6.
12 Ivory-colored papules between the 1. Hurler D
1 angles of the scapulae are 2. Scheie
characteristic of which syndrome: 3. Morquio
4. Hunter
5. Sanfilippo
6.
12 What is the characteristic radiographic 1. Enchondromas D
2 finding in type I Gaucher disease? 2. Osteopoikilosis
3. Melorheostosis
4. Ehrlenmeyer flask deformity
5. Supernumerary vertebrae with
extra ribs
6.
12 A patient with Klinefelter Syndrome 1. Recurrent pulmonary infections B
3 may be expected to experience which 2. Recurrent leg ulcers
of the following: 3. Pulmonary valve stenosis
4. Gastroesophageal reflux
5. Scarring alopecia
6.
12 Dermatofibrosis lenticularis 1. LEMD3 A
4 disseminata and osteopoikilosis are 2. Fibrillin 2
findings seen with mutations of which 3. ABCC6
of the following genes? 4. Lysyl hydroxylase
5. Lysyl oxidase
6.
12 The NEMO gene is defective in Bloch- 1. Hypohidrotic ectodermal A
5 Sulzberger syndrome. What other dysplasia with immune deficiency
syndrome has been linked with defects 2. Hypomelanosis of Ito
in the NEMO gene? 3. Tuberous sclerosis
4. Waardenburg syndrome
5. Piebaldism
6.
12 Which of the following subtypes of 1. Type 1 (Gravis) B
6 Ehlers-Danlos Syndrome (EDS) is 2. Type 4 (Vascular)
associated with early demise? 3. Type 5 (X-linked)
4. Type 7 (Arthrochalasis multiplex
congenita)
5. Type 10 (Fibronectin)
6.
12 A patient presents with several light 1. Schopf-Schulz-Passarge A
7 blue cyst-like lesions on the eyelid. 2. Gardner syndrome
They consult their list of problems and 3. Hypohidrotic ectodermal
bring up plantar hyperkeratosis and dysplasia
dysplastic toenails. On oral exam, you 4. Cowden syndrome
note that they have both upper and 5. Cronkhite-Canada
lower dentures. The patient relates 6.
that after losing their "baby teeth",
only 3 teeth grew in their place. What
syndrome does this person most likely
have?

45
12 Lamellar ichthyosis is caused by 1. Steroid sulfatase E
8 mutations in which of the following 2. Fatty aldehyde oxidoreductase
genes? 3. Phytanoyl coenzyme A
hydroxylase deficiency
4. Arylsulfatase E
5. Transglutaminase
6.
12 Which of the following pairs of 1. Bloom syndrome and A
9 diseases are caused by mutations in Rothmund-Thompson syndrome
DNA helicases? 2. Xeroderma pigmentosum and
Cockayne syndrome
3. Muir-Torre syndrome and Birt-
Hogg-Dube syndrome
4. Dyskeratosis congenita and
Peutz-Jeghers syndrome
5. Cowden syndrome and
Bannayan-Riley-Ruvalcaba
syndrome
6.
13 What is the gene defect in this 1. Keratin 1 and 10 E
0 condition, which is also called Mendes 2. SPINK5
da Costa syndrome? 3. SLURP-1
4. Connexin 26
5. Connexin 31
6.
13 A patient with multiple deeply 1. Reassure the patient and follow E
1 pigmented papules has a skin biopsy up as needed
which reveals an epitheloid blue 2. Schedule prophylactic excision
nevus. The next appropriate step is: of the lesion
3. Begin a malignancy work-up
4. Refer to genetics
5. Order an echocardiogram
6.
13 What condition is associated with this 1. Chloracne E
2 finding of inflammatory keratotic facial 2. Systemic lupus erythematosus
papules which may result in scarring 3. Keratosis pilaris
and atrophy? 4. Reiter's syndrome
5. Ulerythema ophryogenes
6.
13 Which of the following is caused by a 1. Rubenstein-Taybi D
3 defect in cathepsin C? 2. Bullous Icthyosis of Siemens
3. Muir-Torre
4. Papillon-Lefevre
5. Nail-Patella syndrome
6.
13 Mutations affecting the VEGF receptor- 1. Hereditary lymphedema A
4 3 cause which of the following (Nonne-Milroy disease)
disorders? 2. Lymphedema-distichiasis
syndrome
3. Lymphedema and ptosis
4. Noonan syndrome
5. Hereditary hemorrhagic
telangiectasias
6.
13 A 5 month old girl presents with failure 1. Leiner syndrome C

46
5 to thrive. She has had life-long atopic 2. Omenn Syndrome
dermatitis treated with topical 3. Netherton Syndrome
hydrocortisone cream and has 4. Wiskott-Aldrich Syndrome
persisent hypernatremia. On your 5. Severe atopic dermatitis
exam, she has generalized erythema 6.
and scaling of her body and
trichorrhexis invaginata on
examination of hairs from her
eyebrows. Which syndrome is she
most likely to have?
13 Which of the following are cutaneous 1. Loose skin and crumpled ears E
6 features of Marfan syndrome? 2. Dermatofibrosis lenticularis and
striae
3. Fat herniation and cutaneous
atrophy
4. Sclerodermoid changes and
dyspigmentation
5. Striae and elastosis perforans
serpiginosa
6.
13 Non-bullous icthyosiform erythroderma 1. Transglutaminase-1 gene E
7 is caused by which of the following (TGM1)
mutations: 2. 12R-lipoxygenase gene
(ALOX12B)
3. Lipoxygenase-3 gene (ALOXE3)
4. Both 12R-lipoxygenase gene
(ALOX12B) and lipoxygenase-3
gene (ALOXE3) are correct
5. All of these answers are correct
6.
13 In patients with diffuse congenital 1. Liver A
8 hemangiomatosis, the most common 2. Thyroid
site for extracutaneous involvement is 3. Lungs
the : 4. Colon
5. Brain
6.
13 Which syndrome is characterized by 1. Turner Syndrome C
9 hyperhidrosis, lack of pain sensation, 2. Noonan Syndrome
hypersalivation, and absent fungiform 3. Riley-Day
papillae? 4. Rubinstein-Taybi syndrome
5. Cornelia de lange Syndrome
6.
14 Erythematous keratotic plaques of KID 1. Face A
0 Syndrome most commonly occur in 2. Neck
which location? 3. Chest
4. Back
5. Abdomen
6.
14 Which syndrome is characterized by 1. Klinefelter D
1 broad thumbs, a large beaked nose, 2. Proteus syndrome
and capillary malformation? 3. Bloom syndrome
4. Rubinstein-Taybi
5. Ehlers-Danlos syndrome
6.
14 A 50 year man presents with 1. Copper levels C

47
2 generalized metallic-grey 2. Lead levels
hyperpigmentation. His past medical 3. Iron levels
history includes diabetes, 4. Arsenic levels
hepatomegaly and arrythmias. 5. Cyanide levels
Laboratory tests should include: 6.
14 Patients with hemochromatosis are at 1. Vibrio vulnificus infections E
3 increased risk for which of the 2. Yersenia infections
following? 3. Polyarthritis
4. Generalized metallic-grey
hyperpigmentation
5. All of these options are correct
6.
14 A child presenting with the scalp 1. Adams-Oliver syndrome A
4 findings shown and a right arm 2. Bart's syndrome
hypoplasia would be diagnosed with 3. Progeria
which of the following syndromes? 4. Dunnigan syndrome
5. None of these options are
correct
6.
14 Familial macular and lichen 1. Sipple syndrome A
5 amyloidosis is a feature of which of the 2. Peutz-Jeghers syndrome
following conditions? 3. Marfan syndrome
4. Dyskeratosis congenita
5. Birt-Hogg-Dube syndrome
6.
14 "Mousy" odor of urine is characteristic 1. Phenylketonuria A
6 of which of the following syndromes? 2. Hunter disease
3. Alkaptonuria
4. Maple Syrup Urine disease
5. Hurler disease
6.
14 Which of the following laboratory test 1. Complete blood count with C
7 might prove useful in the diagnosis of differential
Fabry disease: 2. Fasting lipids
3. Urinary sediment exam with
polarizing light microscopy
4. Stool guaiac
5. Bleeding time
6.
14 What is the mode of transmission for 1. Autosomal dominant B
8 lamellar ichthyosis? 2. Autosomal recessive
3. X-linked dominant
4. X-linked recessive
5. Sporadic
6.
14 Which of the following is NOT a 1. Thrombocytopenia C
9 complication of Kasabach-Merritt 2. CHF
Syndrome: 3. Ataxia
4. Disseminated intravascular
coagulation
5. Gastrointestinal bleeding
6.
15 A seven month old infant diagnosed 1. Stool samples C
0 with eczema on her face returns for a 2. Fontanelle examination
diaper-rash follow-up. A one-month 3. Examination of palms and soles

48
trial of topical antifungals has failed to 4. Palpation of abdomen
improve the infant�s systems. The 5. Hearing test
part of the physical exam that might 6.
prove most useful include:
15 Epidermal nevus syndromes 1. Sporadic A
1 inheritance pattern is: 2. X-linked recessive
3. X-linked dominant
4. Autosomal recessive
5. Autosomal dominant
6.
15 Which disease is found more 1. Sarcoidosis E
2 commonly in mothers of patients with 2. Erythema nodosum
chronic granulomatous disease? 3. Churg-Straus disease
4. Wegener's disease
5. Discoid lupus erythematous
6.
15 Pseudoxanthoma elasticum is caused 1. Fibrillin 1 D
3 by mutations in which of the following 2. Elastin gene
genes? 3. Lysyl oxidase
4. ABCC6 gene
5. Collagen 5
6.
15 Findings of eyelid papules (string of 1. Lipoid proteinosis A
4 pearls) and a hoarse cry in infants is 2. Amyloidosis
characteristic of which of the following 3. Pseudoxanthoma elasticum
syndromes? 4. Disseminated xanthomas
5. None of these answers are
correct
6.
15 Mutations in calcium transporters 1. Erythrokeratodermia variabilis E
5 cause which pair of diseases? and progressive symmetric
erythrokeratodermia
2. Lamellar ichthyosis and
nonbullous congenital ichthyosiform
erythroderma
3. Refsum syndrome and Sjogren-
Larsson syndrome
4. Chondrodysplasia punctata and
CHILD syndrome
5. Darier�s disease and Hailey-
Hailey disease
6.
15 Which of the following is not classically 1. Multiple Endocrine Neoplasia D
6 associated with pheochromoctyoma? Type IIA
2. Multiple Endocrine Neoplasia
Type IIB
3. Von-Hippel-Lindau Syndrome
4. Cobb Syndrome
5. Neurofibromatosis
6.
15 Patients with x-linked icthyosis are 1. Pancreatic and acute D
7 more prone to getting which two lymphocytic leukemia (ALL)
malignancies? 2. Pancreatic and acute
myelogenous leukemia (AML)
3. Testicular and AML

49
4. Testicular and ALL
5. Renal cell and ALL
6.
15 Hyperextesible skin, gaping wounds, 1. Marfan syndrome C
8 cigarette-paper scars, molluscoid 2. Congenital contractural
pseudotumors, and calcified arachnodactyly
subcutaneous nodules are 3. Ehlers-Danlos Syndrome
characteristic of which syndrome? 4. Cutis laxa
5. Pseudoxanthoma elasticum
6.
15 Which finding would you expect in a 1. Increased risk for pancreatic B
9 child with dyskeratosis congenita? cancers
2. Oral leukoplakia
3. Hirsutism
4. Normal hematologic status
5. Normal nails
6.
16 What is this syndrome which is 1. Ehlers Danlos Syndrome B
0 histologically characterized by widely 2. Pseudoxanthoma Elasticum
dispersed granular material amidst 3. Buschke-Ollendorf Syndrome
normal fibers? 4. Focal Dermal Hypoplasia
5. Lipoid Proteinosis
6.
16 Beare-Stevenson cutis gyrata 1. Fibroblast growth factor A
1 syndrome is linked with mutations in: receptor 2
2. BSCL2
3. LMNA
4. ATP7A
5. None of these answers are
correct
6.
16 In a patient suspected of having 1. Calcitonin A
2 multiple endocrine neoplasia type IIb, 2. Glucagon
which lab test would be appropriate? 3. Parathyroid hormone
4. Calcium
5. Cortisol
6.
16 Pseudoxanthoma elasticum (PXE) can 1. ABCC6 A
3 be transmitted in an autosomal 2. MAN1
dominant, recessive or sporadic 3. Collagen III
manner. Which of the following genes 4. Collagen V
is mutated in PXE? 5. Collagen I
6.
16 A patient with this autosomal recessive 1. Acute leukemia A
4 disorder caused by a defect in helicase 2. Renal cell carcinoma
is an increased risk for which 3. Medullary thyroid carcinoma
malignancy? 4. Squamous cell carcinoma of the
lung
5. Prostate carcinoma
6.
16 What finding is seen on brain imaging 1. Tram track calcifications D
5 of patients with Papillon-Lefevre 2. Calcification of the falx cerebri
Syndrome? 3. Calcification of the
hippocampus
4. Calcification of the dura

50
5. Agenesis of the corpus
callosum
6.
16 Which of the following syndromes is 1. Bazex syndrome A
6 characterized by follicular 2. Rombo syndrome
atrophoderma, hypohidrosis, 3. Rasmusen syndrome
hypotrichosis and multiple basal cell 4. Gorlin syndrome
carcinomas? 5. Incontinentia Pigmenti
6.
16 An infant presents with multiple 1. Congestive Heart Failure A
7 congenital hemangomas in an 2. Obstructive jaundice
generalized distribution. What is the 3. Portal hypertension
most serious associated condition? 4. All of the answers are correct
5. None of the answers are correct
6.
16 A triangular-shaped lunula is a 1. Darier's disease E
8 characteristic finding in which disease? 2. Dyskeratosis congenita
3. Epidermal nevus syndrome
4. Incontinentia pigmenti
5. Nail-patella syndrome
6.
16 In ataxia telangectasia, the ATM gene 1. Responds to DNA damage by A
9 is mutated. The product of the ATM phosphorylating key DNA repair
gene is an enzyme which: substrates
2. Binds transforming growth
factor beta protein
3. Is the VEGF receptor 3
4. Participates in NF-kB activation
5. Is an inhibitor of G1 cyclin/Cdk
complexes
6.
17 A 16 year-old girl presents with a 1. APC A
0 family history of Gardner syndrome. 2. STK11
Her mother is very concerned that her 3. CYLD
daughter may have the syndrome as it 4. PTCH
runs in her family and she has many 5. ABCC6
skin complaints. Gardner syndrome 6.
has been linked to defects in beta-
catenin mediated transcription. Which
of the following genes dysfunction is
responsible?
17 Hypoplasia of the breast can be seen 1. Anhidrotic ectodermal dysplasia A
1 in which disease? 2. Maffucci syndrome
3. Congenital syphilis
4. Marfan syndrome
5. Osteogenesis imperfecta
6.
17 Which of the following syndromes are 1. Banayan-Zonana D
2 linked to a PTEN gene mutation? 2. Proteus
3. Cowden
4. All of these options are correct
5. Lhermitte-Duclos disease
6.
17 The most common neoplasm seen in 1. Enchondromas A
3 Maffucci Syndrome is: 2. Angiosarcomas

51
3. Osteosarcomas
4. Lymphangiosarcomas
5. chondrosarcoma
6.
17 What is the most common genetic 1. Neurofibromin A
4 defect associated with this syndrome? 2. Merlin
3. Tuberin
4. Hamartin
5. Folliculin
6.
17 A patient with thyroid carcinoma and 1. Trichoepitheliomas C
5 cobblestone-like changes of the oral 2. Fibrofolliculomas
mucosa will also likely have: 3. Tricholemmomas
4. Cylindromas
5. Syringomas
6.
17 What is the underlying gene defect for 1. SLURP-1 A
6 this transgrediens form of 2. TOC gene
palmoplanter keratoderma 3. Plakoglobin
4. Keratin type 1
5. Keratin type 9
6.
17 Which of the following is NOT part of 1. Peg or conical teeth A
7 the Carney complex? 2. Cardiac, cutaneous or
mammary myxomas
3. Pigmented skin lesions
4. Endocrine abnormalities
5. Primary pigmented nodular
adrenocortical disease
6.
17 Giant lysosomal granules are seen in 1. Chediak-Higashi syndrome A
8 which disease? 2. Griscelli syndrome
3. Piebaldism
4. Incontinentia pigmenti
5. Carney complex
6.
17 Osteopathia striata is seen in which of 1. Gaucher�s disease D
9 the following disorders? 2. Albright�s syndrome
3. Aplasia cutis congenita
4. Focal dermal hypoplasia
5. Scleroderma
6.
18 Medulloblastoma is seen in which 1. Gardner�s syndrome D
0 syndrome? 2. Multiple endocrine neoplasia 2b
3. Muir-Torre syndrome
4. Basal cell nevus syndrome
5. Neurofibromatosis Type 1
6.
18 The most common cardiovascular 1. Atrial septal defect D
1 defect in patients with Noonan 2. Ventricular septal defect
syndrome is: 3. Enlarged aorta
4. Pulmonic valve stenosis
5. Aortic stenosis
6.
18 Dermatofibrosis lenticularis 1. Ehlers-Danlos syndrome E

52
2 disseminata is seen in which of the 2. Marfan syndrome
following conditions? 3. Pseudoxanthoma elasticum
4. Focal dermal hypoplasia
5. Buschke-Ollendorf syndrome
6.
18 A 32 year-old woman is 5 weeks 1. Aplasia cutis congenita A
3 pregnant and is diagnosed with 2. Meningocele
hyperthyroidism. Her doctor gives her 3. Encephalocele
a prescription for on methimazole 10 4. Spina bifida
mg PO tid. Which of the following fetal 5. Dermoid cyst
abnormalities could be caused by this 6.
exposure?
18 A patient has recurrent infections and 1. Breast cancer C
4 is found to have a disorder of 2. Vitiligo
phagocytic cells from an inability of 3. Discoid lupus
phagocytes to undergo the respiratory 4. Thyroid disease
burst needed to kill certain types of 5. Cataracts
bacteria and fungi. Carriers of this 6.
disease are also at risk for which
condition?
18 Which of the following is correct about 1. known as Gardner syndrome D
5 nevoid basal cell carcinoma syndrome? 2. autosomal recessive
3. multiple keratoacanthomas
4. calicification of the falx
5. odontogenic steatomas
6.
18 You receive a hospital consult from the 1. TOC gene A
6 gastroenterology service for a 42-year 2. Desmoplakin
old woman with esophageal cancer. 3. Plakoglobin
They would like your opinion on the 4. Connexin 30.3
yellow, thickened areas on her palms 5. Connexin 31
and soles in areas of pressure. When 6.
you speak with her, she says that her
father had similar problems and it runs
in her family. Which of the following is
defective?
18 A 4-year old boy presents with 1. Congenital ichthyosiform A
7 generalized white scale. The mother erythroderma
reports that her son was born with a 2. Neutral lipid storage disease
tight membrane enveloping his body. 3. Lamellar ichthyosis
Peripheral blood smear is within 4. Netherton syndrome
normal limits. What is the most likely 5. Ichthyosis vulgaris
diagnosis? 6.
18 Patients with junctional epidermolysis 1. Laminin 5 E
8 bullosa have been found to have 2. Bullous pemphigoid antigen 2
mutations in: 3. Collagen 17
4. BP180
5. All of the answers are correct
6.
18 Keratins 1 and 10 are important 1. Netherton syndrome D
9 intermediate filaments in suprabasal 2. Sjogren-Larsson syndrome
keratinocytes. Genetic mutation of 3. Pachyonychia congenita type 1
these keratins can lead to which of the 4. Epidermolytic hyperkeratosis /
following disorders? Bullous ichthyosiform erythroderma
5. White sponge nevus

53
6.
19 Which syndrome is due to a defective 1. Netherton's syndrome D
0 secreted mammilian Ly6/uPAR-related 2. Refsum's syndrome
protein-1? 3. Sjogren-Larsson syndrome
4. Mal de Meleda syndrome
5. Haim-Munk syndrome
6.
19 The combination of painful 1. Naxos syndrome C
1 palmoplantar keratoderma and 2. Vohwinkel syndrome
pseudoherpetic keratitis is 3. Richner-Hanhart syndrome
characteristic of which of the following 4. Howel-Evans syndrome
syndromes? 5. Schopf-Schulz-Passarge
syndrome
6.
19 A deficiency in sialophorin, a surface 1. Wiskott-aldrich syndrome A
2 glycoprotein, is thought to play a role 2. Chediak-higashi syndrome
in which genetic disorder? 3. Werner syndrome
4. Bloom Syndrome
5. Cornelia de Lange syndrome
6.
19 Which of the following is defective in 1. Tenascin-X A
3 Ehlers-Danlos syndrome (EDS) with 2. Lysyl oxidase
congenital adrenal hyperplasia? 3. Lysyl hydroxylase
4. None of these answers are
correct
5. All of these answers are correct
6.
19 What protein is deficient in the 1. Calcium ATP�ase IIC1 A
4 condition shown? 2. Calcium ATP�ase IIA2
3. PEX-7
4. SPINK5
5. Desmoglein 3
6.
19 �Coast of Maine� caf� au lait 1. Hypomelanosis of Ito C
5 macules are characteristic of which 2. Carney complex
condition 3. McCune-Albright syndrome
4. Gaucher�s syndrome
5. Tuberous sclerosis
6.
19 Dyshcromatosis symmetrica 1. DKC C
6 hereditaria (DSH) is a disorder 2. PTEN
characterized by asymptomatic hypo- 3. ADAR1
and hyperpigmented macules on the 4. ERCC4
face and dorsal extremities. Mutations 5. STK11
in which of the following genes is 6.
associated with this condition?
19 Bilateral ovarian cystadenomas and 1. Cowden syndrome B
7 parotid monomorphic adenomas are 2. Proteus syndrome
seen in which of the following 3. Von-Hippel-Lindau syndrome
conditions? 4. Noonan syndrome
5. Beckwith-Wiederman syndrome
6.
19 Which of the following is NOT a 1. Syringomas E
8 characteristic skin finding in patients 2. Elastosis perforans serpiginosa
with Down Syndrome? 3. Single palmar crease

54
4. Flat nipples
5. Small tongue
6.
19 This form of EB simplex has a defect in 1. Dowling Meara D
9 actin assembly, not keratin formation. 2. Weber-Cockayne
3. Koebner
4. Kindler
5. Ogna
6.
20 Mutations in nuclear lamins are 1. Lipomatosis E
0 associated with which clinical feature? 2. Sarcomas
3. Lymphedema
4. Aplasia cutis congenita
5. Lipodystrophy
6.
20 Urticaria pigmentosa is linked to a 1. Piebaldism A
1 defect in the c-kit protooncogene. 2. Hypomelanosis of Ito
What autosomal dominant skin disease 3. Waardenburg syndrome
also has been linked to this defect? 4. Hermansky-Pudlak syndrome
5. Incontinentia pigmenti
6.
20 A 4 year-old boy presents with 2 soft, 1. Stool guiac A
2 dark-blue, compressible nodules on his 2. MRI of the abdomen
extremities. His mother has noted that 3. CBC
these lesions have increased sweating 4. Immediate referral to a
and that they were present at birth. No gastroenterologist
one else in the family has had similar 5. Biopsy of a skin lesion
skin lesions. What step is indicated 6.
first to help determine the diagnosis?
20 A BSCL2 gene mutation with the 1. Berardinelli-Seip congenital A
3 cutaneous findings of generalized lipodystrophy
lipodystrophy, hyperlipemia, 2. Familial partial lipodystrophy
hepatomegaly, acanthosis nigricans, 3. Bjornstad syndrome
elevated basal metabolic rate and non- 4. All of the answers are correct
ketotic insulin resistant diabetes 5. None of the answers are correct
mellitus are characteristic of which of 6.
the following syndromes?
20 The treatment for acrodermatitis 1. Zinc supplementation A
4 enteropathica is: 2. Iron supplementation
3. Vitamin B1 supplementation
4. Vitamin B12 supplementation
5. Phlebotomy
6.
20 Ectopia lentis (downward displacement 1. Marfan syndrome B
5 of the lens) is characteristic of: 2. Homocystinuria
3. Phenylketonuria
4. Multiple Carboxylase deficiency
5. Ehlers-Danlos syndrome
6.
20 A Puerto Rican woman is seen in clinic 1. Her platelets lack dense bodies, A
6 for a pruritic rash on her trunk. A causing excess bleeding
punch biopsy is performed. The biopsy 2. Her intrinsic factor is deficient
site continues to bleed, with 3. Her Factor VIII levels are low
hematoma formation. The bleeding is 4. She is congentially deficient in
eventually controlled. On further platelets

55
exam, her skin and hair are light 5. None of the answers are correct
brown. She has a history of 6.
granulomatous colitis. What it the
most likely reason she had excess
bleeding with a simple procedure?
20 Which of the following syndromes 1. Tuzun Syndrome E
7 demonstrate atrophoderma 2. ROMBO Syndrome
vermiculatum? 3. Nicolau-Balus Syndrome
4. Braun-Falco-Marghescu
Syndrome
5. All of these answers are correct
6.
20 The arylsulfatase C gene is mutated in 1. X-linked ichthyosis A
8 which disease? 2. Refsum syndrome
3. Haim-Munk syndrome
4. Naxos syndrome
5. Griscelli syndrome
6.
20 Mucosal malignancy is a complication 1. Oral hairy leukoplakia C
9 of: 2. White sponge nevus
3. Dyskeratosis congenital
4. Chronic candidiasis
5. Focal epithelial hyperplasia
6.
21 Focal Dermal Hypoplasia (Goltz 1. Type of inheritance C
0 Syndrome) can differentiated from 2. Presence of Linear lesions along
Incontinentia Pigmenti by the lines of Blaschko
3. Presence of blistering lesions
4. Hair and teeth abnormality
5. Eye and CNS abnormality
6.
21 Which gene is defective in Wiskott- 1. WAS A
1 Aldrich syndrome? 2. CYBA
3. CYBB
4. NCF1
5. NCF2
6.
21 Which vascular disorder is 1. Sturge-Weber syndome D
2 characterized by facial vascular 2. PHACES
malformation and ipsilateral 3. Encephalotrigeminal
intracranial and retinal arteriovenous angiomatosis
malformations(AVMs)? 4. Bonnet Dechaune Blanc
syndrome
5. Von Lohuizen�s disease
6.
21 What condition is associated with a 1. Ataxia telangiectasia E
3 mutation in CXCR4? 2. Leukocyte adhesion deficiency
3. Chronic granulomatous disease
4. SCID
5. WHIM syndrome
6.
21 Which of the following is caused by a 1. Fabry disease C
4 mutation in a gene which codes for 2. Lesch-nyhan disease
steroid sulfatase? 3. X-linked ichthyosis
4. Lamellar icthyosis

56
5. Chediak-Higashi
6.
21 Which opthamologic disease is 1. Glaucoma A
5 associated with this disorder? 2. Ectopia lentis
3. Cataracts
4. Posterior subcasular lentiular
opacity
5. Retinitis pigmentosa
6.
21 In which of the following 1. Marfan's Disease D
6 Genodermatoses would one find 2. Hemochromatosis
cutaneous hyperpigmentation, blue 3. Gaucher's Disease
lunulae and Kayser-Fleishcher rings: 4. Wilson's Disease
5. Osteogenesis Imperfecta
6.
21 Which of the following syndromes is 1. Wiskott-Aldrich syndrome C
7 associated with markedly increased 2. Chronic granulomatous disease
IgE levels, cold abscesses and a 3. Job syndrome
characteristic coarse facies? 4. Severe combined
immunodeficiency
5. Leiner's disease
6.
21 Ichthyosis linearis circumflexa is one of 1. SPINK5 A
8 the findings seen in the syndrome 2. SLURP1
caused by which of the following 3. GJB2
genes? 4. LMX1B
5. CYLD
6.
21 Ichthyosis hystrix is characterized by 1. Keratins 1 and 9 B
9 the following gene defects? 2. Keratins 1 and 10
3. Keratins 5 and 14
4. Keratins 6 and 16
5. None of these answers are
correct
6.
22 Which of the following syndromes is 1. Netherton�s E
0 associated with cutis marmorata? 2. Papillon-Lefevre syndrome
3. Hemansky-Pudlak syndrome
4. Maffucci syndrome
5. Cornelia de Lange syndrome
6.
22 Spastic ditetraplegia is associated with 1. Sjogren-Larsson syndrome A
1 which of the following disorders? 2. X-linked ichthyosis
3. Lamellar ichthyosis
4. KID syndrome
5. Refsum syndrome
6.
22 Naxos syndrome is characterized by a 1. Desmoglein 1 C
2 right sided cardiomyopathy, wooly 2. Desmoglein 3
hair, and keratoderma. The epidermal 3. Plakoglobin
structure defective in Naxos syndrome 4. Desmoplakin
is: 5. Keratin 1/10
6.
22 A 11 year-old female patient with 1. Ectodermal dysplasia D
3 hypoparathyroidism is referred to your 2. Insulin dependent diabetes

57
clinic secondary to chronic mellitus
mucocutanous candidiasis which is 3. Chronic autoimmune hepatitis
refractory to standard treatments. The 4. Addison's disease
patient also has malabsorption and 5. Hypothyroidism
severe chronic diarrhea. You determine 6.
that she has autoimmune
polyendocrinopathy-candiasis-
ectodermal dystrophy
syndrome(APECED). Since only two of
three major criteria are needed to
make this diagnosis, this patient
having demonstrated
hypoparathyroidism and chronic
mucocutanous candidiasis is
diagnosed with APECED. What is the
third major criteria that would qualify a
patient to meet the diagnosis of
APECED?
22 What is the most likely nail findings in 1. Koilonychia B
4 a patient who has this autosomal 2. Red and white longitudinal
dominant disease with these keratotic bands
papules and cobblestoning of the oral 3. Melanonychia
mucosa? 4. Half and half nails
5. Pincer nails
6.
22 The gene defect in Griscelli Syndrome 1. Myosin Va or Rab27a A
5 is: 2. LYST or CHS1
3. P gene
4. TRP1
5. None of these answers are
correct
6.
22 A patient with a white, spongy 1. Keratin 1/10 C
6 overgrowth of the buccal mucosa that 2. Keratin 6b/17
has passed in an autosomal dominant 3. Keratin 4/13
fashion is most likely related to a 4. Keratin 6a/16
mutation in which of the following? 5. None of these options are
correct
6.
22 Sphenoid wing dysplasia is seen in: 1. Tuberous sclerosis D
7 2. Tay Syndrome
3. Mafucci syndrome
4. NF-1
5. NF-2
6.
22 Adenosine deaminase deficiency is 1. Wiskott Aldrich syndrome E
8 associated with which of the following 2. Gout
disorders? 3. Job syndrome
4. Xeroderma pigmentosum
5. Severe combined
immunodeficiency
6.
22 A child presents with sparse, short hair 1. Bjornstad syndrome A
9 and sensorineural deafness. On 2. Menkes kinky hair syndrome
microscopic examination of the hair, 3. Argininosuccinic aciduria

58
pili torti is noted. Which of the 4. Trichothiodystrophy
following syndromes is the most likely 5. None of the options are correct
diagnosis? 6.
23 Eyelid string of pearls are seen in 1. Focal dermal hypoplasia B
0 which of the following conditions? 2. Lipoid proteinosis
3. Hutchinson-Gilford syndrome
4. Beare-Stevenson cutis gyrata
syndrome
5. Gaucher�s disease
6.
23 Beckwith-Wiederman syndrome is 1. Hemangioblastomas, renal E
1 characterized by which of the following cysts and renal cell carcinoma
triads? 2. Epistaxis, telangictases, and
gastrointestinal tract bleeding
3. Enlarged limb, port wine stain,
and deep venous thrombosis
4. Omphalocele, venous
malformations, and ataxia
5. Exomphalos, macroglossia, and
gigantism
6.
23 A child has ichthyosis and is found to 1. Chanarin-Dorfman syndrome A
2 have leukocytes with vacuoles filled 2. Ichthyosis hystrix
with lipids on peripheral smear. She 3. Naxos disease
most likely has: 4. Ichthyosis bullosa of Siemens
5. Ichthyosis vulgaris
6.
23 Angioid streaks on retinal exam are 1. Pseudoxanthoma elasticum A
3 characteristic of which of the following 2. Choroid malformations
syndromes? 3. Eyelid papillomas
4. Lester iris
5. Salt & pepper retinitis
pigmentosa
6.
23 Which of the following syndromes is 1. Birt-Hogg-Dube C
4 associated with tricholemmomas? 2. Brooke-Spiegler
3. Bannayan-Riley-Ruvacalba
4. Basex
5. Rasmussen's
6.
23 A child presents with the hair finding 1. Keratin 1/10 B
5 seen in the image in addition to brittle 2. Keratin hHb1/hHb6
nails, keratosis pilaris, abnormal teeth 3. Keratin 6/16
and cataracts. Which of the following 4. Keratin 6/17
abnormalities is the most likely 5. Keratin 2e
mutated? 6.
23 The gene defect in LEOPARD syndrome 1. PTPN11 A
6 is: 2. PRKAR1A
3. KIP2
4. ATM
5. Neurofibromin
6.
23 Regarding the inheritance of Ehlers- 1. The type with the lysyl oxidase A
7 Danlos syndrome, which subtype is deficiency
inherited in an X-linked recessive 2. The type with the lysyl

59
manner? hydroxylase deficiency
3. The type with a collagen 5
defect
4. The type with a collagen 3
defect
5. The type with a defect in
procollagen aminopeptidase
6.
23 Patients with homocystinuria are 1. Alpha galactosidase A D
8 deficient in: 2. Transglutaminase
3. Lysyl oxidase
4. Cystathionine synthetase
5. Steroid sulfatase
6.
23 A patient presents with bilateral retinal 1. VHL A
9 hemangioblastomas and a capillary 2. Endoglin
malformation on her neck. Which gene 3. PTH/PTHrP type I receptor
mutation is most likely? 4. VEGF receptor-3
5. MFH1
6.
24 A 18 yo man presents for evaluation of 1. Head CT D
0 foot lesions. There are thick 2. Thoracic CT
hyperkeratotic plaques symmetrically 3. Hepatic ultrasound
on only the weight bearing plantar 4. Endoscopy
surfaces. What test(s) should this 5. Knee films
patient be referred for? 6.
24 Which disease can clinically mimic 1. Wilsons C
1 pellagra but is inherited in an 2. Hemochromatosis
autosomal recessive fashion and is due 3. Hartnup Disease
to a defect in the transport of neutral 4. Fabry
amino acids? 5. Gaucher�s
6.
24 Meleda is an island off the coast of 1. SLURP1 gene A
2 Croatia. Its inhabitants have an 2. Keratin 1/9
increased frequency of a malodorous 3. TOC gene
transgradiens palmoplantar 4. Loricrin
keratoderma in a stocking-glove 5. Cathepsin C
distribution. This condition is 6.
autosomal recessive with a defect in:
24 Comma-shaped corneal opacities are 1. Ichthyosis vulgaris B
3 characteristic of which type of 2. X-linked ichthyosis
ichthyosis? 3. Lamellar ichthyosis
4. Nonbullous congenital
ichthyosiform erythroderma
5. Refsum syndrome
6.
24 Acropigmentation of Dohi is 1. Reticulated pigmentation of the C
4 characterized by: axillae, neck, and groin
2. Linear palmar pits and
pigmented macules on volar and
dorsal hands and feet
3. Pigmented and depigmented
macules on the distal dorsal
extremities and face
4. Hyperpigmented macules on

60
the lips and oral mucosa
5. Flaccid, superficial pustules
that burst and leave pigmented
macules
6.
24 Patients with Hermansky-Pudlak 1. Arteriovenous malformations B
5 syndrome may experience which of 2. Pulmomary fibrosis
the following systemic complications 3. Gastroesophageal reflux
as a result of their disease? disease
4. Aortic stenosis
5. Rectal abscesses
6.
24 A patient with multiple lentigines and 1. Deafness C
6 blue nevi may also have: 2. Pulmonary valve stenosis
3. Atrial myxoma
4. Mental retardation
5. GI malignancy
6.
24 Germline KILLIN methylation is 1. Bannayan-Riley-Ruvalcaba C
7 associated with which of the following Syndrome
syndromes? 2. Proteus Syndrome
3. Cowden Syndrome
4. Birt-Hogg-Dube Syndrome
5. Galli-Galli Disease
6.
24 Primary pigmented nodular 1. Hypomelanosis of Ito B
8 adrenocortical disease and 2. Carney complex
psammomatous melanotic 3. McCune-Albright syndrome
schwannomas are characteristic of 4. Gaucher�s syndrome
which of the following syndromes? 5. Tuberous sclerosis
6.
24 Deficiency of filaggrin is the strongest 1. Autosomal dominant B
9 known predisposing genetic factor for 2. Autosomal semidominant
the development of atopic dermatitis. 3. Autosomal recessive
What is the mode of inheritance of 4. X-linked dominant
mutations in filaggrin? 5. X-linked recessive
6.
25 Nevoid basal carcinoma syndrome 1. Polyostotic fibrous dysplasia C
0 (Gorlin syndrome) is autosomal 2. Stippled epiphyses
dominant transmitted mutation of the 3. Calcification of falx cerebri
patched gene. Symptoms include 4. Osteopoikilosis
innumberable basal cell carcinomas, 5. Sphenoid wing dysplasia
painful odontogenic jaw keratocysts, 6.
palmoplantar pits, frontal bossing, bifid
ribs and what other bony abnormality?
25 A patient Buschke-Ollendorff syndrome 1. Waxy papules along the eyelids D
1 has osteopoikilosis and which 2. Caf� au lait macules
cutaneous finding? 3. Port wine stain
4. Juvenile elastoma
5. Epidermal nevi
6.
25 The diagnostic test for chronic 1. Dimethylglyoxime test B
2 granulomatous disease is: 2. Nitroblue tetrazolium reduction
assay
3. Histamine skin test

61
4. Bone marrow biopsy
5. Potassium hydroxide
6.
25 Epidermolysis bullosa simplex is 1. Granular layer keratinocyte C
3 caused by blistering in which 2. Spinous layer keratinocyte
structure? 3. Basal layer keratinocyte
4. Lamina densa
5. Sublamina densa
6.
25 A child with phenylketonuria likely 1. Blue-gray generalized C
4 presents with which cutaneous hyperpigmentation
problems? 2. Alopecia universalis
3. Generalized hypopigmentation
4. Generalized hyperpigmentation
5. Leg ulcers
6.
25 Which of the following findings is 1. Lipoatrophic sclerodermoid skin E
5 characteristic of a mutation in lamin A? 2. Alopecia
3. Craniomegaly with small face
4. Severe premature
atherosclerosis with early death
5. All of the answers are correct
6.
25 A patient with port wine stain on a 1. Arteriovenous fistulas A
6 lower extremity, hemihypertrophy of 2. Multiple cafe-au-lait macules
the limb and lymphatic and deep 3. Macroglossia
venouse insufficiency of the affected 4. Cutis marmorata
limb would be considered to have 5. Distichiasis
Klippel-Trenaunay-Weber syndrome. 6.
What additional feature would need to
be present to define the patient as
having Parkes-Weber syndrome?
25 What is the genetic defect of this 1. PTEN D
7 autosomal dominant disorder? 2. PTPN11
3. Calcium ATPase 2A2
4. Calcium ATPase 2C1
5. SPINK5
6.
25 A patient with multiple facial 1. Basal cell carcinoma B
8 trichilemmomas is at risk of which of 2. Breast carcinoma
the following cancers? 3. Acute leukemia
4. Cylindroma
5. Oral squamous cell carcinoma
6.
25 Birt-Hogg-Dube syndrome is most 1. Basal cell carcinoma C
9 strongly associated with which of the 2. Medulloblastoma
following malignancies? 3. Renal cell carcinoma
4. Trichoepithelial carcinoma
5. Eccrine syringofibroadenoma
6.
26 Adenosine deaminase deficiency is 1. Wiskott-Aldrich syndrome D
0 seen in which immunodeficient 2. Chronic granulomatous disease
disease? 3. Job syndrome
4. Severe combined
immunodeficiency syndrome

62
5. Leineri's disease
6.
26 Tyrosinase positive albinism 1. Tyrosinase B
1 (oculocutaneous albinism type 2) is 2. P gene
caused by a mutation in which of the 3. Tyrosinase related protein 1
following: 4. C-kit
5. NEMO
6.
26 Findings of milia, cylindromas and the 1. Gorlin's syndrome C
2 condition shown in the pathology 2. Familial cylindromatosis
image are characteristic of which of 3. Brook-Spiegler syndrome
the following syndromes? 4. Rasmusen syndrome
5. Rombo syndrome
6.
26 Enchondromas and chondrosarcomas 1. Proteus syndrome C
3 are most strongly associated with 2. Osler-Weber-Rendu syndrome
which of the following syndromes? 3. Maffucci syndrome
4. Nonne-Milroy disease
5. Blue rubber bleb nevus
syndrome
6.
26 Which eye findings would be expected 1. Dendritic corneal ulcerations D
4 in an individual with this disorder 2. Salt and pepper retinitis
associated with atherosclerosis? pigmentosa
3. Ectopia lentis
4. Angoid streaks
5. Keratoconus
6.
26 The coast of Maine cafe au lait macule 1. Tuberous Sclerosis C
5 is a common finding in which of the 2. Neurofibromatosis type I
following syndromes? 3. McCune-Albright syndrome
4. Watson syndrome
5. Russell-Silver syndrome
6.
26 Ichthyosiform erythroderma in lines of 1. Refsum syndrome B
6 Blaschko, follicular atrophoderma, and 2. Chondrodysplasia punctata
stippled epiphyses are characteristic of 3. Erythrokeratodermia variabilis
which of the following disorders? 4. Netherton syndrome
5. Sjogren-Larsson syndrome
6.
26 Which of the following metals is 1. Copper A
7 deficient in the serum of patients with 2. Iron
Menkes kinky hair syndrome? 3. Selenium
4. Zinc
5. Biotin
6.
26 A patient with myotonic dystrophy and 1. Beta-catenin A
8 multiple skin lesions most likely has 2. Desmoplakin
activating mutations in which of the 3. Plakoglobin
following? 4. Desmoglein
5. Alpha 6-beta 4 integrin
6.
26 Which of the following bony defect is 1. Polyostotic fibrous dysplasia B
9 found in CHILD syndrome? 2. Stippled epiphyses
3. Calcification of falx cerebri

63
4. Osteopoikilosis
5. Sphenoid wing dysplasia
6.
27 What is the most likely gene mutation 1. Connexin 26 B
0 in this individual who has migratory 2. Connexin 30.3 and 31
patches and fixed plaques as depicted 3. Calcium ATPase 2C1
in this picture? 4. Loricrin
5. Keratins 1 and 10
6.
27 A 12 year-old boy with pits on his 1. Arsenic exposure D
1 palms and lateral fingers may have: 2. A hereditary keratoderma
3. A corynebacteria infection
4. An inherited cancer syndrome
5. Secondary syphilis
6.
27 Which of the following syndromes is X- 1. Anhidrotic ectodermal dysplasia D
2 linked dominant? 2. Dyskeratosis congenita
3. X-linked icthyosis
4. Orofaciodigital syndrome 1
5. Menkes kinky hair syndrome
6.
27 Connexin 30 (GJB6 gene) is defective 1. KID syndrome D
3 in which of the following syndromes? 2. Vohwinkel syndrome
3. Vohwinkel syndrome variant
4. Clouston syndrome
5. Erythrokeratoderma variabilis
6.
27 Patients with Russell-Silver syndrome 1. Clinodactyly of fifth finger A
4 exhibit: 2. Peg teeth
3. Osteopathia striata
4. Broad thumbs
5. Shortened 4th and 5th
metacarpals
6.
27 Which of the following diseases is seen 1. Griscelli syndrome D
5 only in females? 2. Piebaldism
3. Hypomelanosis of Ito
4. Incontinentia pigmenti
5. Carney complex
6.
27 The characteristic dental findings in 1. Peg teeth C
6 patients with tuberous sclerosis are: 2. Anodontia
3. Enamel pits
4. Odontogenic cysts
5. Retention of primary teeth
6.
27 You are consulted on a patient with 1. Scalp B
7 possible Nethertons Syndrome. Which 2. Eyebrow
location of the body would most likely 3. Eyelash
have hairs demonstrating trichorrhexis 4. All of these answers are correct
invaginata? 5. None of these answers are
correct
6.
27 To help diagnose trichothiodystrophy, 1. Arginine E
8 which of the following levels are 2. Histidine

64
decreased in hairs of affected 3. Phenylalanine
individuals? 4. Glycine
5. Cysteine
6.
27 Which one of the following is the most 1. HPV-5 A
9 common oncogenic virus in patients 2. HPV-8
with epidermodysplasia verruciformis? 3. HPV-13
4. HPV-16
5. HPV-33
6.
28 A patient with hypohydrosis and 1. Dyskeratosis congenita C
0 hyperpyrexia, anodontia, and sparse 2. Pachyonychia congenita
hair has which syndrome: 3. Anhidrotic ectodermal dysplasia
4. Hidrotic ectodermal dysplasia
5. Papillon Lefevre
6.
28 You are examining a child with mild 1. Giant lysosomal granules are A
1 albinism, immunodeficiency and silver present in neutrophils in the blood
grey highlights in his hair. You smear
diagnose the child with Chediak- 2. Griscelli syndrome does not
Higashi syndrome. Why are you have albinism as a feature
confident that this isn�t Griscelli 3. Griscelli syndrome has no
syndrome? changes in hair color
4. All of these answers are correct
5. None of these answers are
correct
6.
28 Menke�s kinky hair syndrome is 1. Mitochondrial gene D
2 caused by a defect in: 2. DNA helicase
3. Gap junction protein
4. Copper Transporting ATPase
5. Proto-oncogene
6.
28 All of the following disorders are 1. Bloom syndrome C
3 exacerbated by UV radiation except: 2. Hartnup�s disease
3. Refsum syndrome
4. Cockayne syndrome
5. Rothmund-Thomopson
syndrome
6.
28 A patient with Bloom Syndrome is 1. Decreased immunoglobulins A
4 most likely to have which laboratory 2. Macrocytic anemia
abnormalities: 3. Elevated IgE
4. Thrombocytopenia
5. Positive ANA
6.
28 Which of the following eye findings is 1. Angioid streaks A
5 caused by the rupture of Bruch's 2. Blue sclerae
membrane? 3. Retinal detachment
4. Ruptured globe
5. Keratoconus
6.
28 Pili trianguli et canaliculi is 1. Uncombable hair syndrome A
6 characteristic of which of the following 2. Bjornstad syndrome
syndromes? 3. Menkes kinky hair syndrome

65
4. Leiners disease
5. Netherton's syndrome
6.
28 Patients with progeria typically die of 1. Infection C
7 which of the following conditions? 2. Metastatic carcinoma
3. Atherosclerotic heart disease
4. Nail atrophy
5. Progressive systemic sclerosis
6.
28 What is the genetic defect of this 1. ABCA12 gene A
8 syndrome? 2. Transglutaminase 1 (TGM1).
3. GJB2 (connexin 26) gene.
4. Steroid sulfatase (STS) gene
5. Profilaggrin gene
6.
28 Palmoplantar keratoderma with 1. SLURP-1 C
9 deafness is caused by a defect in 2. Plakophilin
which gene? 3. Mitochondrial serine
transferase RNA
4. Lysosomal papain like cysteine
proteinase
5. Unknown
6.
29 In biopsies from blisters in patients 1. Basal cell layer of the epidermis B
0 with junctional epidermolysis bullosa, 2. Lamina lucida
the split is found in the: 3. Lamina densa
4. Squamous cell layer of the
epidermis
5. None of the answers are correct
6.
29 Which of the following is the first 1. Conjunctival telangictases D
1 symptom of ataxia telangiectasias? 2. Facial telangiectases
3. Hematologic malgignancy
4. Cerebellar ataxia
5. Breast cancer
6.
29 Which of the following is not a feature 1. Dwarfism D
2 of Cockayne syndrome? 2. Salt and pepper retina
3. Increased sister chromatid
exchange
4. Increased risk of skin cancer
5. Poikiloderma
6.
29 A patient with coarse facies, broad 1. Hypertriglyceridemia D
3 nasal bridge, and extensive eczema 2. High Copper levels
might be expected to have which 3. Low Zinc levels
abnormal laboratory value? 4. High IgE levels
5. Anemia
6.
29 Patients with Chondrodysplasia 1. CHILD syndrome A
4 punctata can have findings of stippled 2. Incontinentia Pigmenti
epiphyses on X-ray examination. 3. Focal Dermal Hypoplasia
Which other x-linked dominant 4. Goltz syndrome
condition can have stippled epiphyses? 5. Bazex syndrome
6.

66
29 Which of the following features is not 1. Normal intelligence A
5 associated with Cornelia de Lange 2. Characteristic facies with
Syndrome? downturned mouth, hirsutism,
synophrys, trichomegaly,
anteverted nostrils, long philtrum
and low set ears
3. Cryptorchidism
4. Fifth finger clinodactyly
5. Recurrent lung infections
6.
29 Cutaneous meningiomas have been 1. Neurofibromatisis A
6 associated with what syndrome? 2. Cowden
3. Neurocutaneous melanosis
4. Gorlin
5. Glomangiomatosis
6.
29 What is the most likely syndrome that 1. A. Klippel Trenaunay Weber A
7 this woman has in this photograph? 2. Filariasis
3. Neurofibromatosis
4. Proteus syndrome
5. Turner syndrome
6.
29 A patient presents with starfish 1. Connexin 31 D
8 keratoses, pseudoainhum, 2. Connexin 26
honeycombed PPK, and generalized 3. Connexin 30
ichthyosis. What is the most likely 4. Loricrin
genetic defect? 5. Connexin 33
6.
29 A 17 y/o man presents with facial acne 1. Testicular A
9 that he would like treated. You notice 2. Abdominal
that he has fine brown scale on his 3. Lymph node
neck and do a complete skin exam. 4. Lung
This scale is present on the remainder 5. Rectal
of his body, sparing his palms, soles 6.
and flexural areas. He informs you that
his uncles on his mother�s side have
similar skin findings. He is not
concerned about the skin and would
like to proceed with acne treatment
only. What other clinical exam should
you perform to screen for a potential
malignancy to which this patient is at
higher risk for acquiring?
30 Which of the following conditions is 1. Epidermolysis bullosa simplex D
0 inherited in an X-linked recessive 2. Ichthyosis vulgaris
manner? 3. Sjogren-Larsson syndrome
4. Wiskott-Aldrich Syndrome
5. Netherton�s Syndrome
6.
30 Which of the following is a potentially 1. Development of C
1 serious complication of the blue rubber chondrosarcomas
bleb nevus syndrome? 2. Development of angiosarcomas
3. Gastrointestinal hemorrhage
4. Development of lymphedema
5. Development of fluid retention

67
6.
30 Ichthyosis bullosa of Siemens is a 1. Keratin 2e A
2 condition characterized by fragile 2. Keratin 5/14
blisters at birth, hyperkeratotic 3. Keratin 6a/16
plaques on elbows/knees later in life, 4. Keratin 1/10
and a gene mutation in: 5. None of these options are
correct
6.
30 Cronkhite-Canada is associated with 1. Autosomal recessive E
3 gastrointestinal polyposis, nail atrophy, 2. Autosomal dominant
alopecia, generalized pigmentation of 3. X-linked recessive
skin and melanotic macules on the 4. X-linked dominant
fingers. Which of the following 5. Sporadic
describes its inheritance pattern? 6.
30 A child presents with macroglossia, 1. KIP2 A
4 exopthalmos and gigantism. He has a 2. ATM
history of omphalocele repair and has 3. VHL
circular depressions on the rim of the 4. FLT4
posterior helices. Although this 5. FOXC2
syndrome is most often transmitted in 6.
a sporadic manner, 15% of cases have
defects in which gene?
30 What is the gene defect in harlequin 1. Transglutaminase C
5 fetus? 2. Steroid sulfatse
3. ABCA12
4. ABCC6
5. None of these answers are
correct
6.
30 Retention of primary teeth a dental 1. Hypomelanosis of Ito E
6 finding of which of the following 2. Letterer-Siwe disease
conditions? 3. Tuberous sclerosis
4. Jackson Sertoli syndrome
5. Hyper-IgE syndrome
6.
30 The hair abnormality shown in the 1. Naxos disease B
7 image is characteristic of which of the 2. Trichothiodystrophy
following diseases? 3. Bjornstad syndrome
4. Monilethrix
5. All of the options are correct
6.
30 Yellow-brown depositions in 1. Hemochromatosis C
8 Descemet's membrane of the corneas 2. Neurofibromatosis
is diagnostic for: 3. Hepatolenticular degeneration
4. Pseudoxanthoma elasticum
5. Diabetes mellitus
6.
30 Medulloblastomas are seen in which of 1. Bazex syndrome E
9 the following syndromes? 2. Familial cylindromatosis
3. Nicolau-Balus syndrome
4. Birt-Hogg-Dube syndrome
5. Nevoid basal cell carcinomas
syndrome
6.

68
4. General Dermatology
# Question Answers Ans
1 The most common finding in patients with 1. Migratory polyarthritis B
systemic sclerosis is: 2. Esophageal dysfunction
3. Pulmonary fibrosis
4. Renal disease
5. Cardiac conduction defects
6. Show Explaination
2 The causative organism for mucocutaneous 1. Leishmania aethiopica C
leishmaniasis is: 2. Leishmania infantum
3. Leishmania brasiliensis
4. Leishmania tropica
5. Leishmania major
6. Show Explaination
3 The first site in body that shows yellowish 1. Face A
pigmentation in carotenoderma is 2. Trunk
3. Palms and soles

69
4. Nails
5. Sclera
6. Show Explaination
4 Increased chylomicrons are a feature of which 1. Type I A
type of hyperlipoproteinemia? 2. Type IIa
3. Type IIb
4. Type III
5. Type IV
6. Show Explaination
5 Which antibody is 93% specific for Sjogren's 1. anti-Ro C
syndrome? 2. anti-La
3. anti-fodrin
4. anti-Schirmer
5. RF
6. Show Explaination
6 Solid confluent palmoplantar keratosis, salmon- 1. Lichen Sclerosis E
orange follicular papules and diffuse 2. Dermatomyosistis
symmetrical involvement with characteristic 3. Phrynoderma
small islands of normal skin within affected 4. Reiter’s syndrome
areas are all clinical characteristics of what skin 5. Pityriasis rubra pilaris
disease? 6. Show Explaination
7 This woman has restricted ability to open her 1. Topoisomerase I antibodies (formerly Scl-70) A
mouth with tight bound down skin of her hands 2. Fibrillarin antibodies
and pulmonary fibrosis. In addition to a positive 3. Anti-U3RNP antibodies
ANA with a nucleolar pattern, what is the most 4. Anti-PM-Scl antibodies
common autoimmune antibody that is 5. Anti-thyroglobulin antibodies
associated with pulmonary fibrosis? 6. Show Explaination
8 Which of the following is a feature of Schnitzler's 1. Monoclonal IgG gammopathy E
Syndrome? 2. Thromboembolic events
3. Bronchospasm
4. Hematuria
5. Sensorimotor neuropathy
6. Show Explaination
9 Which of the following is an example of a large 1. Takayasu arteritis A
vessel vasculitis? 2. Wegener's granulomatosis
3. Chrug-Strauss syndrome
4. Polyarteritis nodosa
5. Henoch-Schonlein purpura
6. Show Explaination
10 In this patient with an autoimmune disorder, 1. U1RNP C
which autoantibody would be indicative of 2. DsDNA
increased risk of pulmonary disease? 3. anti-Jo-1 antibody
4. Mi-2 antibody
5. Scl-70 antibody
6. Show Explaination
11 Which statement best describes the mechanism 1. Non-steroid inhibitor of DHT binding to the E
of action of spironolactone? androgen receptor
2. Progestin
3. GnRH agonist
4. Mineralocorticoid
5. Steroid inhibitor of androgen receptor and
androgen biosynthesis
6. Show Explaination
12 A patient presents with lesion that can be 1. Calcification of the elastic fibers C
herniated into the skin on palpation. What is 2. Formation of excessive collagen
thought to be important in the pathogenesis of 3. Focal loss of elastic tissue
this condition? 4. Atrophy of the epidermis
5. Inflammation of the panniculus
6. Show Explaination
13 The lesions shown here are most likely to: 1. Occur in infants and elderly patients D
2. Occur as an adverse reaction to medication
3. Be refractory to treatment
4. Follow a streptococcal infection
5. Erupt after the appearance of a �herald
patch�
6. Show Explaination
14 Behcet's disease is associated with which HLA 1. HLA-B51 A
type? 2. HLA-B17
3. HLA-B13
4. HLA-DR4
5. HLA-Cw6

70
6. Show Explaination
15 Which of the following is true regarding this 1. Extragenital lesions commonly involve the C
entity? torso and are usually pruritic
2. Childhood onset is reported in ~5% of cases
3. 20% of both men and women with this
diagnosis have at least one autoimmune
disease
4. Girls outnumber boys 2:1
5. Genital disease represents 50% of childhood
cases
6. Show Explaination
16 Acquired C1 esterase inhibitor deficiency results 1. A normal level of serum C1q B
in: 2. A decreased level of serum C1q
3. None of these answers are correct
4. A presentation of angioedema very early in
life
5. A normal level of C4
6. Show Explaination
17 Which of the following medications is not 1. Lithium E
associated with exacerbating this condition? 2. Prednisone
3. Phenytoin
4. Isoniazid
5. Propranolol
6. Show Explaination
18 Which of the following is not a common food or 1. Dermatophagoides pteronyssimus C
environmental allergen implicated in atopic 2. Wheat
dermatitis? 3. Corn
4. Eggs
5. Fish
6. Show Explaination
19 RNP antibodies include: 1. anti-SSA, anti-centromere, anti-SSB B
2. anti-SSA, anti-SSB, anti-Sm, anti-U1RNP
3. anti-SSA, anti-SSB, anti-Sm, anti-dsDNA
4. ANA, anti-dsDNA
5. anti-SSA, anti-U1RNP, anti-centromere
6. Show Explaination
20 A 16 year old developmentally normal male 1. Pseudo-herpetic corneal ulcerations D
presents to his pediatrician intermittent vague 2. Brushfield spots
epidodes of hand and feet paresthesias and non 3. Hyperpigmentation of the retinal epithelium
specific episode of GI distress. He is referred to 4. Corneal opacities
you to evaluate numerous punctate to 5 slightly 5. Lester irides
verrucous, deep-red to blue-black papules 6. Show Explaination
distributed diffusely on his trunk in a bathing suit
distribution. Polarization microscopy of the
sediment of his urine demonstrates birefringent
lipid globules (ie, renal tubular epithelial cells or
cell fragments with lipid inclusions) with the
characteristic Maltese cross configuration. What
is the classic ocular finding in this disorder?
21 A pregnant woman in her third trimester 1. Premature labor B
presents with nonpruritic erythematous plaques 2. Placental insufficiency
and pustules in the intertriginous regions, the 3. No risk to fetus or mother
trunk, and the extremities as well as systemic 4. Small for gestational age
symptoms of malaise and fever. Which 5. Microcephaly
complication is most associated with this 6. Show Explaination
condition?
22 A 9-year-old boy presents with calcinosis cutis, 1. Fodrin C
periorbital violaceous erythema, and periungual 2. Fibrillarin
telangiectasias. Which substance is most likely 3. Thrombospondin-1
to be elevated in this patient? 4. Complement
5. Triglycerides
6. Show Explaination
23 Lacrimal gland involvement is characteristically 1. Lymphoma B
seen in patients with: 2. Sarcoid
3. Rosacea
4. Syphilis
5. Pemphigus
6. Show Explaination
24 A child has a cutaneous ossification disorder 1. Fibrodysplasia ossificans progressiva A
characterized by endochondral bone formation, 2. Progressive osseous heteroplasia
deafness, baldness, and mental retardation. 3. Plate-like osteoma cutis
Which form of osteoma cutis does this patient 4. Albright's hereditary osteodystrophy

71
have? 5. Calcinosis cutis
6. Show Explaination
25 Which of the following is an innate antimicrobial 1. A-defensin 1 B
peptide expressed by keratinocytes in response 2. A-defensin 2
to injury or inflammation? 3. TNF-A
4. IL-4
5. IL-13
6. Show Explaination
26 This variant of amyloidosis is derived from 1. Lichen amyloidosis A
degenerated tonofilaments of keratinocytes: 2. Nodular amyloidosis
3. Primary systemic amyloidosis
4. Dialysis-related amyloidosis
5. Secondary systemic amyloidosis
6. Show Explaination
27 Pilomatricomas has been associated with which 1. Acute myelogenous leukemia D
of the following conditions? 2. Multiple sclerosis
3. Crohns Disease
4. Myotonic dystrophy
5. Pneumothorax
6. Show Explaination
28 P. acnes activation of which of the following may 1. TLR-2 A
stimulate a pro-inflammatory cytokine cascade? 2. TLR-5
3. IL-1
4. IL-8
5. TNF-000
6. Show Explaination
29 Teenage boys with severe, eruptive cystic acne 1. Dimpling above the 5th MCP B
and fever, leukocytosis, myalgias and other 2. Osteolytic lesions of the clavicle
constitutional symptoms can have which of the 3. Osteopoikilosis
following bony change? 4. Stippled epiphyses
5. Jaw cysts
6. Show Explaination
30 An overweight, post-menopausal woman 1. Angiolipomas B
presents with intense pain in the fatty deposits 2. Adiposis dolorosa
on her knees, thighs and hips. She also 3. Congenital lipodystrophy
describes swelling of her hands and feet, 4. Lipodermatosclerosis
associated fatigue, and history of depression. On 5. Erythema nodosum
clinical exam the pain in the adipose tissue 6. Show Explaination
appears out of proportion to the clinical findings.
The diagnosis is:
31 Increased LDL and VLDL is seen in which type of 1. Type IIb A
hyperlipoproteinemia? 2. Type I
3. Type III
4. Type IV
5. Type V
6. Show Explaination
32 A 50-year-old female with sarcoid has 1. Mikulicz's syndrome A
enlargement of the parotid, submandibular, and 2. Heerfordt-Waldenstromn syndrome
lacrimal glands that is also known as: 3. Darier-Roussy sarcoid
4. Lofgren's syndrome
5. Blau's syndrome
6. Show Explaination
33 What is the minimum amount of time that you 1. 1 week B
would advise this woman to avoid childbearing 2. 1 month
after completing a course of isotretinoin? 3. 6 months
4. 1 year
5. 3 years
6. Show Explaination
34 Which of the following leukodermas has a normal 1. Albinism A
number of epidermal melanocytes? 2. Vitiligo
3. Piebaldism
4. Waardenburg's syndrome
5. Ziprowski-Margolis syndrome
6. Show Explaination
35 Which of the following is true regarding topical 1. Vitamin D3 analogues deactivate salicylic E
therapies for psoriasis? acid
2. Retinoids are effective in decreasing lesional
erythema
3. Anthralin can cause irreversible staining of
peri-lesional skin
4. Calcipotriol is deactivated by UV light
5. Anthralin inhibits PMNs and monocytes

72
6. Show Explaination
36 A 20 year old woman presents with a sunken 1. Decreased C3 nephritic factor B
appearance to her face and trunk with normal 2. Decreased C3
legs. The patient has attributed her appearance 3. Decreased C1
to stress from college, but is seeing you because 4. Increased C1
of her family's concern. What lab finding would 5. Increased C4
you expect in this patient 6. Show Explaination
37 Eosinophilia, asthma, neuropathy and sinus 1. Churg-Strauss syndrome A
abnormalities are associated with which of the 2. Polyarteritis nodosa
following systemic vasculitidies? 3. Microscopic polyangiitis
4. Wegener's granulomatosis
5. Kawasaki disease
6. Show Explaination
38 An atypical finding in patients with Lofgren 1. Fever E
syndrome is? 2. Arthritis
3. Subcutaneous Tender Nodules
4. Acute Iritis
5. Beaded papules on nasal rim
6. Show Explaination
39 Carcinoid tumors in which site do not lead to 1. Appendix D
flushing or other manifestations of the carcinoid 2. Ileum
syndrome? 3. Ascending colon
4. Rectum
5. Brochus
6. Show Explaination
40 Patients with this HLA type, which has the most 1. HLA-B13 D
definitive association with psoriasis, have a 2. HLA-B17
relative risk of having psoriasis that is 9-15 times 3. HLA-Bw57
normal. 4. HLA-Cw6
5. HLA-B27
6. Show Explaination
41 Which of the following would be an important 1. Lacrimal gland enlargement A
diagnostic sign of this non-infectious disorder? 2. Periungual fibromas
3. Cafe-au-lait macules
4. Peg shaped teeth
5. Dystopia canthorum
6. Show Explaination
42 The risk of developing systemic involvement in a 1. 1-4% B
patent with nodular amyloidosis is: 2. 5-10%
3. 25-30%
4. 50-60%
5. >95%
6. Show Explaination
43 This patient also has poliosis and deafness. What 1. Normal ophthalmologic exam E
is the associated ophthalmologic finding? 2. Retinal detachment
3. Lisch nodules
4. Corneal opacities
5. Granulomatous uveitis
6. Show Explaination
44 You prescribe oral erythromycin to a 35 year-old 1. Oral contraceptives E
woman. Co-administration of which of the 2. Warfarin
following medications could lead to potential 3. Carbamazepine
adverse outcomes? 4. Methylprednisolone
5. Warfarin, carbamazepine, or
methylprednisolone
6. Show Explaination
45 What is the treatment of choice for this 1. Topical antifungal medication B
condition: 2. High potency topical steroid
3. Topical antibiotic cream
4. Oral antibiotic
5. Excision
6. Show Explaination
46 What is the most common paraproteinemia in 1. IgA C
scleromyxedema? 2. IgG kappa
3. IgG lambda
4. IgM
5. It is extremely uncommon to see a
paraproteinemia with scleromyxedema
6. Show Explaination
47 Histologic examination of this chronic pruritic 1. Von Kossa A
plaque on the back, which of the following stains 2. Cotton dyes

73
would NOT be positive? 3. Crystal violet
4. PAS
5. Thioflavin T
6. Show Explaination
48 The most effective treatment for symptomatic 1. Lubricant eye drops D
ocular rosacea is: 2. Combination topical antibiotic and
corticosteroid eye drops
3. Dilute baby shampoo washes
4. Doxycycline
5. Artificial tears
6. Show Explaination
49 Most often, xanthelasma is associated with: 1. An IgM paraprotein C
2. An IgG paraprotein
3. No associated disease
4. Type I hyperlipoproteinemia
5. Type III hyperlipoproteinemia
6. Show Explaination
50 What is the diagnosis of this lesion on the penile 1. Sclerosing lymphangitis A
shaft? 2. Loaloa
3. Varicose vein
4. Syphilis
5. Lymphogranuloma venereum
6. Show Explaination
51 What is the most likely diagnosis? 1. Voight line B
2. Linea nigra
3. Fuchter line
4. Phytophotodermatitis
5. Incontinentia pigmenti
6. Show Explaination
52 A patient presents to the ER with skin pain and 1. glucose greater than 150 mg/dL B
large sheets of epidermal detachment on greater 2. age over 40 years
than 10% of his body surface area after starting 3. heart rate greater than 100
bactrim. No purpuric macules or target lesions 4. history of diabetes
are noted. Which of the following can help 5. BUN greater than 15 mg/dL
predict mortality in this patient? 6. Show Explaination
53 Patients with Gottron�s papules and periungual 1. Anti-SM E
telangiectasias will likely have a positive 2. Scl-70
serologic test to which antibody? 3. Anti-Ro
4. Anti-centromere
5. Anti- Jo-1
6. Show Explaination
54 What is the condition which is a diagnostic 1. Discoid lupus B
cutaneous manifestation of sarcoid? 2. Lupus pernio
3. Lupus vulgaris
4. Granulomatous rosacea
5. Rhinoscleroma
6. Show Explaination
55 Which of the following statements comparing 1. Human �-defensin 2 (HBD-2) and LL 37 E
inflammatory lesions from patients with psoriasis levels in psoriasis lesions are significantly
and patients with atopic dermatitis is true? lower than in atopic dermatitis lesions
2. Expression of HBD-2 and LL 37 is negligible
in psoriatic lesions
3. HBD-2 and LL 37 are significantly elevated in
atopic dermatitis lesions
4. Expression of HBD-2 and LL 37 in psoriasis is
similar to normal skin, whereas it is
decreased in atopic dermatitis lesions
5. HBD-2 and LL 37 are significantly decreased
in atopic dermatitis lesions compared to
psoriasis lesions
6. Show Explaination
56 A patient with holocarboxylase synthetase 1. Niacin C
deficiency is most at risk for developing a 2. Pyridoxine
deficiency in: 3. Biotin
4. Zinc
5. Essential fatty acids
6. Show Explaination
57 You are suspicious for a new diagnosis of lupus 1. anti-dsDNA A
in a patient recently treated with penicillamine. 2. anti-histone
Which antibodies would you expect to have been 3. anti-SSA
induced by this medication? 4. anti-SSB
5. anti-Mi-2

74
6. Show Explaination
58 Which cellular organelle is affected in the partial 1. Nucleolus E
lidodystrophies? 2. Rough endoplasmic reticulum
3. Golgi apparatus
4. Cell membrane
5. Nuclear lamina
6. Show Explaination
59 Anti-Jo-1 antibodies in patients with 1. Pulmonary fibrosis A
Dermatomyositis are associated with: 2. Cardiac disease
3. Photosensitivity
4. Calcinosis
5. Heliotrope rash
6. Show Explaination
60 Cigarette smoking has been shown to: 1. Increase the incidence of BCC E
2. Not affect the survival of surgical repairs
3. Demonstrate less solar elastosis than
photoaged skin
4. Have no correlation with facial wrinkling
5. Increase cutaneous vasoconstriction
6. Show Explaination
61 A 48 year-old man with a long history of alcohol 1. Vitamin B12 D
abuse and dependence presents with angular 2. Vitamin C
cheilitis, atrophic glossitis, a scrotal dermatitis 3. Pyridoxine
that spares the midline and extends to the 4. Vitamin B2
thighs, photophobia, and blepharitis. Which 5. Iron
nutritional deficiency do you suspect? 6. Show Explaination
62 A 2 year-old boy with crusted skin papules is 1. Osteogenesis imperfecta D
found to have osteolytic defects and diabetes 2. Conradi-Hunnerman disease
insipidus. These features are seen in: 3. Metastatic small cell lung carcinoma
4. Hand-Schuller-Christian disease
5. Epidermal nevus syndrome
6. Show Explaination
63 What is the most common malignancy 1. Brain cancer D
associated with this condition in this female? 2. Thyroid cancer
3. Breast cancer
4. Ovarian cancer
5. Liver cancer
6. Show Explaination
64 The majority of patients with positive U1RNP 1. Mixed connective tissue disorder (MCTD) C
autoantibodies have which diagnosis? 2. Rheumatoid arthritis
3. Systemic lupus erythematosus
4. Sjogren's syndrome
5. Dermatomyositis
6. Show Explaination
65 A 45-year old woman from the United States 1. Ovarian Cancer A
develops erythema and swelling around her 2. Lung Cancer
eyelids and symmetric weakness of her 3. Leukemia
shoulders and hips. What malignancy is 4. Uterine Cancer
overrepresented in patients with this condition 5. Breast Cancer
compared to the general population? 6. Show Explaination
66 A 52 year-old man presents with large 1. Erlotinib chemotherapy C
comedones as well as inflammatory papules, 2. A pituitary adenoma
pustules, and cysts on the malar cheeks, 3. 2,3,7,8 tetrachlorobenzodioxin exposure
postauricular scalp, and scrotum. Which of the 4. PTEN mutations
following could be a cause of this presentation? 5. Isotretinoin overdose
6. Show Explaination
67 In the absence of the C1 esterase inhibitor, 1. Kallikrein E
which of the following proteins mediates 2. C1
angioedema most directly: 3. Plasmin
4. C3 convertase
5. C2 kinin
6. Show Explaination
68 The pigmentation shown here is most likely due 1. Chloroquine C
to which of the following drugs? 2. Amiodarone
3. Minocycline
4. Doxycycline
5. TMP-SMX
6. Show Explaination
69 What systemic condition is often associated with 1. Diabetes mellitus E
this disease? 2. Tobacco use
3. Alopecia areata

75
4. Coronary artery disease
5. Hepatitis C
6. Show Explaination
70 Malignancy of the aerodigestive tract is most 1. Bazex's syndrome A
closely associated with which paraneoplastic 2. Paraneoplastic pemphigus
dermatosis? 3. Sweet's syndrome
4. Acanthosis nigricans
5. Dermatomyositis
6. Show Explaination
71 A patient with a recent diagnosis of tuberculosis 1. Vitamin B12 deficiency B
on treatment presents with a new photosensitive 2. Drug induced pellagra
eruption on his face, neck and upper chest. In 3. Drug induced riboflavin deficiency
addition, he has progressive diarrhea and 4. Carcinoid syndrome
depression. What is the etiology of his 5. Photoallergic contact dermatitis
symptoms? 6. Show Explaination
72 Which of the following is a clinical manifestation 1. Renal disease D
of neonatal lupus erythematosus: 2. Cerebral vascular accident
3. Arthritis
4. Thrombocytopenia
5. Neutrophilia
6. Show Explaination
73 Which of the following is true regarding neonatal 1. Most cases involve boys E
lupus erythematosus? 2. Lesions generally resolve spontaneously by 6
months, healing with scarring
3. Photosensitivity is generally not a feature
4. 75% of mothers have symptomatic systemic
lupus erythematous at the time of delivery
5. Congenital heart block may be the only
manifestation of the disease
6. Show Explaination
74 What is the most common infection that can 1. Streptococcus pyogenes A
cause guttate psoriasis? 2. Coccidiomycosis
3. Herpes
4. Mycoplasma pneumonia
5. Tuberculosis
6. Show Explaination
75 The diagnosis is: 1. Psoriasis A
2. Alopecia areata
3. Onychomycosis
4. Lichen planus
5. Tetracycline-induced photo-onycholysis
6. Show Explaination
76 A 52 year old patient with longstanding psoriasis 1. CEA D
has been treated with methotrexate for the past 2. transforming growth factor beta
6 months. He has had almost complete 3. tumor necrosis factor alpha
clearance of his disease. He now complains of a 4. Procollagen III aminopeptide
subtle right upper quadrant pain and nausea. 5. C3 and C4
Laboratory evaluation of hepatic 6. Show Explaination
aminotransferases (AST and ALT) and bilirubin
are within normal limits. Which if the following
noninvasive tests may be helpful for monitoring
methotrexate-induced hepatotoxicity?
77 Probiotics, which are cultures of potentially 1. Celiac disease B
beneficial gut microflora bacteria, have been 2. Atopic dermatitis
studied in the primary prevention of which of the 3. Psoriasis
following diseases? 4. Cutaneous T-cell lymphoma
5. Asthma
6. Show Explaination
78 Which of the following is true regarding acquired 1. This condition generally occurs in the first or C
C1 esterase inhibitor deficiency? second decade of life
2. Serum C1q is normal
3. C2 and C4 are both decreased
4. C1 esterase inhibitor may be at normal levels
with functional impairment
5. Positive family history is common
6. Show Explaination
79 A patient's biopsy is found to have curled hair 1. Hypochondriasis D
shafts with hyperkeratotic plugs. Which of the 2. Gingivitis
following is not an associated feature of this 3. Delayed wound healing
condition 4. Nail changes
5. Tender nodules on the lower extremities
6. Show Explaination

76
80 Which of the following medications is most likely 1. ciprofloxacin B
to induce a flare of this patient's disease? 2. captopril
3. diphenhydramine
4. sertraline
5. acetominophen
6. Show Explaination
81 The dermatosis pictured is most commonly 1. Insulin resistance A
associated with: 2. Cushing's syndrome
3. Acromegaly
4. Polycystic ovarian syndrome
5. Gastric carcinoma
6. Show Explaination
82 Which of the following is true regarding the 1. Ground substance is present only in B
components of amyloid? secondary cutaneous or tumor-associated
amyloidosis
2. The amyloid P component is present in all
forms
3. Protein AA is present in primary systemic
amyloidosis
4. The amyloid present in secondary systemic
amyloid does not lose its birefringence after
treatment with potassium permanganate
5. Protein AL loses its biregringence after
treatment with potassium permanganate
6. Show Explaination
83 Treatment of Raynaud's phenomenon includes: 1. Nifedipine A
2. Metoprolol
3. UVB
4. Tetracyline
5. Simvastatin
6. Show Explaination
84 In addition to the lesions seen on the feet of this 1. HLA-B27 A
young man he also had geographic tongue, 2. HLA-B51
erythematous plaques on his penis and arthritis, 3. HLA-DR1
what HLA type is associated with this syndrome? 4. HLA-DR3
5. HLA-DQW2
6. Show Explaination
85 Acute hemorrhagic edema of childhood is 1. The presence of pupura on the upper trunk E
distinguished from Henoch-Schonlein Purpura 2. The lack of an antecedent infection
based on: 3. The involvement of the synovia
4. The neurologic complications
5. The lack of systemic features
6. Show Explaination
86 The "€œfollicular occlusion triad" consists of: 1. Acne conglobata, hidradenitis suppurativa, E
kerion
2. Acne conglobata, chloracne, pyoderma
faciale
3. Acne fulminans, hidradenitis suppurativa,
dissecting cellulites of the scalp
4. Acne comedonicus, hidradenitis suppurativa,
dissecting cellulites of the scalp
5. Acne conglobata, hidradenitis suppurativa,
dissecting cellulites of the scalp
6. Show Explaination
87 Child abuse is often incorrectly suspected when 1. Molluscum contagiosum B
a young girl presents with which cutaneous 2. Lichen sclerosus et atrophicus
finding? 3. Condyloma accuminata
4. Ecchymoses on face and hands
5. Genital herpes simplex
6. Show Explaination
88 Which special stain is utilized to confirm the 1. H&E sufficient for diagnosis D
diagnosis of Langerhans cell histiocytosis? 2. S100
3. CD20
4. CD1a
5. Chloroacetate esterase
6. Show Explaination
89 Acrokeratosis paraneoplastica is most commonly 1. Squamous cell carcinoma A
associated with which of the following 2. Adenocarcinoma
malignancies: 3. Transitional cell carcinoma
4. T-cell lymphoma
5. Breast cancer
6. Show Explaination

77
90 Sweet's syndrome may be caused by: 1. Bleomycin C
2. Cytoxan
3. Granulocyte colony stimulating factor
4. Intravenous immune globulin
5. Methotrexate
6. Show Explaination
91 A patient with inflammatory bowel disease 1. Fever and malaise C
develops acute tender juicy plaques on the head 2. History of inflammatory bowel disease
and neck, fever, and malaise. The skin lesions 3. Abrupt onset of plaques
respond well to prednisone. Data shows 4. Increased ESR
increased ESR and neutrophilic infiltrate with 5. Good response to prednisone
dermal edema on skin biopsy. Which one of the 6. Show Explaination
following is a major criteria for this condition?
92 What is the treatment of choice for neurotic 1. Risperidone B
excoriations? 2. Doxepin
3. Olanzapine
4. Gabapentin
5. Diphenhydramine
6. Show Explaination
93 Spindle cell lipoma is commonly found on the: 1. Head E
2. Lower extremities
3. Buttocks
4. Breast
5. Posterior shoulder
6. Show Explaination
94 A patient with spontaneous occurrence of 1. Diffuse scaling of the scalp D
petechiae and purpura, particularly around the 2. Acne
eyes, will also most likely have which of the 3. Cheilits
following? 4. Macroglossia
5. Uveitis
6. Show Explaination
95 A 35-year-old man presents with scattered 1. Anterior uveitis A
infiltrative cutaneous plaques, chronic fever, 2. Periungual telangiectasia
parotid gland enlargement, and a facial nerve 3. Nail pits
palsy. What other finding is most likely on 4. Non-scarring alopecia
physical exam? 5. Condyloma lata
6. Show Explaination
96 A 35 year old man with a history of celiac 1. The best therapy is riboflavin 5mg/day B
disease presents with a beefy, red tongue, 2. This condition mimics folate deficiency
hyperpigmented palmar creases, and premature 3. It is often associated with carcinoid tumors
grey hair. Which of the following statements are which divert tryptophan to serotonin
is correct? 4. This condition can be caused by azithioprine,
5-FU, and isoniazid
5. Eating raw egg whites is a risk factor
6. Show Explaination
97 The diagnosis is 1. Psoriasis: D
2. Alopecia areata
3. Onychomycosis
4. Lichen planus
5. Tetracycline-induced photo-onycholysis
6. Show Explaination
98 In lichen planus pemphigoides: 1. Bullae develop characteristically in lesions of E
longstanding lichen planus
2. Circulating IgG antibodies react to the 230
kDa antigen within the basement membrane
zone
3. There is granular deposition of IgG and C3 at
the dermoepidermal junction
4. Bullae result from intense lichenoid
inflammation and extensive liquefactive
degeneration of basal keratinocytes
5. Vesicles may develop de novo on previously
uninvolved skin
6. Show Explaination
99 What is the most likely diagnosis? 1. Dermatofibromasarcoma protuberans E
2. Dermatofibroma
3. Lobomycosis
4. Blastomycosis
5. Keloid
6. Show Explaination
10 What is the treatment of choice for this patient 1. Add oral prednisone A
0 who developed acne fulminans one month after 2. Increase isotretinoin dose

78
starting isotretinoin therapy? 3. Add doxycycline
4. Acne surgery
5. Check lipid levels
6. Show Explaination
10 A patient presents with recurrent genital and 1. HLA-B27 B
1 oral ulcerations and a diagnosis of posterior 2. HLA-B51
uveitis. What HLA type is associated with the 3. HLA-DR3
diagnosis you suspect? 4. HLA-Cw6
5. HLA-DR4
6. Show Explaination
10 What is the most common cause of 1. Drugs C
2 erythroderma in non-HIV patients? 2. Underlying malignancy
3. Pre-existing dermatoses
4. Infection
5. Idiopathic
6. Show Explaination
10 Eyebrow growth: 1. Is androgen-dependent in men but not in C
3 women
2. Is regulated by dehydroepiandrosterone but
not androstenedione
3. Is not androgen-dependent
4. Requires conversion of testosterone to
dihydrotestosterone in the hair follicle
5. Is regulated only by testosterone and
dihydrotestosterone
6. Show Explaination
10 Which of the following is a manifestation of 1. Splinter hemorrhages D
4 psoriasis of the nail matrix? 2. �Oil spots�
3. Subungual hyperkeratosis
4. Pits
5. Onycholysis
6. Show Explaination
10 Concomitant discoid lupus is found in what 1. 5% B
5 percentage of patients with subacute cutaneous 2. 20%
lupus? 3. 40%
4. 60%
5. 80%
6. Show Explaination
10 Which antibody is specific for CREST syndrome? 1. Anti-mitochondrial E
6 2. Anti-histone
3. Anti-ds DNA
4. Anti-nucleolar
5. Anti-centromere
6. Show Explaination
10 Patients with psoriasis treated with cyclosporine 1. Sicca symptoms E
7 should be monitored for: 2. Hypermagnesemia
3. Alopecia
4. Acute interstitial pneumonitis
5. Hyperkalemia
6. Show Explaination
10 A 45-year-old woman develops bilateral 1. Lupus Erythematosus B
8 poikilodermatous patches without atrophy on 2. Dermatomyositis
her lateral thighs. This is most likely a 3. Mycosis Fungoides
manifestation of what disease? 4. Hepatitis C
5. Diabetes Mellitus
6. Show Explaination
10 Which of the following statements is true 1. 80% of patients with early onset disease B
9 regarding this entity? have a positive family history
2. Twin concordance (identical twins) is 55%
3. The inner root sheath and matrix of normal
hair express MHC class I
4. The sisapho pattern involves the occipital
scalp
5. Atopic dermatitis is a predictor of good
prognosis
6. Show Explaination
11 A 25 year-old previously healthy man presents 1. Females and males are equally affected B
0 with the skin findings shown, urethritis, and one 2. A chronic deforming arthritis occurs in 20%
month of peripheral arthritis. Which of the 3. TNF-alpha inhibiting agents have no role in
following is true regarding this condition? the treatment of this condition
4. Patients must have urethritis, conjunctivitis,
and arthritis for diagnosis

79
5. Chlamydia cervicitis is not associated with
this condition
6. Show Explaination
11 Which of the following disorders is primarily 1. Scleromyxedema E
1 associated with a monoclonal gammopathy of 2. Scleredema
the IgA type? 3. Necrobiotic xanthogranuloma
4. Schnitzler's syndrome
5. Pyoderma gangrenosusm
6. Show Explaination
11 The mechanism of action of azithromycin is: 1. Inhibition of bacterial cell wall synthesis C
2 2. Inhibition of RNA-dependent protein
synthesis by binding to the 30s ribosomal
subunit
3. Inhibition of RNA-dependent protein
synthesis by binding to the 50s ribosomal
subunit
4. Inhibition of DNA-dependent RNA polymerase
5. Inhibition of bacterial topoisomerase
6. Show Explaination
11 A positive ANA with a speckled staining pattern 1. Ribonucleoproteins A
3 correlates with staining of what cellular 2. Nucleolar RNA
component? 3. Native DNA
4. Kinetochore
5. Histones
6. Show Explaination
11 A 35 year old man with plaque psoriasis well- 1. The moisturizer impaired penetration of the D
4 controlled with twice daily application of calcipotriene
calcipotriene 0.005% cream noticed a flare of his 2. The moisturizer diluted the calcipotriene
psoriasis shortly after he started using a 3. The two events are unrelated
prescription moisturizer lotion for psoriasis. 4. The prescription moisturizer likely contains
Which is the most likely explanation for the lactic acid.
apparent decreased efficacy of calcipotriene? 5. The decreased efficacy is due to
tachyphylaxis
6. Show Explaination
11 The most common associated disorder in a 1. Marfan syndrome B
5 patient with elastosis perforans serpiginosa is: 2. Down syndrome
3. Osteogenesis imperfecta
4. Ehlers-Danlos syndrome
5. Rothmund-Thomsom syndrome
6. Show Explaination
11 Lichen planus pemphigoides has been 1. Type XVII collagen A
6 associated with an antigen to which structure? 2. Hemidesmosome plaque
3. Desmoglein 3
4. Desmoglein 1
5. Type VII collagen
6. Show Explaination
11 Which of the following cytokines is most 1. Interferon-gamma A
7 characteristic of atopic dermatitis in its chronic 2. IL-4
phase? 3. IL-5
4. IL-10
5. IL-13
6. Show Explaination
11 Menopausal flushing can be effectively treated 1. Nadolol C
8 with: 2. Tryptophan
3. Clonidine
4. Danazol
5. Tamoxifen
6. Show Explaination
11 Which of the following hormones bind the 1. Dehydroepiandrosterone C
9 androgen receptor? 2. Androstenedione
3. Dihydrotestosterone
4. Dehydroepiandrosterone and
dihydrotestosterone
5. Dehydroepiandrosterone, androstenedione,
and dihydrotestosterone
6. Show Explaination
12 Bullous lupus erythematosus is most commonly 1. Type IV collagen E
0 associated with antibodies to: 2. Laminin 5
3. Desmoglein I
4. Plectin
5. Type VII collagen

80
6. Show Explaination
12 Which of the following complement profiles 1. C2 high and C4 high E
1 would suggest a diagosis of hereditary 2. C1 low and C4 high
angioedema (C1 esterase inhibitor deficiency)? 3. C3 high and C1-inh decreased
4. CH 50 low and C1-inh high
5. C1-inh normal, C1 normal and C4 low
6. Show Explaination
12 What is the most common ocular finding in this 1. Blepharitis B
2 patient with this granulomatous disease? 2. Acute anterior uveitis
3. Posterior uveitis
4. Keratoconus
5. Cataracts
6. Show Explaination
12 You prescribe doxycyline to a 22 year-old woman 1. Tetracycline E
3 with acne. Your patient takes an oral 2. Minocycline
contraceptive to prevent pregnancy. Your patient 3. Azithromycin
read in a magazine that the doxycycline may 4. TMP-SMX
decrease the efficacy of her contraceptive, and 5. Rifampin
asks you about this. Which of the following 6. Show Explaination
antibiotics has been definitively shown to reduce
contraceptive efficacy?
12 Immunologic abnormalities in atopic dermatitis 1. Increased CD8 T-cell number and function D
4 include: 2. Increased secretion of IFN-gamma
3. Decreased expression of CD23 on B cells and
monocytes
4. Increased secretion of IL-4
5. Accentuated DTH response
6. Show Explaination
12 What organism caused this disease? 1. Micrococcus sedenditaris A
5 2. Candida albicans
3. Staphylococcus aureus
4. Corynebacterium tenuis
5. Corynebacterium diptheria
6. Show Explaination
12 Most common association with generalized 1. Hodgkin's disease E
6 lesions shown in attached image is 2. Non-Hodgkin's lymphoma
3. Granulomatous mycosis fungoides
4. HIV infection
5. Diabetes mellitus
6. Show Explaination
12 Which of the following is NOT true regarding 1. AA is not the protein component B
7 primary cutaneous amyloidosis? 2. Amyloid is present around blood vessels
3. AL is not the protein component
4. Notalgia paresthetica may be associated with
the macular form
5. Amyloid found in benign and malignant
neoplasms does not represent primary
cutaneous amyloidosis
6. Show Explaination
12 Which contact allergen is found in Krazy glue? 1. Formaldehyde C
8 2. Lanolin alchohol
3. Ethyl cyanoacrylate
4. Octyl-dimethyl-paba
5. Mercaptobenzothiazole
6. Show Explaination
12 A 48 year old woman was recently diagnosed 1. Breast exam D
9 with dermatomyositis. Which examination would 2. Chest xray
be most important in a work up for malignancy? 3. Thyroid exam
4. Pelvic exam
5. Stool guiac
6. Show Explaination
13 The differential diagnosis for the attached image 1. Candidiasis E
0 should include all of the followings except 2. Lichen planus
3. Contact dermatitis
4. Inverse psoriasis
5. Erythrasma
6. Show Explaination
13 After being treated for several months with 1. Tetracycline C
1 doxycycline, this patient develops a gram 2. Bactrim
negative folliculitis. What is the next appropriate 3. Isotretinoin
therapy? 4. Ceftriaxone
5. Cefepime

81
6. Show Explaination
13 A 42 year-old woman has the findings pictured, 1. Anti-SRB C
2 as well as erythematous patches on the upper 2. Anti-Jo-1
chest and shoulders. No Raynaud's, polyarthritis, 3. Anti-Mi-2
pulmonary, or cardiac involvement is noted. 4. Anti-Ku
Which antibody might you expect to be present 5. Anti-La
in this patient? 6. Show Explaination
13 Secondary systemic amyloidosis: 1. Classically involves the tongue and D
3 periorbital skin
2. Involves deposition of AL protein
3. Can involve deposition of beta 2-
microglobulin in the setting of rheumatoid
arthritis
4. Can be noted on biopsy of normal skin
5. Involves deposition of keratin-derived
amyloid
6. Show Explaination
13 MAGIC syndrome involves: 1. Relapsing polychondritis A
4 2. Acne conglobata
3. Livedo reticularis
4. Lipodystrophy
5. Psoriatic arthritis
6. Show Explaination
13 What is the diagnosis? 1. Psoriasis C
5 2. Lichen planus
3. Balanitis circinata
4. Candida
5. Syphilis
6. Show Explaination
13 A patient has allergic contact dermatitis to 1. Adhesive B
6 paraphenylenediamine. Which of the following 2. Sulfa drugs
allergens may show a potential cross- reaction? 3. Lidocaine
4. Fragrance
5. Nickel
6. Show Explaination
13 A 40 year-old man presents with a complaint of 1. 50% of patients present with joint disease D
7 nail changes for several years. He was treated prior to skin involvement
by an outside physician with terbinafine without 2. Bony erosions are not commonly seen on
improvement. On further questioning, he reports radiographs
morning shoulder stiffness and back pain that 3. Cyclosporine is effective in treating the
lasts 1-2 hours and improves with activity. Which arthritis in this condition
of the following is true regarding his condition? 4. A positive rheumatoid factor may be seen
5. Joint disease correlates with severity of skin
involvement
6. Show Explaination
13 Xanthoma striata palmaris are diagnostic of: 1. Familial hypertriglyceridemia (type IV) B
8 2. Familial dysbetalipoproteinemia (type III)
3. Familial lipoprotein lipase deficiency (AR)
4. Familial lipoprotein lipase deficiency (AD)
5. Apoprotein CII deficiency
6. Show Explaination
13 A patient presents with recurrent crops of 1. CD 4 C
9 papules that ulcerate and then spontaneously 2. CD 20
heal. What immunohistochemical stain would be 3. CD 30
helpful in making the diangosis? 4. CD 56
5. CD 68
6. Show Explaination
14 Periorbital purpura is a characteristic cutaneous 1. Sarcoidosis B
0 manifestation of: 2. Amyloidosis
3. Dermatomyositis
4. Lymphoma
5. Thrombocytopenia
6. Show Explaination
14 Which of the following disorders is most strongly 1. Scleredema E
1 associated with a monoclonal gammopathy of 2. POEMS syndrome
the IgG-lambda type? 3. Necrobiotic xanthogranuloma
4. EED
5. Scleromyxedema
6. Show Explaination
14 The most important mediator of retinoid activity 1. RAR-alpha C
2 in the skin is: 2. RAR-beta

82
3. RAR-gamma
4. RXR-alpha
5. RXR-beta
6. Show Explaination
14 Which is the most common site for abdominal 1. Ileum B
3 carcinoid tumors causing the classic carcinoid 2. Appendix
syndrome? 3. Rectum
4. Bronchus
5. Liver
6. Show Explaination
14 What is the most common paraproteinemia in 1. IgA C
4 scleromyxedema 2. IgG kappa
3. IgG lambda
4. IgM
5. It is uncommon to see paraproteinemia with
scleromyxedema
6. Show Explaination
14 What complication can occur if this disease is 1. Development of a squamous cell carcinoma A
5 left untreated? 2. Testicular torsion
3. Erectile dysfunction
4. Impotence
5. Pseudo-ainhum
6. Show Explaination
14 This woman is being treated for corticosteroid- 1. Yellow E
6 induced rosacea with topical metronidazole and 2. Lavender
an oral tetracycline. She is very concerned about 3. Pink
the redness of her face. Judicious use of what 4. Bronze
color concealer can reduce the appearance of 5. Green
redness on the skin? 6. Show Explaination
14 An elderly gentleman with rheumatoid arthritis 1. Chondrodermatitis nodularis helices C
7 has a 2-month history of recurrent painful, red, 2. Recurrent otitis externa
swollen ears and hearing loss. Physical exam is 3. Relapsing polychondritis
notable for sparing of the earlobes. You suspect 4. Systemic lupus erthematosus
he has: 5. Severe seborrheic dermatitis
6. Show Explaination
14 Perifolicular depigmentation is a characteristic 1. Dermatomyositis C
8 cutaneous manifestation of: 2. Secondary syphilis
3. Scleroderma
4. Discoid lupus
5. Sarcoidosis
6. Show Explaination
14 Pyoderma gangrenosum is most commonly 1. Leukemia A
9 reported with which malignancy? 2. Medullary thyroid carcinoma
3. Lung carcinoma
4. Breast carcinoma
5. Prostate carcinoma
6. Show Explaination
15 First line therapy for subcorneal pustular 1. Dapsone A
0 dermatosis is: 2. Tetracycline
3. Vitamin E
4. Topical retinoids
5. Topical steroids
6. Show Explaination
15 Syndrome that is associated with disease shown 1. SAPHO syndrome E
1 in image is known as 2. Follicular occlusion tetrad syndrome
3. LEOPARD syndrome
4. NAME syndrome
5. PAPA syndrome
6. Show Explaination
15 A patient with this cutanous finding along with 1. Anti-Jo-1 antibodies against histidyl-tRNA A
2 peri-ocular erythema, deltoid weakness and synthetase
pulmonary disease is most likely to have which 2. Anti-Jo-1 antibodies against nuclear helicase
laboratory finding? 3. c-ANCA against proteinase-3
4. Lupus anticoagulant
5. Anti-histone antibodies
6. Show Explaination
15 Which of the following is associated with this 1. Thyroid disease B
3 scarring condition? 2. Acne conglobata
3. Sarcoidosis
4. Systemic lupus erythematosus
5. Lichen planus

83
6. Show Explaination
15 Attached image can be associated with all of the 1. Smooth muscle hamartoma E
4 followings excpet: 2. Unilateral breast hypoplasia
3. Acneform lesion
4. Skeletal defect
5. Cardiac defect
6. Show Explaination
15 Androstenedione is produced by: 1. The adrenals C
5 2. The ovaries
3. The adrenals and ovaries
4. Extraglandular conversion
5. The adrenals, ovaries, and extraglandular
conversion
6. Show Explaination
15 Patients with Dermatitis Herpetiformis are most 1. Antibodies to BPAg2 B
6 likely to have: 2. Antibodies to transglutaminase 3
3. Mutations in transglutaminase I
4. Mutations in laminin 5
5. Mutations in plectin
6. Show Explaination
15 This radiograph shows a characteristic finding of 1. Arthritis mutilans D
7 severe psoriatic arthritis: bone proliferation at 2. Axial
the base of the distal phalanx with resorption of 3. Symmetric polyarthritis
the tufts. What is the most common form of 4. Asymmetric oligoarthritis
psoriatic arthritis? 5. Symmetric oligoarthritis
6. Show Explaination
15 Topical calcineurin inhibitors may offer several 1. More cost effective D
8 benefits over topical steroids in treating the 2. Quicker onset of action
scaly, lichenified periorbital plaques in this 3. Decreased irritant potential
patient with atopic dermatitis, including: 4. Absence of reports of cutaneous atrophy
5. Once daily application
6. Show Explaination
15 A 3 year-old girl has a red-yellow papule on her 1. Plastic surgeon D
9 cheek which her mother is extremely anxious 2. Endocrinologist
about. Skin biopsy reveals a juvenile 3. Neurologist
xanthogranuloma. Which specialist should you 4. Ophthalmologist
refer the patient to? 5. Oncologist
6. Show Explaination
16 A patient presents with hemorrhagic 1. Quinolone antibiotics D
0 onycholysis. The drug class most commonly 2. Systemic retinoids
associated with this finding is: 3. Calcineurin inhibitors
4. Taxanes
5. Tetracyclines
6. Show Explaination
16 A 35 year-old otherwise healthy man presents 1. The patient has exceeded the recommeded B
1 with moderate-to-severe plaque psoriasis, dosage of cyclosporine
improved on cyclosporine. The patient weighs 70 2. You offer a decrease of the cyclosporine dose
kilograms and is taking a dose of 300 mg/day. to 225 mg/day and close follow-up
His baseline creatinine was 0.8; on follow-up 3. The change in creatinine is not significant, no
testing it is 1.1. All other exam and laboratory change is needed
parameters are within normal limits. The patient 4. Cyclosporine rarely has renal toxicity in
is pleased with his treatment and asks to young, healthy individuals; thus you must
continue it. Which of the following is correct? work-up other causes of the increased
creatinine
5. The cyclosporine should be stopped
immediately; the patient must avoid
cyclosporine in the future
6. Show Explaination
16 Which of the following is true regarding this 1. The protein deposited is derived from Ig light D
2 diagnosis? chains, kappa subtype
2. Skin is involved in <10% of cases
3. A different protein is found in skin lesions
associated with a plasmacytoma
4. Bullae, when present, are subepidermal
5. Factor V deficiencies are commonly
associated with this entity
6. Show Explaination
16 This 57-year old male complains of weakness 1. IL-1 E
3 when climbing the stairs and this clinical 2. Il-10
presentation. Which cytokine has been 3. Interferon-alpha
imnplicated in the etiology of this condition? 4. Interferon-gamma
5. Tumor necrosis factor alpha

84
6. Show Explaination
16 A 32 year old woman, now 12 weeks pregnant, 1. It has been associated with EBV. E
4 presents to your office with pruritic scaly 2. There is often a flare post-partum and during
papules and plaques. A biopsy reveals focal subsequent pregnancies.
spongiosis and parakeratosis in mounds, a 3. There is no increased incidence in
superficial perivascular dermatitis, and immunocompromised patients.
extravasated red blood cells in the dermis. 4. There is an increased risk of miscarriage in
Which of the following is true? mothers who developed pityriasis rosea
within the first 26 weeks of their pregnancy
5. Acyclovir may be effective in this condition
6. Show Explaination
16 Which systemic anesthetic can be safely used in 1. Lidocaine E
5 mastocytosis? 2. Succinylcholine
3. D-tubocurarine
4. Thiopental
5. Propofol
6. Show Explaination
16 Phrynoderma is associated with deficiency of 1. Vitamin A A
6 which vitamin? 2. Vitamin B1
3. Vitamin B6
4. Vitamin D
5. Vitamin K
6. Show Explaination
16 What type of amyloid is deposited into the skin 1. Amyloid AA C
7 of this pruritic disorder? 2. Amyloid AL
3. Keratin derived
4. Beta-2 microglobulin
5. Transthyretin
6. Show Explaination
16 Which of the following medications is most likely 1. Hydrochlorothiazide D
8 to induce or exacerbate psoriasis in your patient. 2. Lisinopril
3. Amlodipine
4. Metoprolol
5. Diltiazem
6. Show Explaination
16 This 10 year-old girl presents to your office. 1. They should expect her to get more lesions B
9 These lesions have been present for months. in none sun-exposed areas
She denies any other systemic complaints. What 2. She is more likely to develop systemic lupus
will you tell her parents? erythematosus than an adult with these
lesions
3. She is less likely than an adult to develop
renal disease
4. She is more likely than an adult to develop
cardiovascular disease
5. No hematologic labs are required for
evaluation
6. Show Explaination
17 Which of the followings is true about nevus 1. Usually occurs in association with vitiligo C
0 anemicus 2. Occurs more frequently in men than in
women
3. Most commonly involves the upper chest
4. Results from locally decreased vascular
reactivity to catecholamines
5. Contains dilated blood vessels
6. Show Explaination
17 Best treatment option for this stable type of 1. Phototherapy with narrow band UVB D
1 vitiligo is 2. Excimer laser
3. Oral prednisone
4. 20% monobenzyl ether of hydroquinone
5. Nitrogen mustard
6. Show Explaination
17 A pregnant woman presents with mild 1. Topical erythromycin/benzoyl peroxide gel D
2 inflammatory acne which is very bothersome to 2. Topical tretinoin
her. Which of the following is the most 3. Tazarotene 0.1% cream
appropriate treatment option based on FDA 4. Azelaic acid 20% cream
classifications of medication in pregnancy? 5. Bactrim
6. Show Explaination
17 Sarcoidosis presenting as uveitis, facial nerve 1. Heerfordt's syndrome A
3 palsy, fever and parotid gland swelling is 2. Lofgren's syndrome
referred to as: 3. Lupus pernio
4. Darier-Roussy disease
5. Schaumann syndrome

85
6. Show Explaination
17 Carcinoma of which of the following has been 1. Lung cancer A
4 most associated with erythema gyratum repens 2. Breast cancer
3. Colon cancer
4. Prostate cancer
5. Upper GI tract cancer
6. Show Explaination
17 Sneddon-Wilkinson Disease: 1. Most often occurs in elderly women D
5 2. Rarely involves intertriginous areas
3. May occur in association with an IgG
monoclonal gammopathy
4. Can be treated with narrow band UVB
5. Is usually an acute, self-limited condition
6. Show Explaination
17 A young woman presents with tender, 1. TSH A
6 erythematous nodules an the anterior lower 2. ESR
extremities. Which of the following would not be 3. ASO
an appropriate test to consider? 4. Fungal cultures
5. PPD
6. Show Explaination
17 Cutaneous manifestations of vitamin D 1. Alopecia A
7 deficiency include: 2. Follicular hyperkeratosis
3. Edema
4. Angular cheilitis
5. Atrophic glossitis
6. Show Explaination
17 Which of the following dermatoses occurs at the 1. Darkening of nevi B
8 latest stage of pregnancy? 2. Linea nigra
3. Melasma
4. Areolar hyperpigmentation
5. Psoriasis exacerbation
6. Show Explaination
17 A common site for chloracne is the: 1. Occipital scalp C
9 2. Forehead
3. Scrotum
4. Forearms
5. Shoulders
6. Show Explaination
18 Acute hemorrhagic edema of childhood often 1. Facial edema A
0 presents initially with: 2. Laryngospasm
3. Acute abdomen
4. Hematuria
5. Hematochezia
6. Show Explaination
18 Neutrophilic dermatoses en plaque is often 1. IgA monoclonal gammopathy and a benign A
1 associated with which of the following? course
2. IgA monoclonal gammopathy and a
malignant course
3. Myeloma and B lymphomas and a benign
course
4. IgG monoclonal gammopathy and a benign
course
5. IgG monoclonal gammopathy and a
malignant course
6. Show Explaination
18 Which of the following is important in the 1. Activation of toll-like receptor-3 by P. acnes C
2 pathogenesis of acne vulgaris? 2. Activation of toll-like receptor-2 by M. furfur
3. Activation of toll-like receptor-2 by P. acnes
4. P. acnes produces lipase which cleaves
cholesterol into triglycerides
5. Demodex activates complement
6. Show Explaination
18 A 35 year-old woman presents with 1. Congenital adrenal hyperplasia B
3 predominantly lower facial inflammatory acne, 2. Adrenal tumor
hirsutism, and irregular menses. She has failed 3. Polycystic ovary syndrome
multiple conventional treatments. Laboratory 4. Toxicity from prolonged use of spironolactone
work-up reveals a very high level of DHEA-S. You 5. Progestin excess from oral contraceptives
are most concerned about: 6. Show Explaination
18 Which of the following cytokines is associated 1. IL-4 D
4 with follicular plugging and microcomedo 2. IL-13
formation? 3. TNF-á

86
4. IL-1
5. TGF-ooo
6. Show Explaination
18 Acneiform eruptions have been associated with 1. Vitamin C E
5 which of the following vitamins? 2. Vitamin E
3. Vitamin A
4. Biotin
5. Vitamin B12
6. Show Explaination
18 One of your acne patients has been treated with 1. Tetracycline C
6 doxycycline for several months and develops 2. Bactrim
culture positive gram negative folliculitis. What 3. Isotretinoin
is the next appropriate therapy 4. Ceftriaxone
5. Cefepime
6. Show Explaination
18 Which of the following is a progestin used in oral 1. Norgestrel C
7 contraceptives with low intrinsic androgenic 2. Norethindrone
properties? 3. Norgestimate
4. Levonorgestrel
5. Spironolactone
6. Show Explaination
18 A young girl presents with recurrent severe 1. PSTPIP1 A
8 arthritis of the ankles. She also has a large ulcer 2. NOD2
on her leg and severe acne. Which gene is 3. CIAS1
mutated? 4. AIRE
5. FOXP3
6. Show Explaination
18 Which of the following is true regarding Seip- 1. This condition is acquired D
9 Lawrence Syndrome? 2. Mental/cognitive function is unaffected
3. Thyroid function abnormalities cause heat
intolerance
4. Hypertrichosis is often a feature
5. Basic metabolic rate is depressed
6. Show Explaination
19 In a well-designed study, the impact of psoriasis 1. Diabetes A
0 on health-related quality of life was found to be 2. Acne vulgaris
similar to which of the following conditions? 3. Onychomycosis
4. Seasonal allergic rhinitis
5. Hypercholesterolemia
6. Show Explaination
19 The most common laboratory abnormality in 1. Decreased white blood cell count D
1 patients treated with isotretinoin is: 2. Increased cholesterol
3. Elevated liver enzymes
4. Hypertriglyceridemia
5. Elevated CPK
6. Show Explaination
19 ACE inhibitors cause angioedema via stimulation 1. Bradykinins A
2 of? 2. Histamine
3. Prostaglandins
4. Leukotrienes
5. Complement
6. Show Explaination
19 What name is commonly used to describe this 1. Actinic granuloma E
3 condition of nodular elastosis with cysts and 2. Chlorance
comedones? 3. Cutis rhomboidalis nuchae
4. Colloid milium
5. Favre-Racouchot
6. Show Explaination
19 Which auto-antibody correlates with this finding, 1. Anti-Mi-2 antibody A
4 shawl sign and an overall favorable prognosis in 2. Anti-Jo-1 antibody
Dermatomyositis? 3. Anti-SRB antibody
4. Anti-KU antibody
5. Anti-PL7 antibody
6. Show Explaination
19 A 32 year-old pregnant woman presents for 1. Benzoyl peroxide D
5 treatment comedonal and moderately 2. Trimethoprim-sulfamethoxazole
inflammatory acne. You discuss treatment 3. Topical tretinoin
options with her, and she asks about the 4. Azeleic acid
evidence for safety in pregnancy of various 5. Tetracycline
treatments. According to the FDA classifications, 6. Show Explaination
which of the following treatments either shows

87
no risk to the fetus in controlled studies (but
may show risk to animals), or shows no risk in
animal studies (but no human studies have been
conducted)?
19 What condition does this patient have given the 1. Antiphospholipid syndrome B
6 extent of arthritis seen in this X-ray? 2. Multicentric reticulohistiocytosis
3. Alpha-1-antitrypsin syndrome
4. Eosinophilic fasciitis
5. Dermatomyositis
6. Show Explaination
19 A pregnant woman presents with the following 1. Topical erythromycin/benzoyl peroxide gel D
7 condition which is bothersome to her. Which of 2. Topical tretinoin
the following is the most appropriate treatment 3. Tazarotene 0.1% cream
option based on FDA classifications of 4. Azelaic acid 20% cream
medication in pregnancy? 5. Bactrim DS
6. Show Explaination
19 The presentation of a foregut carcinoid tumor 1. The production of histamine, cyanotic flush, B
8 involves: and bronchoconstriction
2. The production of serotonin, intense flushing,
peptic ulcer, and lacrimation
3. To cutaneous findings
4. The production of kallikrein with bronchial
asthma and angioedema
5. Frequent episodes of tongue swelling and
urticaria
6. Show Explaination
19 What is the major constituent of sebaceous 1. Triglyceride A
9 gland lipid? 2. Wax esters
3. Squalene
4. Free cholesterol
5. Cholesterol esters
6. Show Explaination
20 Leukokeratosis nicotina palati: 1. Is not attributed to smoking D
0 2. Represents a premalignant condition
3. Responds to intralesional steroids
4. Represents inflamed minor salivary glands
5. Resolves with tetracycline therapy
6. Show Explaination
20 A 40 year old women presents to the clinic with 1. Sneddon-Wilkinson disease A
1 multiple pustules in annular and serpiginous 2. Reiter's syndrome
patterns on the abdomen, axillae and groin. 3. SAPHO syndrome
Histopathology reveals pustules below the 4. Transient pustular melanosis
stratum corneum with many neutrophils and 5. Pthirus pubis
without any acantholysis. What is the diagnosis? 6. Show Explaination
20 Which of the following forms of angioedema is 1. Hereditary angioedema A
2 mediated by kinin release? 2. Acute allergic angioedema
3. Angioedema associated with urticarial
vasculitis
4. NSAID angioedema
5. Infectious angioedema
6. Show Explaination
20 The first step in the formation of a closed 1. Obstruction of the pilosebaceous duct A
3 comedone is: 2. Increased sebum production
3. Colonization by P. acnes
4. Increased hormonal stimulation of the
pilosebaceous unit
5. Clogging of the pore with make-up
6. Show Explaination
20 Immunohistochemistry of this disease would 1. Perivascular IgA B
4 likely show: 2. IgM staining of colloid bodies
3. Linear band of IgA at the basement
membrane
4. IgM intercellularly
5. Granular C3 at the basement membrane
6. Show Explaination
20 The "flag sign" is seen in which of the following 1. psoriasis D
5 conditions? 2. sarcoidosis
3. hidradenitis suppurativa
4. kwashiorkor
5. telogen effluvium
6. Show Explaination
20 The hyperproliferative epithelium of this mature 1. K1, K10 C

88
6 psoriasis plaque is associated with increased 2. K5, K14
expression of which keratin(s)? 3. K6, K16
4. K17
5. K2e
6. Show Explaination
20 Which of the following is characteristic of lichen 1. Lesions are typically hypopigmented D
7 planus pigmentosus? 2. Most cases present in Caucasians
3. Oral involvement is pathognomonic
4. Can occasionally see epidermotropic T-cells
5. Trunk is typically spared
6. Show Explaination
20 What is the treatment of choice for this 1. Nystatin C
8 condition? 2. Fluconazole
3. Better oral hygiene
4. Penicillin
5. Acyclovir
6. Show Explaination
20 A 22 year-old woman notes irregular menses and 1. Serum prolactin levels D
9 acne that has been refractory to several over- 2. 24-hour urine cortisol
the-counter and prescription regimens. 3. Serum free and total testosterone, LH, FSH,
Appropriate initial work-up includes androstenedione
4. Serum free and total testosterone, LH, FSH,
DHEA-S
5. Serum free and total testosterone, LH, FSH,
17-OH, hydroxylase
6. Show Explaination
21 A premature infant who is being weaned off 1. Parenteral nutrition D
0 breast milk develops vesicobullous and 2. Stress (i.e. infection)
eczematous skin lesions and diarrhea. Which of 3. Diets with mainly cereal grains
the following is not another classic precipitant 4. Liver disease
for this condition 5. Alcoholism
6. Show Explaination
21 The diagnosis is: 1. Onychomycosis E
1 2. Trauma-induced nail changes
3. Chronic paronychia
4. Pseudomonal infection
5. Psoriasis arthritis
6. Show Explaination
21 Which of the following is true regarding relapsing 1. Involvement is often bilateral D
2 polychondritis? 2. Pathogenic antibodies have not yet been
identified
3. The course is chronic, yet mortality is low
4. Both sexes are equally affected
5. Migratory arthralgias are uncommonly seen
6. Show Explaination
21 Patients of which HLA type are more prone to 1. HLA-Cw6 D
3 drug-induced lupus erythematosus when 2. HLA-B7
exposed to hydralazine? 3. HLA-DR3
4. HLA-DR4
5. HLA-B6
6. Show Explaination
21 Melanocytes can be found in all of the following 1. Nevus depigmentosa D
4 except: 2. Tyrosinase positive albinism
3. Nevus anemicus
4. Vitiligo
5. Postinflammatory hypopigmentation
6. Show Explaination
21 Which of the following is a manifestation of 1. Splinter hemorrhages E
5 psoriasis of the nail matrix? 2. Oil spots
3. Subungal hyperkeratosis
4. Onycholysis
5. Pits
6. Show Explaination
21 Which of the following is true regarding 1. Familial mediterranean fever involves AL C
6 syndromes associated with amyloidosis? protein
2. MEN IIa involves AA protein
3. The syndrome that presents with renal
amyloidosis, fevers, limb pains, and deafness
involves AA protein
4. Familial amyloidotic polyneuropathy type IV
involves mutations in apolipoprotein A-1
5. Familial amyloidotic polyneuropathy type III

89
involves mutations in gelsolin
6. Show Explaination
21 A healthy 50 year old man presents to your 1. Q-switched alexandrite laser B
7 office with 3 red-brown indurated plaques on his 2. Pulse dye laser
forehead. Biopsy reveals a focal leukocytoclastic 3. Q-switched ruby laser
vasculitis, with a diffuse infiltration of 4. Nd:YAG laser
neutrophils, eosinophils, and lymphocytes. 5. Frequency-doubled Nd:YAG laser
Which of the following is a potential treatment 6. Show Explaination
option for this patient?
21 Which of the following is true regarding atopic 1. Over 80% of affected individuals present in B
8 dermatitis? the first year of life
2. Natural measles infection has been noted to
improve atopic dermatitis
3. Most children develop worsening of atopic
dermatitis if they develop asthma later in
childhood
4. Staphylococcus aureus is found in ~25% of
atopic dermatitis skin lesions
5. Increased expression of cathelicidins such as
LL 37 has been noted in atopic dermatitis
6. Show Explaination
21 Which of the following medications would likely 1. Hydrochlorothiazide D
9 exacerbate your patient's psoriasis? 2. Lisinopril
3. Amlodipine
4. Metroprolol
5. Diltiazem
6. Show Explaination
22 In the setting of the lupus erythematosus- 1. low C3 and C4 C
0 associated complement deficiency syndrome, 2. low C2 and C3
which of the following would most often be 3. low C2 and C4
observed? 4. low C2 only
5. low C3 only
6. Show Explaination
22 Which genodermatosis may be associated with 1. Cowden syndrome D
1 this plaque of skin between the shoulder blades 2. Peutz-Jeghers syndrome
with rippled hyperpigmented appearance? 3. Gardner syndrome
4. Sipple syndrome
5. Dyskeratosis Congentia
6. Show Explaination
22 Pyostomatitis vegetans is characteristically 1. Pemphigus vulgaris B
2 associated with which systemic disease? 2. Ulcerative colitis
3. Lichen planus
4. Lymphoma
5. Rheumatoid arthritis
6. Show Explaination
22 Patients with xanthoma disseminaturn may 1. Elevated serum beta lipoproteins C
3 demonstrate: 2. Sparing of the oral mucosa
3. Involvement of the pituitary gland
4. Sparing of the flexural areas
5. Increased risk of malignant degeneration
6. Show Explaination
22 The protein component of primary cutaneous 1. SAA protein C
4 amyloidosis is: 2. AL protein
3. Keratin
4. Collagen
5. Bp180
6. Show Explaination
22 Which immunoglobulins are seen in type III 1. Monoclonal IgG and monoclonal IgM E
5 cryoglobulinemia? 2. Monoclonal IgG or monoclonal IgM
3. Monoclonal IgM and polyclonal IgG
4. Polyclonal IgM and monoclonal IgG
5. Polyclonal IgM and polyclonal IgG
6. Show Explaination
22 What is deposited in the upper dermis in this 1. AK A
6 condition? 2. AL
3. Transthyretin
4. Beta2-microglobulin
5. AA
6. Show Explaination
22 The most appropriate treatment for the 1. Topical clindamycin 1% lotion E
7 condition shown is: 2. Topical metronidazole 0.75% gel
3. Azelaic acid 20% cream

90
4. Salicylic acid 2% wash
5. Adapalene 0.1% gel
6. Show Explaination
22 Most common cause of these tender lesions on 1. Inflammatory bowel disease C
8 this 12-year old child is 2. Tuberculosis
3. Îbeta-hemolytic streptococcal infection
4. Yersinia
5. Infectious mononucleosis
6. Show Explaination
22 What is the most common variant of morphea in 1. Plaque E
9 children? 2. Generalized
3. Bullous
4. Deep (morphea profunda)
5. Linear
6. Show Explaination
23 The most common cause of internal 1. Esophageal dysmotility A
0 manifestation of scleroderma is: 2. Sepsis
3. Pulmonary hypertension
4. Acute renal failure
5. Raynaud's phenomenon
6. Show Explaination
23 Sarcoidosis presenting with fever, cough, joint 1. Erythema contusiforme C
1 pains, hilar adenopathy and erythema nodosum 2. Loeffler's syndrome
is known as: 3. Lofgren's syndrome
4. Darier-Roussy sarcoid
5. Heerfordt's syndrome
6. Show Explaination
23 A young man presents with explosive onset of 1. Women are more often affected than men C
2 severe cystic acne with acute, suppurative 2. P. acnes osteomyelitis presents with lytic
nodules and plaques that ulcerate and form a changes on x-rays and bone scans
blackish eschar on the trunk as well as the face. 3. The sternoclavicular joint is often involved in
Which of the following is true regarding this this entity
entity? 4. Systemic corticosteroids are contraindicated
given risk of sepsis
5. High-dose isotretinoin monotherapy is the
treatment of choice
6. Show Explaination
23 You are consulted to evaluate this patient with 1. Exophthalmos B
3 tender, warm plaques on his shins. What other 2. Uveitis
physical signs should you look for? 3. Tachycardia
4. Clubbing of the fingers
5. Cough
6. Show Explaination
23 Which autoantibodies are associated with an 1. anti-SRP D
4 increased risk of malignancy in 2. anti-Mi2
dermatomyositis? 3. Anti-Jo1
4. anti-155/140
5. anti-PL-7
6. Show Explaination
23 Natural infection with which of the following 1. Measles A
5 infectious agents has been shown to mitigate 2. S. aureus
atopic dermatitis? 3. Varicella
4. Rubella
5. Coxsackie virus
6. Show Explaination
23 Diarrhea, Dementia and a photosensitive 1. Niacin A
6 dermatitis are associated with a deficiency of 2. Biotin
which vitamin? 3. Thiamine
4. Riboflavin
5. Pyridoxine
6. Show Explaination
23 This 35 year-old man presents with the lesions 1. Increased staining on Fontana Masson but C
7 shown. He was treated elsewhere for a different not Perls stain
skin condition. Biopsy of these lesions is likely to 2. Increased staining on Perls stain but not
show: Fontana Masson
3. Increased staining on both Fontana Masson
and Perls stain
4. Increased melanin at the basal layer and
within macrophages only
5. Fibrosis and increased mucin deposition
6. Show Explaination

91
23 The most notable histologic difference between 1. Acanthosis B
8 the oral and cutaneous lesions of lichen planus is 2. Parakeratosis
the presence in oral LP of: 3. Compact orthokeratosis
4. Wedge-shaped hypergranulosis
5. Vacuolar changes at the basal layer
6. Show Explaination
23 Patients with chronic idiopathic urticaria should 1. Latex products E
9 avoid: 2. Alcohol
3. Nickel sulfate
4. �-blockers
5. aspirin
6. Show Explaination
24 Ulcerative colitis and crohn’s disease are most 1. Pyoderma gangrenosum C
0 commonly associated with which of the 2. Erythema elevatum diutinum
following: 3. Sweet’s syndrome
4. IgA pemphigus
5. Sneddon-Wilkinson’s disease
6. Show Explaination
24 Which of the following is NOT associated with 1. Diabetes mellitus D
1 this disease of symmetric induration caused 2. Streptococcal infection
mucin deposition? 3. Monoclonal gammopathy
4. Hepatitis C
5. All of the answers are associated with this
disease
6. Show Explaination
24 The main cause of nutritional disease in 1. Unusual diets D
2 developed nations is: 2. Inflammatory bowel disease
3. Malabsorption syndromes
4. Alcoholism
5. Psychiatric illness
6. Show Explaination
24 A 3 month old presents with a diffuse 1. Pseudoparalysis of Parrot A
3 vesiculobullous rash with copper colored 2. Higoumenakis sign
macules on the palms and soles. You ascertain 3. Clutton joints
from the history that his mother had a 4. Mulberry molars
nonpainful erosion on her labia during pregnancy 5. Saddle nose deformity
which spontaneously resolved. What other 6. Show Explaination
symptoms would you expect this infant to have?
24 Which of the following is a cause of immunologic 1. Polymyxin B B
4 urticaria? 2. Amoxicillin
3. Ibuprofen
4. Opiates
5. Tartrazine
6. Show Explaination
24 What characteristic body would be found on 1. Kamino B
5 histopathology of this disease? 2. Civatte
3. Henderson-Patterson
4. Donovan
5. Dutcher
6. Show Explaination
24 The joint most frequently affected in acne 1. Elbow D
6 fulminans is the: 2. Intervertebral
3. Distal interphalangeal joints of the hand
4. Sternoclavicular
5. Sacroiliac
6. Show Explaination
24 A 62 year-old man with chronic renal failure on 1. AA amyloid is likely causative C
7 hemodialysis presents with carpal tunnel 2. AL amyloid is likely causative
syndrome, bone cysts, and spondyloarthropathy. 3. beta 2-microglobulin is likely causative
A diagnosis of amyloidosis is suspected. Which 4. keratin-derived amyloid is likely causative
of the following is true regarding his most likely 5. Amyloid P component will not be found in
diagnosis? affected tissues
6. Show Explaination
24 The Dunnigan variant of partial lipodystrophy is 1. AGPAT2 E
8 caused by a mutation in which gene? 2. Seipin
3. Zinc metalloproteinase
4. Neutrophil elastase
5. LMNA
6. Show Explaination
24 The arthritis of Behcet's disease is 1. Symmetric, erosive polyarthritis C
9 characteristically: 2. Asymmetric, erosive polyarthritis
3. Asymmetric, non-erosive polyarthritis

92
4. Asymmetric, erosive monoarthritis
5. Symmetric, non-erosive polyarthritis
6. Show Explaination
25 A patient recently underwent parotid surgery 1. Facial nerve C
0 and now reports unilateral flushing and sweating 2. Maxillary nerve
around mealtime. The nerve injured in this 3. Mandibular nerve
syndrome is a branch of the 4. Cervical nerve
5. Frontal nerve
6. Show Explaination
25 Monotherapy for acne with topical antibiotics is 1. Slow onset of comedolytic action D
1 discouraged because of: 2. Potential for irritation
3. Lack of anti-inflammatory action
4. Potential for bacterial resistance
5. Poor patient compliance
6. Show Explaination
25 Which of the following statements is true 1. It is often misdiagnosed as cellulitis A
2 regarding Morbihan's Disease? 2. It presents with blepharitis, conjunctivitis,
iritis, and keratitis
3. Histopathology reveals perifollicular and
perivascular noncaseating epithelioid
granulomas
4. It occurs around the mouth and/or nose and
eyes and may be triggered by topical steroid
use
5. It presents with large coalescent nodules and
confluent draining sinuses occupying most of
the face
6. Show Explaination
25 The most common systemic manifestations of 1. Gastrointestinal A
3 systemic sclerosis are: 2. Cardiovascular
3. Pulmonary
4. Renal
5. Neurologic
6. Show Explaination
25 Which organic system is least likely to be 1. Liver C
4 involved in the attached image 2. Spleen
3. Genitourinary
4. Gastrointestinal tract
5. Bone marrow
6. Show Explaination
25 The constellation of erythema nodosum, bilateral 1. Loeffler's syndrome C
5 hilar lymphadenopathy, uveitis, fever, and 2. Schnitzler's syndrome
arthritis in patients with sarcoidosis is known as: 3. Lofgren's syndrome
4. Heerfordt's syndrome
5. Mikulicz's syndrome
6. Show Explaination
25 A 16 year old developmentally normal male 1. Autosomal dominant D
6 presents to his pediatrician intermittent vague 2. Autosomal recessive
epidodes of hand and feet paresthesias and non 3. X-linked dominant
specific episode of GI distress. He is referred to 4. X-linked recessive
you to evaluate numerous punctate to 5 slightly 5. Not an inherited disorder
verrucous, deep-red to blue-black papules 6. Show Explaination
distributed diffusely on his trunk in a bathing suit
distribution. Polarization microscopy of the
sediment of his urine demonstrates birefringent
lipid globules (ie, renal tubular epithelial cells or
cell fragments with lipid inclusions) with the
characteristic Maltese cross configuration. How
is this disorder inherited?
25 Which of the following is not associated with this 1. Poor oral hygeine C
7 condition? 2. Antibiotic use
3. Aging
4. Thrush
5. Smoking
6. Show Explaination
25 Kidney disease in Henoch-Schonlein Purpura 1. Limited skin involvement B
8 may be predicted by: 2. Spread of purpura to the upper trunk
3. Synovial involvement
4. Bullous lesions
5. Facial edema
6. Show Explaination
25 Which of the following is a feature of Sneddon's 1. Venous thromboses A

93
9 Syndrome? 2. Hepatosplenomegaly
3. anti-Scl-70 autoantibodies
4. M. tuberculosis infection
5. C2 deficiency
6. Show Explaination
26 All of the following are true regarding Reiter'™s 1. The classic clinical triad is urethritis, B
0 syndrome except: conjunctivitis and arthritis
2. Usually occurs in young women of HLA-B27
genotype
3. May be associated with keratoderma
belnnorrhagicum
4. May be associated with Chlamydia
trachomatis
5. Nails may become thick and brittle with
heavy subungual hyperkeratotic deposits
6. Show Explaination
26 Kveim-Siltzback test is useful in the diagnosis of: 1. Scarlet fever B
1 2. Sarcoidosis
3. Leprosy
4. Leishmaniasis
5. Chancroid
6. Show Explaination
26 This patient was started on isotretinoin but failed 1. Acne fulminans B
2 to discontinue the tetracycline. He is at risk for 2. Pseudotumor cerebri
what complication? 3. Diarrhea
4. Depression
5. Myositis
6. Show Explaination
26 What bacteria is thought to possibly play a role 1. B. burgdorferi A
3 in the pathogenesis of this disease? 2. H. pylori
3. E. coli
4. S. aureus
5. S. enteritidis
6. Show Explaination
26 All of the following drugs have been reported to 1. Hydroxyurea E
4 cause rash similar to the attached image except 2. D-penicillamine
3. Statins
4. Phenytoin
5. Captopril
6. Show Explaination
26 Which of the following treatments for acne 1. Erythromycin A
5 inhibits RNA-dependent protein synthesis by 2. Tetracycline
binding to the 50s ribosomal subunit? 3. Trimethoprim-sulfamethoxazle
4. Benzoyl peroxide
5. Azeleic acid
6. Show Explaination
26 Which of following medications may increase the 1. Spironolactone E
6 likelihood of pseudotumor cerebri in patients 2. Dapsone
taking oral isotretinoin? 3. Amoxicillin
4. Rifampin
5. Tetracycline
6. Show Explaination
26 A teenager comes into your office requesting 1. FSH and LH D
7 laser hair removal of her significant facial hair. 2. Glucose and hemoglobin A1C
Before you agree to treat her, you order which of 3. Ferritin and TIBC
the following laboratory analyses? 4. Testosterone and DHEA-S
5. TSH and T3
6. Show Explaination
26 The condition shown here may be exacerbated 1. Comedogenic make-up foundation D
8 by the inappropriate use of which of the 2. Sunscreens
following? 3. Erythromycin 2% ointment
4. Topical corticosteroids
5. Mineral oil-containing moisturizers
6. Show Explaination
26 What is the most common extracutaneous 1. Arthralgias E
9 manifestation of Sweet'€™s syndrome? 2. Conjunctivitis
3. Renal involvement
4. Sterile osteomyelitis
5. Fever
6. Show Explaination
27 A potentially dangerous side effect of 1. Hypokalemia B

94
0 spironolactone is: 2. Hyperkalemia
3. Hypercalcemia
4. Hyponatremia
5. Hypernatremia
6. Show Explaination
27 Which of the following receptor is the most 1. RAR-gamma A
1 important mediator of retinoid activity in the 2. RXR-alpha
skin? 3. RXR-gamma
4. RAR-beta
5. RXR-beta
6. Show Explaination
27 A young man treated with minocycline 1. Increased melanin at the basal layer of the C
2 developed blue-black discoloration with acne epidermis
scars at his cheeks. A Perls stain would show: 2. Black staining granules within macrophages
3. Blue staining granules within macrophages
4. �Muddy brown� pigment granules
5. Giant melanosomes
6. Show Explaination
27 Efficacy of oral contraceptives is reduced by co- 1. Minocycline C
3 administration with which of the following drugs? 2. Doxycycline
3. Rifampin
4. TMP-SMX
5. All of these answers are correct.
6. Show Explaination
27 The most common location of 1. Trunk A
4 dermatofibrosarcoma protuberans is: 2. Head and neck
3. Extremities
4. None of these answers are correct
5. head and neck and extremities
6. Show Explaination
27 Which of the following is associated with 1. Ulcerative colitis D
5 Reiter�s syndrome: 2. Multiple sclerosis
3. HLA-Cw6
4. Asymmetric arthritis
5. Perioral dermatitis
6. Show Explaination
27 A 28 year old previously healthy man presents 1. Females and males are equally affected B
6 with thick crusting/hyperkeratosis of the palms 2. A chronic deforming arthritis occurs in 20%
and soles, urethritis, and one month of 3. TNF-alpha inhibiting agents have no role in
peripheral arthritis. What of the following is true the treatment of this condition
regarding his condition 4. Patients must have urethritis, conjunctivitis,
and arthritis for diagnosis
5. Clamydia cervicitis is not associated with this
condition
6. Show Explaination
27 Almost all patients with SLE have positive ANAs. 1. ssDNA A
7 A patient can have ANA-negative SLE if they only 2. Sm
make antibodies to: 3. U1RNP
4. Ro
5. dsDNA
6. Show Explaination
27 The most common associated malignancy with 1. Colon carcinoma E
8 dermatomyositis is: 2. Hepatocellular carcinoma
3. Renal cell carcinoma
4. Esophageal carcinoma
5. Ovarian carcinoma
6. Show Explaination
27 Which of the following is a function of 1. Downregulation of TLR-2 expression C
9 Propionibacterium acnes (P. acnes) in 2. Inhibition of complement
contributing to the pathogenesis of acne? 3. Activation of TLR-2
4. Inhibition of Il-1
5. Reduction in free fatty acid levels
6. Show Explaination
28 Which of the following are risk factor(s) for post- 1. Renal transplant E
0 transplant CTCL? 2. Liver transplant
3. Cyclosporine therapy
4. Female Sex
5. Renal transplant and Cyclosporine therapy
6. Show Explaination
28 Which of the following medications is NOT 1. Lithium E
1 associated with exacerbating or causing this 2. Prednisone
eruption? 3. Phenytoin

95
4. Isoniazid
5. Propranolol
6. Show Explaination
28 A 32 year-old woman presents with moderate 1. Stein-Leventhal syndrome C
2 hirsutism. She has normal menses, normal-sized 2. Cushing syndrome
ovaries, no evidence of tumors of the adrenal or 3. Idiopathic hirsutism
ovary, and normal adrenal function, but does 4. Occult virilizing tumor
have slight elevations of plasma 5. Kruckenberg tumor
androstenedione and testosterone. What is the 6. Show Explaination
most likely diagnosis?
28 A mother brings her 4 year old boy with silvery 1. Biotin has shown improvement in a few cases D
3 blond hair into your office and reports that her 2. There are usually no systemic findings
child's hair cannot be combed flat. You notice 3. Longitudinal grooving can be seen on
that it has a spun-glass appearance. Which of electron microscopy
the following is false regarding this condition? 4. The hair abnormality tends to persist with
age
5. The condition is autosomal dominant
6. Show Explaination
28 Presence of which of the following 1. anti-U1RNP E
4 autoantibodies is diagnostic of SLE and not 2. anti-dsDNA
reported in patients with other connective tissue 3. anti-Ro
diseases? 4. anti-La
5. anti-Sm
6. Show Explaination
28 First line treatment of chromoblastomycosis 1. Inhibits squalene epoxidase B
5 includes surgery and a medication with which 2. Inhibits 14-alpha-demethylase
mechanism of action? 3. Disrupts microtubule mitotic spindle
formation
4. Inhibits sythesis of beta-1,3-diglucan and
disrupts cell walls
5. Inhibits fungal cytochrome P-450 mediated
14 alpha-lanosterol demethylation
6. Show Explaination
28 Acquired angioedema is characterized by: 1. Inheritance D
6 2. Decreased levels of C1
3. Self-limited course
4. Association with underlying malignancy
5. Increased C1 esterase inhibitor
6. Show Explaination
28 This patient has similar lesions on his distal 1. Hemogram B
7 extremities. Which laboratory test can be done 2. Alkaline phosphatase
in order to make a diagnosis? 3. Creatinine
4. Potassium
5. TSH
6. Show Explaination
28 Which form of sarcoidosis is associated with 1. Lofgren's syndrome E
8 camptodactyly? 2. Heerfordt's syndrome
3. Darier-Roussy
4. Mikulicz syndrome
5. Blau syndrome
6. Show Explaination
28 Which HLA is associated with psoriatic arthritis in 1. HLA-B17 E
9 disequilibrium, especially if spondylitis is 2. HLA-B13
present? 3. HLA-Bw57
4. HLA-Cw6
5. HLA-B27
6. Show Explaination
29 What is the most common site of extracutaneous 1. Lungs A
0 involvement in this non-infectious disease? 2. Eyes
3. Bone Marrow
4. Liver
5. Nasal mucosa
6. Show Explaination
29 What surgical dressing would you use if you 1. Alginates D
1 wished to debride a wound, reduce pain, and 2. Foams
provide a cooling effect? 3. Hydrocolloid
4. Hydrogel
5. Films
6. Show Explaination
29 Mutations in the MEFV gene product, pyrin, 1. Familial Mediterranean Fever A
2 produce an autosomal recessive syndrome 2. PAPA syndrome
characterized by recurrent fevers, peritonitis, 3. TNF receptor associated periodic syndrome

96
pleuritis, arthritis and erysipelas-like erythema. 4. Hyper IgD syndrome
Which of the following syndromes is described 5. Familial cold autoinflammatory syndrome
above? 6. Show Explaination
29 Pernio or Chilblains is an inflammatory skin 1. Nifedipine A
3 condition which is triggered by cold, wet, non- 2. Nicotinamide
freezing environmental conditions. Acral skin has 3. Phenoxybenzamine
violaceous discoloration accompanied by 4. Psoralen+UVA
burning or itching. While avoidance and 5. Aspirin
prevention is best, the most effective 6. Show Explaination
pharmacologic treatment is:
29 You suspect that a patient has acquired 1. C1q, C3 C
4 angioedema. Levels of which of the following 2. C1q, bradykinin
would you expect to be low? 3. C4, C1q
4. bradykinin, C3
5. C4, C3
6. Show Explaination
29 A 35 year old man has a history of intensely 1. Non-Hodgkin lymphoma A
5 pruritic papules and vesicles on the extensor 2. Colon cancer
surfaces of his lower extremities. Antibodies to 3. Lung cancer
epidermal transglutaminase are detected. What 4. CLL
is this patient at risk for developing? 5. Esophageal cancer
6. Show Explaination
29 Which of the following is true regarding this 1. The subcutaneous variant is most common in A
6 entity? children
2. 75% of localized lesions are still present 2
years after diagnosis
3. Localized lesions often ulcerate and heal with
scarring
4. The perforating variant classically involves
the lower extremities
5. Diabetes is present in the majority of
patients with the generalized variant
6. Show Explaination
29 What type of collagen is overrepresents in this 1. Collagen I B
7 lesion? 2. Collagen III
3. Collagen IV
4. Collagen V
5. Collagen VII
6. Show Explaination
29 Twenty-nail dystrophy, nail plate splitting and 1. Lichen Planus A
8 pterygium formation are nail changes seen in: 2. Darier's disease
3. Psoriasis
4. Scleroderma
5. Dermatomyositis
6. Show Explaination
29 A patient develops pulmonary fibrosis, 1. Bleomycin A
9 Raynaud's phenomenon and skin changes 2. Penicillamine
similar to systemic sclerosis. These changes 3. Hydralazine
resolve following discontinuation of which of the 4. Minocycline
following medications? 5. Isoniazide
6. Show Explaination
30 The primary component of human sebum is: 1. Triglycerides A
0 2. Free fatty acids
3. Ceramides
4. Cholesterol
5. Squalene
6. Show Explaination
30 The most common non-specific cutaneous 1. Leukocytoclastic vasculitis D
1 manifestation associated with this disease is: 2. Ertythema multiforme
3. Acne vulgaris
4. Erythema nodosum
5. Lichen planus
6. Show Explaination
30 What is the best muscle to biopsy in 1. triceps A
2 dermatomyositis? 2. biceps
3. quadriceps
4. gluteus maximus
5. deltoid
6. Show Explaination
30 Which allergen is the most likely cause of this 1. Mercaptobenzothiazole D
3 eyelid dermatitis? 2. Ethyleneurea melamine formaldehyde
3. Ethylenediamine dihydrochloride

97
4. Tosylonamide formaldehyde resin
5. Benzalkonium chloride
6. Show Explaination
30 Which of the following is most associated with 1. Purpura annularis telangiectodes E
4 pruritus? 2. Gougerot–Blum syndrome (pigmented
purpuric lichenoid dermatitis)
3. Lichen aureus
4. Schamberg\'s disease
5. Ducas and Kapetanakis pigmented purpura
6. Show Explaination
30 The protein component of nodular amyloidosis 1. SAA protein B
5 is: 2. AL protein
3. Keratin
4. Collagen
5. Bp180
6. Show Explaination
30 What is the most common presentation of 1. Asymmetric oligo- or polyarthritis A
6 psoriatic arthritis? 2. Symmetric polyarthritis
3. Spondylitis (axial)
4. Distal interphalangeal joint (DIP) disease
5. Arthritis mutilans
6. Show Explaination
30 Which of the following is NOT a feature 1. Accelerated blanch response A
7 associated with this condition? 2. White dermatographism
3. Anterior subcapsular cataracts
4. Keratoconus
5. Pityriasis alba
6. Show Explaination
30 Which enzyme can be a useful adjunct test to 1. Zinc sulfatase B
8 diagnose zinc deficiency where the serum zinc 2. Alkaline phosphatase
level is normal or near normal? 3. AST
4. ALT
5. Creatinine kinase
6. Show Explaination
30 The best diagnosis for this congenital 1. Hypomelanosis of Ito E
9 melanopenic lesion without extracutaneous 2. Segmental vitiligo
associations is 3. Ash leaf spots
4. Nevus anemicus
5. Nevus achromicus
6. Show Explaination
31 A patient presents with cutaneous lesions 1. Sensitivity: 60 Specificity: 80 A
0 suggestive of sarcoidosis. You consider ordering 2. Sensitivity: 90 Specificity: 60
a serum ACE level to help with the diagnosis. 3. Sensitivity: 95 Specificity: 80
What is the sensitivity and specificity of 4. Sensitivity: 60 Specificity: 60
checking an ace level in this pt? 5. Sensitivity: 90 Specificity: 25
6. Show Explaination
31 Which of the following HLA types is associated 1. HLA-B13 B
1 with psoriasis and predicts earlier onset and 2. HLA-B17
more severe disease? 3. HLA-Bw57
4. HLA-Cw6
5. HLA-B27
6. Show Explaination
31 Which HLA type is more commonly associated 1. HLA-B7 C
2 with this clinical finding? 2. HLA-B15
3. HLA-B27
4. HLA-B51
5. HLA-DR4
6. Show Explaination
31 Which of the following HLA types is associated 1. HLA-B13 or HLA-B17 C
3 with early onset psoriasis? 2. HLA-B27
3. HLA-B57, HLA-Cw6, or HLA-DR7
4. HLA-Cw2
5. HLA-DR3
6. Show Explaination
31 Nekam's Disease: 1. Is generally responsive to topical and D
4 intralesional steroids
2. Characteristically lacks scale
3. Rarely involves the buttocks
4. Presents with a reticulate pattern on the
dorsal hands and feet
5. Presents with hypopigmented, atrophic

98
lesions on the extremities
6. Show Explaination
31 TH2 immune responses: 1. Are associated with cell-mediated immunity B
5 2. Produce IL-6
3. Produce IFN-gamma
4. Produce TNF-beta
5. Produce IL-2
6. Show Explaination
31 Phrynoderma can be seen in all of the following 1. Vitamin A D
6 nutritional deficiency except 2. Vitamin B
3. Vitamin C
4. Vitamin D
5. Vitamin E
6. Show Explaination
31 What is the eponym used for this inherited, 1. Meige disease B
7 congenital condition? 2. Milroy disease
3. Proteus syndrome
4. Klippel-Trenaunay syndrome
5. Stuart-Treves syndrome
6. Show Explaination
31 Which of the following is correct about 1. painful C
8 eosinophilic folliculitis? 2. only seen in adults
3. classified as an AIDS-defining illness
4. more common in females
5. associated with P. acnes infection of hair
follicles
6. Show Explaination
31 Macroglossia can present in all of the following 1. Primary systemic amyloidosis E
9 disorders except 2. Down's syndrome
3. Mucopolysaccharidoses
4. Cretinism
5. Behchets disease
6. Show Explaination

99
6. Bengin & Malignant Tumor
# Question Answers An
s
1 Which of the following is true regarding digital HPV- 1. The rate of metastasis approaches 15%. C
associated squamous cell cancers? 2. HPV18 is the most common associated sybtype.
3. Mohs micrographic surgery yields a 20% recurrence
rate.
4. Women outnumber men 2:1.
5. These lesions only occur in association with
immunosuppression.
2 What virus is most closely associated with these 1. Human herpes virus 2 C
lesions in this HIV infected patient? 2. Human herpes virus 6
3. Human herpes virus 8
4. Cytomegalovirus
5. Ebstein Barr virus
3 All of the following are true regarding lentigo 1. High rates of recurrence E
maligna except: 2. Occurs mostly on head and neck
3. Mostly in sun-exposed areas
4. Margins difficult to evaluate
5. Spares oral mucosa
4 All of the followings can be used for treatemt of this 1. Cryotherapy E
condition except 2. Topical Imiquimod 5% cream
3. Topical 5-flurouracil
4. Topical retinoids
5. Surgical excision
5 The most common location of superficial spreading 1. Lower legs B
melanoma in men is: 2. Back
3. Upper extremities
4. Head and neck
5. Digits
6 On histologic examination of the adenocarcinoma of 1. PAS E
the perineal area, which of the following stains 2. Mucicarmine
would NOT be positive? 3. CEA
4. EMA
5. HMB45
7 Which is the most common neoplasm in patients 1. Basal cell carcinoma C
who have had long-term PUVA therapy? 2. Atypical fibroxanthoma
3. Squamous cell carcinoma
4. Cutaneous T cell lymphoma
5. Melanoma
8 Which of the following are true for the sign of Leser- 1. Lesions are commonly located on the chest and back E
Trelat? 2. Classically described to appear in a �Christmas tree�
pattern
3. Usually associated with internal malignancies
4. Eruptive nature
5. All of these answers are correct
9 All of the following statements are true regarding 1. They occur more commonly in Caucasians than in non- C
angiosarcomas EXCEPT: Caucasians
2. Men are more often affected than women
3. They are rarely symptomatic
4. Radiation is usually employed after surgical excision
5. Cervical lymph nodes are a common site of
metastases
10 Merkel cell carcinoma stains positively for: 1. Leukocyte common antigen B
2. Neuron specific enolase
3. Vimentin
4. S-100
5. None of these answers are correct
11 The following lesion is the classic presentation of: 1. BCC E
2. Merkel cell carcinoma
3. Melanoma
4. CTCL
5. Keratoacanthoma
12 Which of the following are the most reliable 1. Breslow�s depth and ulceration A
prognostic factors in malignant melanoma? 2. Breslow�s depth and Clark level
3. Clark level and ulceration
4. Breslow�s depth and Clark level
5. None of these answers are correct
13 The mucin found in this lesion is produced by: 1. Fibroblasts A
2. Keratinocyte
3. Merkel cells
4. Nail matrix
5. Osteoblast
14 The incidence of cutaneous squamous cell 1. 2 fold E
carcinoma in organ transplant recipients is 2. 5 fold
increased by how much compared with the general 3. 10 fold
population? 4. 20 fold

100
5. 65 fold
15 An elderly man presents to your office with a telangiectatic, violaceous 1. Mohs micrographic surgery is D
1cm dome-shaped nodule on the neck. Biopsy reveals large, solid nests of contraindicated in treatment of this
cells of intermediate size, with a trabecular pattern at the periphery. These lesion
cells involve the dermis and spread into the subcutaneous fat, but spare 2. Vimentin and desmin stains may
the overlying epidermis. The cells are round and uniform in size, with a be positive
round to oval nucleus, small nucleoli, and evenly dispersed chromatin. 3. S-100 stains should be positive
Numerous mitotic figures and necrotic areas are seen. Neuron specific 4. This lesion may contain ACTH
enolase is positive. Which of the following is true regarding this patient's 5. The neoplasm should not
diagnosis? contain gastrin
16 In which of the following ethnic groups is squamous 1. Asian Indians A
cell carcinoma the most common type of skin 2. Caucasians
cancer? 3. Hispanics
4. Japanese
5. Chinese
17 The treatment of choice for this lesion shown is: 1. Radiation therapy E
2. Imiquimod
3. 5 Flourouracil
4. Wide excision with 2cm margins
5. Mohs surgery
18 All of the following may increase the incidence of 1. Sun exposure E
SCC, EXCEPT: 2. Immunosuppression
3. Increasing age
4. Proximity to the equator
5. All of the following may increase the incidence of SCC
6.
19 Which of the following is a clinical stage of 1. Plaque E
keratoacanthomas? 2. Mature
3. Proliferative
4. Resolving
5. All of these answers are correct except plaque
20 What is the most common location for pagetoid 1. Trunk C
reticulosis? 2. Head and neck
3. Hands and feet
4. Flexural sites of upper and lower extremities
5. Genitals
21 A 30 year-old gentleman presents with multiple self 1. Epidermodysplasia verruciformis B
healing lesions similar to the one pictured. Other 2. Ferguson-Smith syndrome
family families also have the same disease. What is 3. Nevoid basal cell carcinoma syndrome
the most likely diagnosis? 4. Dyskeratosis congenita
5. Basex syndrome
22 Which of the following is not considered a high-risk 1. Eyelids E
location of SCC: 2. Nose
3. Ear
4. Lips
5. Chest
23 40 year-old male presented with a slow growing 1. Imatinib A
large tumor on the back. The patient undergoes 2. Cytarabine
wide excision with adjunctive chemotherapy. Which 3. Doxorubicin
of the chemotherapeutic agents was used? 4. Vincristine
5. Cisplatin
24 Which of the following are features of 1. Commonly located on the extremities E
dermatofibromas, which help differentiate it from 2. Factor XIIIa negative
dermatofibrosarcoma protuberans? 3. Factor XIIIa positive
4. Commonly located on the trunk
5. Commonly located on the extremities and factor XIIIa
positive
25 The risk of a melanoma developing in a giant 1. 3% B
congenital melanocytic nevus is approximately: 2. 6%
3. 12%
4. 50%
5. Virtually all of these patients will develop melanoma
26 A patient with multiple skin nodules has a biopsy 1. Polymerase chain reaction A
suggestive of cutaneous lymphoma. The next 2. Complete blood count
appropriate step in making the diagnosis is: 3. Flow cytometry
4. Immunohistochemical stains
5. Chest xray
27 An elderly white man presents with a slowly enlarging, well-demarcated 1. Conventional excision E
pink, scaly plaque on the neck. Biopsy of the lesion reveals epidermal 2. Mohs micrographic surgery
dysplasia and keratinocytic disorganization with preservation of the 3. Imiquimod
basement membrane. Hyperkeratosis and parakeratosis are also present, 4. Conventional excision and Mohs
and numerous atypical keratinocytes are seen throughout the epidermis, micrographic surgery
with loss of polarity, atypia, and mitoses. A chronic inflammatory 5. All of these answers are correct
infiltrate is present in the upper dermis. Which of the following
treatment(s) should be considered for this patient?
28 Intermittent sun exposure with painful sunburns is a 1. Atypical nevi D
predisposing factor for the development of: 2. Seborrheic keratosis
3. Malignant melanoma
4. Atypical nevi and Malignant melanoma
5. All of these answers are correct
29 The cure rate of cryotherapy as a treatment for 1. 80% E
actinic keratoses is: 2. 85%
3. 90%
4. 95%
5. 99%

101
30 Compared with the general population, what is the 1. 4 fold A
overall risk of developing cutaneous and systemic 2. 10 fold
malignancies in organ transplant recipient? 3. 25 fold
4. 50 fold
5. 100 fold
31 What would be the characteristic histopathologic 1. Full thickness keratinocytic atypia B
findings of this lesion? 2. Cornoid lamella
3. Pale staining cells
4. Horn pseudocysts
5. Wedge shaped granular layer with lichenoid infiltrate
32 The most common location for primary mucinous 1. neck B
carcinoma is: 2. eyelid
3. areola
4. scrotum
5. nose
33 Commonly used immunohistochemical markers that 1. Chromogranin A/B C
are positive in Merkel cell carcinomas include all of 2. Synaptophysin
the following EXCEPT: 3. Glial fibrillary acidic protein
4. Cytokeratin 20
5. Cytokeratin 8
34 The Pinkus BCC is frequently located on the: 1. Digits E
2. Eyelid
3. Lips
4. Ear
5. Lumbosacral area
35 The most important mutated gene associated with a 1. PTCH B
predisposition to develop malignant melanoma is: 2. CDKN2A
3. PTCH2
4. None of these answers are correct
5. All of these answers are correct
36 What temperature must be achieved for adequate 1. -10 degrees Celcius E
treatment of a small superficial squamous cell 2. -20 degrees Celcius
carcinoma with cryotherapy? 3. -30 degrees Celcius
4. -40 degrees Celcius
5. -50 degrees Celcius
37 Denileukin diftitox (ONTAK�): 1. is a systemic treatment option for CTCL E
2. is a diphtheria fusion toxin
3. targets the interleukin-2 receptor
4. None of these answers are correct
5. All of these answers are correct
38 The Grzybowski type of keratoacanthoma: 1. Is characterized by rapid growth of a single lesion E
reaching a diameter of 9 cm or more
2. Typically invades underlying cartilage
3. Demonstrates simultaneous central healing
4. Presents in childhood on sun-exposed surfaces
5. Presents with hundreds of disseminated lesions
39 Desmoplastic trichoepithelioma: 1. Is most common in middle-aged patients D
2. More commonly appears in males than in females
3. Does not display foci of calcification or ossification
4. Presents as a well-circumscribed lesions located in the
upper dermis
5. None of these answers are correct
40 Seventy-five year old, male patient, with a 3cms 1. Radiation D
pink, pearly nodule on his chest. You notice that the 2. Conventional excision
patient has a scar on the same area. The treatment 3. Cryotherapy
of choice is: 4. Mohs surgery
5. Imiquimod
41 What is the eponym for this metastasis to the 1. Tripe palm E
umbilicus? 2. Sign of Leser-Trelat
3. Pityriasis rotunda
4. Trousseau syndrome
5. Sister Mary Joseph Nodule
42 Which of the following is the most common initial 1. Lungs B
site of metastasis from a primary BCC? 2. Regional lymph nodes
3. Bone
4. Liver
5. Pleura
43 A young woman presents with a single small, firm, 1. Reassurance and no further treatment E
umbilicated papule on the face. Biopsy reveals a 2. Close clinical follow-up
well-circumscribed lesion located in the upper 3. Topical 5-fluorouracil
dermis. Strands of basaloid cells are seen 4. Cryotherapy
surrounded by fibrotic or desmoplastic stroma. Horn 5. Local surgical excision
cysts and foci of sebaceous cells and calcification
are also noted. How should this patient and her
lesion be treated?
44 Which of the following is an immunhistochemical 1. S-100 D
marker for Merkel Cell Carcinoma?: 2. Vimentin
3. HMB-45
4. Neuron specific enolase
5. All of these answers are correct
45 Which of the following immunologic drugs has been 1. Etanercept D
shown to increase survival of patients with stage III 2. Adalimumab
and stage IV melanoma? 3. Ustekinumab
4. Ipilimumab
5. Infliximab
46 What is the most location for this lesion which lacks 1. Back C

102
phosphorylase in the epidermal cells? 2. Buttock
3. Leg
4. Neck
5. Sacrum
47 A 50 year-old female patient presents with a reddish indurated 1. Microcystic adnexal carcinoma C
plaque located on her right clavicular area (see image). The 2. Seborrheic keratosis
biopsy shows the presence of monomorphic spindle cells in a 3. Dermatofibrosarcoma protuberans
storiform pattern, and deep subcutaneous infiltration. According 4. Merkel cell carcinoma
to the aforementioned information, the most likely diagnosis is: 5. None of these answers are correct
48 The most common site for intra oral melanoma is? 1. buccal C
2. soft palate
3. hard palate
4. gingiva
5. uvula
49 Which of the following is true regarding actinic 1. Salicylic acid, tretinoin, and alpha-hydroxy acids are B
keratoses? not useful in treatment
2. Low fat diets may decrease the incidence of AKs
3. Cure rates for treatment with topical 5-fluorouracil are
higher than for treatment with cryotherapy
4. UVA from sunlight is most responsible for AK
development
5. Male gender is not a risk factor for AKs.
50 What is the mean time-frame for development of 1. Less than 1 year E
Stewart-Treves Syndrome? 2. 1 year
3. 5 years
4. 10 years
5. >20 years
51 Histologically, this lesion is shows plump, polygonal 1. Colloidal iron D
cells arranged in nests and fascicles with granular 2. Von kossa
cytoplasm. Which immunohistochemical stain would 3. Warthin-starry
be positive? 4. S-100
5. CD68
52 What is the most common location of oral SCC? 1. Soft palate E
2. Buccal mucosa
3. Gingiva
4. Dorsal tongue
5. Lateral tongue
53 Which phase of the cell cycle does p53 regulate? 1. G1 A
2. G2
3. S phase
4. Mitosis
5. Meiosis
54 All of the following statements regarding Bowen's 1. Lesions arising on the lower limbs are more common in A
disease are true EXCEPT: men than in women
2. The basement membrane remains intact on
histopathology
3. 5% of patients with Bowen's disease develop invasive
squamous cell carcinoma
4. Chronic sun exposure is a risk factor for Bowen's
disease
5. Mucosal Bowen's disease may appear as a verrucous
plaque
55 A 55 year-old female presents with an ulcerated 1. Stage IB B
malignant melanoma with Breslow dept of 1.5mm. 2. Stage IIA
Sentinel lymph node biopsy is negative. Which of 3. Stage IIB
the following is the correct staging classification? 4. Stage IIIA
5. Stage IIIB
56 Dermoscopic features suggestive of malignant 1. Presence of 2 or more colors within the lesion E
melanoma include: 2. Blue-whitish veil
3. Asymmetric radial streaming
4. Abrupt interruption of pigment network in the
periphery
5. All of these answers are correct
57 Which substance does p53 normally activate to 1. p21 C
promote apoptosis via inhibition of bcl-2? 2. p16
3. Puma
4. Mdm2
5. Akt
58 A 65 year-old female with multiple actinic keratosis 1. The patient must immediately stop treatment since B
on the face under treatment with 5-FU. According to unexpected side effects have developed
the image and aforementioned information, you 2. The patient has been compliant with 5-FU treatment
may conclude that: and the appearance of inflammation, erythema and erosions
are expected
3. The image is not relevant to 5-FU treatment
4. None of these answers are correct
5. All of these answers are correct
59 Which of the following features of thin melanomas 1. Regression A
(<1 mm thick) has been associated with an 2. Location
increased risk of metastasis? 3. Size of lesion
4. Gender
5. P53 expression
60 The risk of metastasis from SCC increases with: 1. Tumor size E
2. Depth of invasion
3. Degree of differentiation
4. Immunosupression
5. All of these answers are correct

103
61 Which of the following is a common location of 1. Chest B
melanoma in women? 2. Lower legs
3. Genitals
4. Digits
5. Scalp
62 Sentinel lymph node biopsy in malignant melanoma: 1. Has gained acceptance for the treatment of MM of E
intermediate thickness (1-4mm)
2. Is mainly being used for a more accurate staging
3. Is not recommended
4. None of these answers are correct
5. Has gained acceptance for the treatment of MM of
intermediate thickness (1-4mm) and is mainly being used for a
more accurate staging
63 The keratoacanthoma variant characterized by the 1. Gorlin syndrome C
sudden appearance during childhood or adolescence 2. Xeroderma pigmentosa
of multiple KAs is called: 3. Ferguson-Smith
4. Grzybowski
5. None of these answers are correct
64 A 40 year-old female patient presents with the 1. Microcystic adnexal carcinoma A
following lesion (see image). The biopsy report 2. Seborrheic keratosis
shows duct like structures, tadpole structures within 3. Dermatofibrosarcoma protuberans
a fibrotic stroma. According to the aforementioned 4. Merkel cell carcinoma
information, the most likely diagnosis is: 5. None of these answers are correct
65 Mutations of the p53 gene has been associated with 1. Mmelanoma C
the development of: 2. Kaposi�s sarcoma
3. Actinic keratosis
4. Molluscum contagiosum
5. All of these answers are correct
66 Which test should be used to detect monoclonal 1. Northern blot B
gene rearrangements in cutaneous T-cell 2. Southern blot
lymphoma? 3. Western blot
4. ELISA
5. Electrophoresis
67 Which of the following is most likely to present with 1. Lung cancer D
cutaneous metastases in men? 2. Colon cancer
3. Prostate cancer
4. Melanoma
5. Esophageal Cancer
68 When performing a biopsy of a suspected 1. Fusiform incision through the entire KA may be E
keratoacanthoma: performed
2. It is necessary to biopsy down to subcutaneous fat
3. A full-thickness shave biopsy is acceptable
4. A complete excisional biopsy may be performed
5. All of these answers are correct
69 At what location is this tumor LEAST likely to recur 1. Nasolabial fold C
after surgical excision? 2. Lateral canthus
3. Mid forehead
4. Preauricular
5. Scalp
70 Knowing that the patient in this picture has a biopsy 1. Conventional excision D
confirmed BCC, which of the following would be the 2. Imiquimod
treatment of choice: 3. Radiation
4. Mohs micrographic surgery
5. Cryosurgery
6.
71 Which of the following melanoma subtypes is more 1. Acral lentiginous melanoma A
consistently seen in dark-skinned individuals? 2. Superficial spreading melanoma
3. Lentigo maligna melanoma
4. Nodular melanoma
5. Superficial spreading melanoma and Lentigo maligna
melanoma
72 Which one of the following malignancies is 1. Verrucous carcinoma A
associated with HPV infection? 2. Metastatic melanoma
3. Basal cell carcinoma
4. Sebaceous carcinoma
5. Atypical fibroxanthoma
73 Which of the following is commony seen in 1. Maroon lagoons D
seborrheic keratoses when examined with 2. Network
dermoscopy? 3. Pigment globules
4. Milia like cysts
5. Blue-gray veil
74 Which one of the following agents has demonstrated 1. Prostaglandin E2 D
potential benefit as a chemopreventive to UV- 2. Vitamin D
induced skin cancer? 3. Arachidonic acid
4. Celecoxib
5. Vitamin E
75 The most common location for angiosarcoma is: 1. Legs C
2. Arms
3. Head and neck
4. Trunk
5. Digits
76 A 56-year old woman with a history significant for 1. Angiosarcoma A
chronic lymphedema after radical mastectomy 2. Bacillary angiomatosis
twelve years ago presents with this growth on her 3. Castleman's syndrome
arm. What is the diagnosis? 4. Kaposi's sarcoma
5. Metastatic breast carcinoma

104
77 Which of the following melanoma scenarios have 1. Twenty-one year old female with primary lesion located A
the best prognosis? on the right lower leg
2. Twenty-one year old male with primary lesion located
on the chest
3. Thirty-six year old male with primary lesion located on
the back
4. Twenty-one year old female with primary, ulcerated
lesion located on the right lower leg
5. Thirty-six year old male with primary lesion located on
his left lower leg with palpable inguinal lymph nodes
78 Bazex syndrome can be differentiated clinically from 1. Multiple basal cell carcinomas E
Rombo syndrome by presence of 2. Trichiepitheliomas
3. Milia
4. Bollicular atrophoderma
5. Hypohidrosis
79 If left untreated, which of the following is not at risk 1. Bowenoid papulosis E
for malignant transformation? 2. Cutaneous horn
3. Actinic cheilitis
4. Leukoplakia
5. Stucco keratosis
80 What kind of T-lymphocyte is the most common 1. CD4 A
neoplastic cell in Cutaneous T-Cell Lymphoma: 2. CD8
3. Natural killer
4. None of these answers are correct
5. All of these answers are correct
81 All of the following statements regarding the patient 1. Spina bifida may be an associated finding C
pictured are true EXCEPT: 2. The patient likely has a mutation in the PTCH gene
3. Ameloblastoma is a tumor associated with this disease
4. This patient likely had a similarly affected parent
5. These lesions appeared in childhood
82 A patient has a malignant melanoma 1.6mm thick 1. IIC C
with ulceration and a micrometastasis in 1 node. 2. IIIA
The patient's staging according to the American 3. IIIB
Joint Committee on Cancer Staging System is: 4. IIIC
5. IV
83 What is the most common location for an epitheloid 1. head and neck C
sarcoma? 2. proximal extremities
3. hands and forearms
4. lower legs
5. groin and buttocks
84 Mutations in which gene would likely be found in the 1. PATCH B
neoplastic cells of this lesion? 2. p53
3. Fumarate hydratase
4. CREBBP
5. p63
85 Which of the following is the most important 1. Age B
prognostic indicator in a patient with cutaneous 2. Primary vs secondary cutaneous
lymphoma? 3. Gender
4. Extent of cutaneous involvement
5. Subtype of lymphoma
86 The patient is a 45 year old male complaining of red, 1. Pulsed Dye Laser C
chapped lower lip. Which of the following lasers is 2. Nd:YAG laser
the most appropriate to treat this condition? 3. CO2 laser
4. Diode laser
5. Laser treatment is not an option
87 What is the most common site of metastasis for this 1. Lung A
dermal tumor? 2. Brain
3. Kidney
4. Liver
5. Bone
88 Which of the following statements regarding 1. Increasing age has a positive effect on survival. D
prognosis for malignant melanoma is true? 2. Patients with primary lesions located on the
extremities (except acral lesions) have a worse prognosis than
those with tumors located on the trunk.
3. There is a higher survival rate for patients with
palpable metastatic nodes compared to those with
micrometastatic nodal disease.
4. For stage IV disease, patients with non-visceral
metastases (eg skin, subcutis, distant lymph nodes) have a
better prognosis compared with those with visceral
metastases.
5. Male gender has a positive effect on survival.
89 Which of the following are features of microcystic 1. Deep subcutaneous infiltration E
adnexal carcinoma that help distinguish it from 2. Perineural invasion
desmoplastic trichoepitheliomas? 3. CEA positive staining
4. Commonly located on the face
5. Deep subcutaneous infiltration,perineural invasion,
and CEA positive staining
90 Which of the following neoplasms has demonstrated 1. Mycosis fungoides B
an association with HTLV-1 infection? 2. Adult T cell lymphoma
3. Follicular lymphoma
4. Multiple myeloma
5. Hodgkin�s disease
91 Chloroma is a characteristic cutaneous 1. Tuberous sclerosis D
manifestation of: 2. Sweet�s syndrome
3. Neurofibromatosis

105
4. Leukemia
5. Pseudomonas sepsis
92 What is the diagnosis? 1. Condyloma acuminata C
2. Molloscum contagiosum
3. Pearly penile papules
4. Lichen planus
5. Psoriasis
93 Periungual Squamous cell carcinoma has been 1. 6, 11 C
linked to which HPV type(s)? 2. 2, 4
3. 16
4. 13
5. 8
94 What would you expect to see under dermoscopy of 1. Red sacculae A
this vascular neoplasm? 2. Arborizing blood vessels
3. Hair pin telangectasia
4. Milky red globules
5. Blue-grey ovoid nests
95 Which HPV serotype has been associated with 1. 6 B
periungual SCC? 2. 16
3. 11
4. All of these answers are correct
5. HPV is not associated with SCC
96 The Gorlin syndrome is characterized by: 1. Multiple BCCs during childhood E
2. Macrocephaly
3. Odontogenic keratocysts of the jaw
4. Autosomal-recessive inheritance pattern
5. All of these answers are correct except Autosomal-
recessive inheritance pattern
97 Immunohistochemistry of dermatofibrosarcoma 1. CD34 negative and factor XIIIa positive D
protuberans typically reveals: 2. CD34 negative and factor XIIIa negative
3. CD34 positive and factor XIIIa positive
4. CD34 positive and factor XIIIa negative
5. None of these answers are correct
98 The most common locations of microcystic adnexal 1. Perioral C
carcinoma include all of the following, except: 2. Nasolabial
3. Trunk
4. Periorbital
5. Perioral,nasolabial, and periorbital
99 Maple leaf-like structures seen on dermoscopy are 1. Seborrheic keratoses B
characteristic of which lesion? 2. Pigmented basal cell carcinoma
3. Dermal nevi
4. Melanoma
5. Hemangioma
10 Topical 5-Fluorouracil: 1. Interferes with the synthesis of DNA and RNA D
0 2. Is an alternative for the treatment of actinic keratosis
3. May cause pruritus and burning at the site of
application
4. All of these answers are correct
5. None of these answers are correct
6.
10 UVB induced mutations on the PTCH gene is 1. BCC A
1 associated with the development of: 2. Merkel cell carcinoma
3. Angiosarcoma
4. BCC and Merkel cell carcinoma
5. Merkel cell carcinoma and Angiosarcoma
6.
10 Mohs micrographic surgery the treatment of choice 1. 1 cm SCC located on the chest A
2 for all of the following, except: 2. 2 cms BCC on lower extremities
3. 1 cm BCC on the eyelid
4. Morpheaform BCC on the cheek
5. Recurrent BCC on the chest
10 Histopathological evidence of epidermotropism may 1. Merkel cell carcinoma C
3 be seen in which of the following conditions: 2. SCC
3. CTCL
4. None of these answers are correct
5. All of these answers are correct
10 Currently, the surgical margin for melanomas that 1. 1 mm C
4 measure less than 2 mm in thickness is: 2. 0.5 cm
3. 1 cm
4. 2 cms
5. 3 cms
10 Which of the following immunohistochemistry 1. CD31 B
5 marker is negative in angiosarcomas? 2. CEA
3. Cytokeratin
4. CD34
5. Factor VIII
10 The human papilloma virus type associated with red 1. HPV 1 E
6 brown smooth and warty papules is 2. HPV 5
3. HPV 7
4. HPV 13
5. HPV 16
10 All of the following disorders may manifest as 1. Behchets disease A
7 exfoliative dermatitis except 2. Psoriasis
3. Pemphigus foliaceus
4. Drug reaction

106
5. Sezary syndrome
10 Dermoscopy would be the least helpful in evaluating 1. Pigmented basal cell carcinoma D
8 with lesion? 2. Congenital melanocytic nervus
3. Seborrheic keratosis
4. Amelanotic melanoma
5. Hemangioma
10 This tumor, also called a Shagreen patch, is 1. Neurofibromatosis-1 C
9 characteristic of which of the following 2. Neurofibromatosis-2
genodermatoses? 3. Tuberous sclerosis
4. NAME syndrome
5. Buschke-Ollendorf
11 Keratoacanthomas have been linked etiologically to: 1. Ultraviolet exposure E
0 2. Human papilloma virus
3. Chemical carcinogens such as tar and pitch
4. Smoking
5. All of these answers are correct
11 Amplification of which of the following genes is 1. L-Myc A
1 associated with Merkel cell carcinoma? 2. C-Myc
3. GLI1
4. CDKN2A
5. PTEN
11 What cancer in women most commonly 1. Breast cancer A
2 metastasizes to the skin? 2. Medullary thyroid carcinoma
3. Glioblastoma multiforme
4. Colon adenocarcinoma
5. Cervical cancer
11 Which of the following is true regarding 1. Majority of cases are predominantly CD8(+) C
3 poikilodermatous mycosis fungoides? 2. Patients typically have a later age of onset compared
to classic mycosis fungiodes
3. Can be associated with LyP
4. More women than men affected
5. Patients typically do not respond well to phototherapy
11 All of the following statements regarding Basal Cell 1. Associated tumors include medulloblastoma B
4 Nevus Syndrome are true EXCEPT: 2. It is inherited in an autosomal recessive manner
3. Hypertelorism is a feature
4. The affected gene is mutated in 30-40% of sporadic
basal cell carcinomas
5. All of these answers are correct
11 What is the most common location for this rapidly 1. Head and neck A
5 growing tumor? 2. Chest
3. Back
4. Arms
5. Legs
6.
11 A patient with a innumerable disseminated 1. Is unlikely to have palmoplantar involvement E
6 keratoacanthomas, including lesions on the larynx 2. Likely has an underlying immune deficiency
and oral mucosa: 3. Is at high risk for myelodysplasia
4. Likely inherited their condition in an autosomal
dominant manner
5. Likely developed them during adulthood
6.
11 The most common location of Merkel cell carcinoma 1. Digits E
7 is: 2. Trunk
3. Genitalia
4. Extremities
5. Head and neck
11 This lesion is associated with which of the following: 1. HPV 5 B
8 2. HPV 11
3. HPV 4
4. HPV 7
5. HPV 1
11 Characteristics indicative of a high risk of metastasis 1. Mitotic rate D
9 with SCC include: 2. Extremity location
3. Well differentiated
4. Deep invasion
5. Papillomavirus infection
12 Which of the following would you not expect to see 1. Maple leaf pattern E
0 under dermoscopy? 2. Arborizing blood vessels
3. Blue-grey ovoid nests
4. Orange crust
5. Milky red globules
12 Which of the following ethnic groups are commonly 1. Asians C
1 diagnosed with dermatosis papulosa nigra: 2. Hispanics
3. African-Americans & Hispanic patients
4. Caucasians
5. No difference between ethnic groups
12 A child presents with a giant congenital nevus 1. A head CT B
2 overlying the back of the skull, extending onto the 2. A head MRI
shoulders. It is ~15% body surface area, sparing the 3. A skull plain film X-ray
face and anterior neck. Which test should be 4. A skin biopsy
ordered? 5. A PET scan
12 Which area of the face receives the most cumulative 1. Dorsum of nose C
3 exposure to UV radiation? 2. Orbital region
3. Base of the nose
4. Chin

107
5. Central cheek
12 Desmoplastic trichoepitheliomas are commonly 1. Legs D
4 located on: 2. Chest
3. Back
4. Face
5. Scalp
12 Pseudorosettes in Merkel cell carcinoma: 1. Are seen in the trabecular variant A
5 2. Are seen in the intermediate-cell type
3. Are seen in the small-cell type
4. All of these answers are correct
5. None of these answers are correct
12 Sezary syndrome: 1. Has characteristic Sezary cells in peripheral blood E
6 2. Is the leukemic variant of Mycosis fungoides
3. Is characterized by the triad of pruritic erythroderma,
generalized lymphadenopathy, and presence of Sezary cells in
peripheral blood
4. Is associated with a poor prognosis
5. All of these answers are correct
12 A patient has a 4.5 mm ulcerated melanoma without 1. In situ C
7 regional or distant metastases. According to the new 2. Stage I
AJCC criteria, what melanoma stage does this 3. Stage II
correspond to? 4. Stage III
5. Stage IV
12 Which of the following variants of mycosis fungoides 1. Woringer-Kolopp pagetoid reticulosis B
8 is best diagnosed using a punch biopsy instead of a 2. Syringotropic mycosis fungoides
broad superficial shave biopsy? 3. Ketron-Goodman pagetoid reticulosis
4. Poikilodermatous mycosis fungoides
5. Sezary syndrome
12 Which of the following best describes the incidence 1. BCC>melanoma>SCC>Merkel cell B
9 of skin cancer in transplant recipients from greatest 2. SCC>BCC>melanoma>Merkel cell
to least? 3. BCC>SCC>melanoma>Merkel cell
4. SCC>BCC>Merkel cell>melanoma
5. Merkel cell>SCC>melanoma>BCC
13 Which of the following lesions are �better felt than 1. Seborrheic keratosis C
0 seen�: 2. SCC in situ
3. Actinic keratosis
4. Sebaceous hyperplasia
5. All of these answers are correct
13 Dermatofibrosarcoma protuberans is: 1. Cytogenetically characterized by reciprocal E
1 translocation t(17;22)(q22;q13)
2. Factor XIIIa positive
3. CEA positive
4. CD 34 positive
5. Cytogenetically characterized by reciprocal
translocation t(17;22)(q22;q13) and CD 34 positive
13 A 60 year-old female presents with a well- 1. BCC C
2 demarcated, scaly, erythematous plaque on her 2. SCC
right shin. The biopsy shows full thickness epidermal 3. Bowen's disease
atypia with scattered mitotic figures and overlying 4. Angiosarcoma
parakeratosis. Howerver, the basement membrane 5. CTCL
remains intact. According to the aforementioned
information, the diagnosis is:
13 A 3-year-old girl presents with multiple small, 1. Turner's B
3 angulated, firm nodules. There is a positive "tent 2. Myotonic dystrophy
sign" and biopsy reveals ghost cells and germinative 3. Rubenstein-Taybi
cells. Of the following conditions associated with 4. Sarcoidosis
multiple pilomatricomas, which is thought to be 5. Gardner's syndrome
most closely linked?
13 All of the following are risk factors for metastasis 1. Increasing tumor size E
4 from a primary squamous cell carcinoma EXCEPT: 2. Location on the ear
3. Recurrent tumor
4. Tumor within ulcer
5. All of these answers are correct
13 Sixty-year old, male patient that presents with a 1. Actinic keratosis D
5 �stuck on�, waxy, hyperkeratotic and 2. SCC in situ
hyperpigmented plaque on his back. The most likely 3. Angiosarcoma
diagnosis is: 4. Seborrheic keratosis
5. None of these answers are correct
13 Merkel cell carcinoma has been found to be 1. Herpes virus B
6 associated with which of the following viruses? 2. Polyomavirus
3. Paramyxovirus
4. Flavivirus
5. Enterovirus
13 A patient presents with tender papules with a 1. Gastric carcinoma C
7 pseudo-Darier's sign. She has other family members 2. Ovarian carcinoma
with the same condition. Screening should be 3. Renal cancer
performed to rule out which malignancy? 4. Testicular cancer
5. Lung cancer
13 What is the most likely diagnosis? 1. Acquired digital fibrokeratoma A
8 2. Supernumery digit
3. Verruca
4. Infantile digital fibroma
5. Acrochordon
13 Which of the following immunohistochemical 1. Chromogranin C
9 markers may be indicative of metastatic potential 2. Synaptophysin
for merkel cell carcinoma? 3. CD44

108
4. CK20
5. TTF-1
14 All of the following are true regarding curettage of 1. Treatment remains a controversial topic C
0 giant congenital melanocytic nevi except: 2. Best performed during the first 2 weeks of life
3. Decreases the risk of melanoma
4. Offers an adequate alternative to surgical excision
5. Mandates careful long-term follow-up
14 A patient having Mohs surgery for a squamous cell 1. T0 E
1 carcinoma on the ear has tumor invading the 2. T1
cartilage. What stage disease does the patient 3. T2
have? 4. T3
5. T4
14 All of the following cytokines have demonstrated 1. IFN-alpha E
2 therapeutic benefit in the treatment of melanoma 2. IL-2
EXCEPT: 3. TNF-alpha
4. IL-10
5. GM-CSF
14 All of the following are true of thick melanomas (>3 1. Predominantly nodular type B
3 mm) except: 2. Women affected more than men
3. Predilection for the head and neck
4. Mainly in older patients (>50 years)
5. Associated with fewer nevi
14 All of the following are true of melanoma-associated 1. Lesions resemble vitiligo B
4 leukoderma except: 2. Portends a worse prognosis
3. Seen in patients with metastatic disease but no
primary lesion
4. Histology resembles that of a halo nevus
5. Lesions develop distant to melanoma
14 Which of the following markers do not stain 1. Neuron specific enolase A
5 melanocytic lesions: 2. Vimentin
3. S-100
4. HMB-45
5. All of these answers are correct
14 Merkel cell carcinoma should be treated with what 1. 2mm D
6 size surgical margins? 2. 5mm
3. 1cm
4. 3cm
5. 5cm
14 Imiquimod is an immune response modifier that 1. IL-1 B
7 stimulates innate and cell mediated immune 2. IL-4
pathways. It induces all of the following cytokines 3. IL-5
EXCEPT: 4. IL-6
5. IL-8
14 Which neoplasm is associated with the Stewart- 1. Renal leiomyomas C
8 Treves syndrome? 2. Basal cell carcinoma
3. Angiosarcoma
4. Keratoacanthoma
5. T cell lymphoma

109
7. Medical Mycology
# Question Answers An
s
1 The major endemic area for 1. Africa E
Histoplasmosis 2. Central America
3. Southwest United States
4. South America
5. Eastern United States
6.
2 Nondermatophytes growing on 1. Amber C
Dermatophyte Test Media cause the media 2. Red
to turn what color? 3. Yellow
4. Black
5. Green
6.
3 All of the following parameters are used to 1. Nutritional requirements C
distinguish dermatophytes except: 2. Colony morphology
3. Sucrose hydrolysis
4. Hair fluorescence
5. Growth temperature
6.
4 This organism is often considered a 1. Sepedonium C
contaminant but has been reported to 2. Curvularia
cause onychomycosis: 3. Scopulariopsis
4. Penicillium
5. Phialophora
6.
5 Trichophyton tonsurans sporulates via 1. Arthroconidia C
thick-walled round cells resistant to the 2. Blastoconidia
environment known as: 3. Chlamydoconidia
4. Sporangia
5. Mycelia
6.
6 Trichosporon ovoides is a cause of: 1. Black piedra B
2. White piedra
3. Tinea nigra palmaris
4. Ectothrix tinea capitis
5. Endothrix tinea capitis
6.
7 A patient with white nodules on the hair 1. Corynebacterium D
shaft has a KOH which shows hyphae and minutissimum
arthroconidia. The etiology is: 2. Exophiala wernickii
3. Piedraia hortae
4. Trichosporon beigelii
5. Corynebacterium tenuis
6.
8 A fungal culture demonstrates a suedelike 1. Causes ectothrix infection B
cream-colored colony of teardrop and 2. Requires partial thiamine
balloon-shaped microconidia which for growth
produce a red-brown pigment. Which of 3. Does not have
the following is true of this organism? arthroconidia

110
4. Causes fluorescent hair
infection
5. Is not a cause of tinea
unguim
6.
9 Mycelia can form structures with a comb- 1. Racket forms C
lke appearance called: 2. Favic chandeliers
3. Pectinate bodies
4. Spiral hyphae
5. Nodular bodies
6.
10 Growth of which of the following is not 1. Cryptococcus neoformans D
inhibited on Mycosel media? 2. Yeast forms of Histoplasma
3. Yeast forms of Blastomyces
4. Microsporum gypseum
5. Scytalidium species
6.
11 Which of the following organisms would 1. T. tonsurans C
you expect to recover from a fluorescent 2. M. gypseum
tinea capitis? 3. M. canis and M. gypseum
4. T. violaceum
5. T. rubrum
6.
12 Which of the following fungi can cause an 1. T. tonsurans C
endothrix tinea capitis and favus? 2. M. canis
3. T. schoenleinii
4. M. audouinii
5. T. rubrum
6.
13 All of the following statements about 1. It is most common in male C
paracoccidioidomycosis are true EXCEPT? agricultural workers
2. It has a characteristic
�mariner�s wheel�
appearance on
histopathology
3. Disease is almost always
confined to the skin
4. It is endemic to Brazil
5. It may cause
mucocutaneous lesions
6.
14 Which of the following statements 1. Malassezia species do not B
regarding superficial mycotic infections is fluoresce under a Wood�s
TRUE? lamp
2. Phaeoannellomyces
wernecki causes tinea
nigra
3. Trichophyton beigelii
causes white piedra
4. Trichosporon hortae causes
black piedra
5. Micrococcus sedentarius
causes tinea nigra
6.
15 Which of the following usually does not 1. M. audouinii C

111
fluoresce bright green upon Woods lamp 2. M. canis
examination? 3. T. violaceum
4. T. schoenleinii
5. M. distortum
6.
16 Which of the following DOES NOT typically 1. Aspergillus species E
cause white superficial onychomycosis? 2. Fusarium species
3. Trichophyton
mentagrophytes
4. Scopulariopsis species
5. Trichophyton rubrum
6.
17 Sabouraud Dextrose Agar (SDA) Emmons 1. Peptones A
Modification contains which of the 2. Phenol red
following? 3. Chloramphenicol
4. Gentamicin
5. Cycloheximide
6.
18 This 16 year-old patient was recently 1. Distal Onychomycosis C
diagnosed with HIV, the diagnosis is: 2. Proximal Subungual
Onychomycosis
3. Proximal White Subungual
Onychomycosis
4. White Superficial
Onychomycosis
5. Paronychia with Candida
Onychomycosis
6.
19 Which two characteristics combined form 1. 1, 2 B
a significant risk for acquiring 2. 1, 3
sporotrichosis? 1. Animal handler 2. Male 3. 3, 4
gender 3. Alcoholism 4. Filipino or African 4. 4, 5
decent 5. Genetic predisposition 5. 3, 4
6.
20 Septate true hyphae are characterized by: 1. Transverse cross walls A
forming within the hyphae
2. Discrete unicellular bodies
3. Constrictions at septations
4. Branching occuring at
septations
5. A terminal cell that is
smaller than the others
6.
21 A 12 year old boy has a pruritic bullous 1. Trichophyton verrucosum C
eruption on his feet. A KOH is positive and 2. Microsporum canus
a fungal culture shows microconidia in 3. Trichophyton
grape-like clusters. The etiology is: mentagrophytes
4. Microsporum gypseum
5. Trichophyton tonsurans
6.
22 The most reliable method for 1. Morphology of D
distinguishing between Trichophyton microconidia
rubrum and T. mentagrophytes is: 2. Morphology of
macroconidia
3. Pigmentation studies

112
4. Hair perforation test
5. Colony morphology
6.
23 Culturing T. rubrum from a white opacity 1. Diabetes mellitus D
on the fingernail plate should prompt 2. Hypothyroidism
testing for what? 3. Hyperthyroidism
4. HIV
5. Cirrhosis
6.
24 Which actinomycotic organism has red 1. Streptomyces somaliensis D
grains? 2. Nocardia asteroides
3. Actinomadura madurae
4. Actinomadura pelletieri
5. Nocardia brasiliensis
6.
25 This dermatophyte has a growth 1. Trichophyton equinum E
requirement for inositol and thiamine 2. Trichophyton violaceum
3. Trichophyton concentricum
4. Trichophyton tonsurans
5. Trichophyton verrucosum
6.
26 Which dermatophyte is most commonly 1. T. mentag B
responsible for tinea corporis gladiotorum? 2. T. tonsurans
3. T. schonlenleinii
4. T. rubrum
5. E. floccosum
6.
27 Which of the following statements 1. A mass of hyphae is B
regarding mycelium is NOT true? mycelium
2. Mycelium has reproductive
capability
3. Hyphae forming
corkscrew-like turns are
spiral hyphae
4. Pectinate bodies are
hyphae resembling a comb
5. Racket forms are club-
shaped cells
6.
28 Which of the following is a fluorescent 1. Trichophyton rubrum C
ectothrix dermatophyte? 2. Trichophyton
mentagrophytes
3. Microsporum ferrugineum
4. Trichophyton violaceum
5. Microsporum nanum
6.
29 Which of the following is NOT used for 1. Chlorazol Black-E A
histologic examination for fungal 2. Gormori Methanamine
infection? Silver
3. Periodic Acid Schiff
4. Fontana-Masson
5. Mayer's mucicarmine
6.
30 Which of the following statements 1. Terbinafine is the C
regarding histoplasmosis is TRUE? treatment of choice

113
2. Histoplasma capsulatum is
a yeast at 25�C
3. The organisms are
intracellular on histological
examination
4. The disease is most
common in the San
Joaquin Valley
5. The disease is primarily a
mucocutaneous infection
6.
31 Which pathogen is not inhibited by 1. Cryptococcus B
cycloheximide? 2. Candida albicans
3. Prototheca
4. Scopuloniopsos brevicaulis
5. Scytalidium species
6.
32 A landscape worker complained of several 1. Nocardiosis C
tender nodules on the right dorsal hand 2. Blastomycosis
and forearm. The biopsy of the lesion 3. Sporotrichosis
showed asteroid bodies but no organisms. 4. Candidiasis
The likely diagnosis is: 5. Coccidioidomycosis
6.
33 All of the following statements regarding 1. Trichosporon species can B
superficial mycotic infections are true cause systemic disease in
EXCEPT: immunocompromised
patients
2. Malassezia furfur is the
most common cause of
tinea versicolor
3. Trichosporon beigelii
causes white piedra
4. Malassezia species may be
associated with neonatal
cephalic pustulosis
5. Piedraia hortae causes
black piedra
6.
34 Which of the following statements about 1. Arthroconidia are formed B
arthroconidia is correct? by budding
2. Arthroconidia are formed
by fragmentation of
hyphae
3. Arthroconidia are thick-
walled round cells
4. Arthroconidia are spores
that are produced in a sac
5. Arthroconidia are yeast
forms of dimorphic fungi
6.
35 All of the following are features of 1. Swelling B
mycetoma except: 2. Self-limited
3. Granules
4. Fascia and bone
involvement

114
5. Draining sinuses
6.
36 Which of the following statements 1. Trichophyton schoenleinii E
regarding dermatophyte infection FALSE? is a common cause of
favus
2. Microsporum canis is
associated with ectothrix
tinea capitis
3. Trichophyton violaceum is
normally associated with
endothrix tinea capitis
4. Microsporum audouinii
displays yellow
fluorescence with Wood�s
lamp examination
5. Trichophyton rubrum is
always an ectothrix
infection
6.
37 The outstanding characteristics of 1. Blastoconidia B
Aspergillus species on biopsy include: 2. Hyaline, septate
dichotomously branching
hyphae
3. Copper pennies
4. Ribbon-like filaments that
may be twisted and
distorted branching at
right angles
5. Stain with Fontana-Mason
stain
6.
38 Which one of the following is not caused 1. Angular cheilitis E
by a Candida infection? 2. Balanitis
3. Median rhomboid glossitis
4. Leukoplakia
5. Mycetoma
6.
39 Although this organism is not a fungus, it 1. Rhinosporidium seeberi D
stains with PAS and GMS and produces 2. Coccidioides immitis
spherules in tissue. This organism can be 3. Penicillium marneffei
identified as: 4. Prototheca wickerhami
5. Leishmania mexicana
6.
40 A biopsy shows broad-based budding thick 1. This yeast has a yeast D
walled yeast cells, 10-15 um with a double phase at room
contoured appearance. temperature
2. Does not grow at 37� C
3. Usually produces a severe
characteristic pulmonary
disease
4. May be found in dogs
5. Is transmitted by
mosquitoes
6.
41 A 56-year-old male with HIV presents with 1. Cryptococcus E

115
multiple umbilicated brown papules on the 2. Histoplasmosis
face. Which of the following is the least 3. Coccidiomycosis
likely cause of his lesions? 4. Penicilliosis
5. Blastomycosis
6.
42 Which of the following causes tinea 1. Epidermophyton floccosum B
imbricata? 2. Trichophyton concentricum
3. Trichophyton rubrum
4. Trichophyton
mentagrophytes
5. Trichophyton tonsurans
6.
43 Which of the following stains is specific for 1. KOH D
chitin? 2. KOH with DMSO
3. Swartz Lamkins
4. Chlorazol Black E
5. Calcofluor White
6.
44 Which of the following methods of direct 1. KOH D
microscopic examination for fungi is 2. Swartz Lamkins
glucan specific? 3. Chlorazol black E
4. Calcofluor white
5. Mayer's mucicarmine
6.
45 The organism that causes white piedra: 1. Does not cause C
onychomycosis
2. Is known as Piedraia
hortae
3. May also cause post-
operative wound infections
4. Grows as small, compact,
black-greenish velvety
colonies
5. Can only be treated by
cutting off affected hairs
6.
46 Penicillium marneffei is an infection 1. Mexico D
endemic to which part of the world? 2. South America
3. Africa
4. Southeast Asia
5. The Caribbean
6.
47 Black granules are found in mycetoma 1. Madurella grisea E
caused all the following organisms except: 2. M. mycetomatis
3. Exophiala jeanselmei
4. Curvularia
5. Scedosporium
apiospermum
6.
48 A neutropenic patient has erythematous 1. 1 C
nodules for several days. A biopsy shows 2. 1, 2
branching septate hyaline hyphae. Which 3. 2, 3
of the following conditions should be 4. 3, 4
included in the differential? 1. 5. 2, 3, 5
Coccidioidomycosis 2. Fusariosis 3. 6.

116
Aspergillosis 4. Zygomycosis 5.
Phaeohyphomycosis
49 A patient returns from a vacation in Brazil 1. Actinomycosis B
with keloidal-like nodules on the face and 2. Lobomycosis
arms. The patient denies any sick contacts 3. Chromoblastomycosis
or exposures and reports only lying on the 4. Mucormycosis
beach and swimming with dolphins. She 5. Sporotrichosis
likely has: 6.
50 This organism is the most common cause 1. Madurella mycetomatis C
of eumycotic mycetoma in the US: 2. Nocardia asteriodes
3. Scedosporium
apiospermum
4. Phialophora verrucosa
5. Wangiella dermatitis
6.
51 Which of the following organisms will 1. Epidermophyton floccosum C
cause infections of skin, nails and 2. Microsporum audouinii
endothrix hair? 3. Trichophyton schoenlinii
4. Microsporum canis
5. Trichophyton
mentagrophytes
6.
52 An organism producing an apricot colored 1. 1,5 B
colony and reflexive branching with few 2. 2,4
conidia was cultured from a patient from 3. 2,4,5
Africa. Which of the following is/are true? 4. 3, 4
1. This mostly likely is T. verrucosum 2. 5. 3, 4, 5
This most likely is T. soudanense 3. This 6.
most likely is T. violaceum 4. This
organism causes endothrix tinea capitis 5.
This organism is zoophilic
53 Clinically, actinomycotic mycetoma and 1. Academic B
eumycotic mycetoma appear identical. 2. Selecting the appropriate
The importance of identifying the etiologic therapy
agent is 3. Ordering appropriate
stains
4. Determine if the infection
is contagious
5. Determine if amputation is
necessary
6.
54 Which of the following statments 1. Pseudohyphae are seen in E
regarding pseudohyphae is incorrect? yeasts
2. Are constricted at
septations
3. Branching occurs at
septations
4. The terminal cell is smaller
than the others
5. Are not septated
6.
55 Which of the following organisms is most 1. Nocardia brasiliensis A
likely to cause a sporotrichoid nodule on 2. Phialophora verrucosa
the arm: 3. Rhinospiridium seeberii
4. Fusarium

117
5. Fonsecaea pedrosoi
6.
56 Which of the following dermatophytes 1. Microsporum gypseum C
causes endothrix tinea capitis? 2. Microsporum audouinii
3. Trichophyton tonsurans
4. Microsporum canis
5. Microsporum ferrugineum
6.
57 A creamy white colony might be any of 1. Candida albicans C
these organisms except: 2. Prototheca wickerhamii
3. Curvularia
4. Sporothrix schenckii at
37� C
5. Cryptococcus neoformans
6.
58 This organism grew on Sabouraud 1. Trichophyton rubrum C
Dextrose agar without any antimicrobials. 2. Microsporum distortum
The patient presented with onychomycosis 3. Scytalidium dimidiatum
of both great toenails. The nails were dark 4. Coccidioides immitis
and dystrophic, and thick scale was 5. Trichophyton violaceum
apparent on both feet. Identify the fungus. 6.
59 What organism produces round thick 1. Sporothrix schenckii E
walled spiny macroconidia and pear 2. Blastomyces dermatitidis
shaped microconidia? 3. Paracoccidioides
brasiliensis
4. Cryptococcus neoformans
5. Histoplasma capsulatum
6.
60 Cutaneous lesions of Cryptococcosis may 1. 2, 3, 4 E
be 1. Nodular 2. Papular 3. 2. 3, 4, 5
Granulomatous-ulcerative 4. Herpetiform 3. 2, 3, 4
5. Cellulitis-like: 4. 1, 3, 4, 5
5. All of these answers are
correct
6.
61 What is the most frequently reported 1. Aspergillus flavus A
cause of primary cutaneous aspergillosis? 2. Aspergillus fumigatus
3. Aspergillus niger
4. Aspergillus solani
5. Aspergillus marneffei
6.
62 Which of the following statements 1. Candida species displays A
regarding candidal infection is FALSE? true hyphae on potassium
hydroxide examination
2. Predisposing factors for
candidal infection include
diabetes mellitus,
hyperhidrosis and broad
spectrum antibiotics
3. Candida albicans is the
number one cause of
mucocutaneous infections
4. Candidal infections
typically do not spare the
scrotum

118
5. Candida species may be
associated with granuloma
gluteale infantum
6.
63 Which of the following is the most useful 1. Arthroconidia in every C
morphologic feature in identifying the other cell
mycelial phase of Histoplasma 2. Encapsulated spores 2-5
capsulatum? um
3. Tuberculate macroconidia
8-14 um
4. Small oval conidia on long
thin conidiophores
5. Microconidia laterally
along the hyphae strand
6.
64 A 49-year-old man has painless 1. Acremonium D
subcutaneous nodules on his feet with 2. Curvalaria
sinus tracts and abscesses. Pathology 3. Exophilia jeanselmei
shows "grain" in sinus tract drainage. A 4. Pseudallescheria boydii
diagnosis of eumycotic mycetoma is 5. Nocardia
made. What is the most common cause in 6.
the United States?
65 Of the following Candida species, which is 1. Candida albicans C
likely to cause disseminated Candidiasis 2. C. parapsilosis
with cutaneous lesions? 3. C. tropicalis
4. C. krusei
5. C. neoformans
6.
66 A 37 year old woman from New Mexico, 1. Amphotericin B A
now 30 weeks pregnant, presents with flu- 2. Terbinafine
like symptoms. Chest x-ray revealed 3. Itraconazole
diffuse miliary infiltrates, with blood 4. Griseofulvin
cultures growing Coccidioidis immitis. 5. No therapy
What is the treatment of choice? 6.
67 A pet store owner comes into your office 1. Microsporum canis A
with pruritic scaly plaques on his arms. A 2. Epidermophyton floccosum
fungal culture demonstrates a yellow 3. Trichophyton rubrum
colony with spindle-shaped macroconidia 4. Microsporum gypseum
which grows on polished rice grains. The 5. Trichophyton tonsurans
organism is: 6.
68 Which of the following is true regarding 1. Difficult to grow, requires A
culture of the organisms that cause olive oil overlay
Pityriasis versicolor? 2. Grow easily on Mycosel
media
3. Cannot be cultured
4. Grow easily on
Dermatophyte Test Media
5. Grow easily on Sabouraud
Dextrose Agar
6.
69 Which of the following is most likely to be 1. North American A
a primary cutaneous infection? Blastomycosis
2. Histoplasmosis
3. Paracoccidioidomycosis
4. Cryptococcosis

119
5. Penilliosis
6.
70 A 34-year-old male patient presents with a 1. Coccidioidomycosis C
verrucous lesion of the nasal mucosa; the 2. Protothecosis
biopsy showed spherules ranging in size 3. Rhinosporidiosis
from 200 to 325 um. The mature 4. Pseudallescheriosis
endospores have a rough appearance. The 5. Cryptococcosis
diagnosis is: 6.
71 Which of the following is FALSE with 1. Arthroconidia are formed C
regards to asexual reproduction of fungi? by the fragmentation of
hyphae
2. Sporangia are spores that
are produced in a sac
3. Chlamydoconidia are thin-
walled and are susceptible
to environmental injury
4. Conidia are cells produced
on the sides or ends of
hyphae
5. Dematophytes produce
conidia
6.
72 This agent causes a superficial, 1. Piedra hortae D
asymptomatic infection usually on the 2. Cladosporium carrionii
palms of the hands. The lesions are flat, 3. Aureobasidium pullulans
nonscaly and appear as irregularly shaped 4. Phaeoannellomyces
brown macules: werneckii
5. Exophiala jeanselmei
6.
73 Which of the following is true regarding 1. DTM contains C
Dermatophyte Test Media (DTM)? chlortetracycline and
minocycline
2. Alizarin red is the indicator
present in DTM
3. Non-dermatophytes cause
the media to turn yellow
due to acid byproducts
4. DTM is useful for culturing
dermatophytes from skin
and nails, but not hair.
5. Dermatophytes utilize
glucose as a carbon
source, producing alkaline
by-products.
6.
74 Choose the correct statement regarding 1. The most common form of B
Coccidioidomycosis: primary inoculation is
cutaneous
2. Dissemination may involve
the bones, joints, viscera,
brain and skin
3. Causative organism, C.
immitits, is a thick-walled
spherule with a
polysaccharide capsule

120
demonstrated with Alcain
blue
4. Erythema nodosum is a
poor prognotic sign
5. Outbreaks occur in the
Mississippi and Ohio River
Valley
6.
75 Which one of the following agents 1. Thymidine kinase D
accounts for the depigmentation seen in 2. Ketoconazole
pityriasis versicolor? 3. Acetone
4. Dicarbocyclic acid
5. Postinflammatory effect
6.
76 Which of the following is not true 1. Cycloheximide is not found E
regarding fungal culture media containing in Sabouraud Dextrose
cycloheximide? Agar (SDA) Emmons
Modification
2. Cycloheximide is found in
Mycosel
3. Cycloheximide is found in
Dermatophyte Test Media
4. Cycloheximide is found in
Mycobiotic
5. Cycloheximide inhibits
bacterial flora
6.
77 Septate hyphae with 90� branching and 1. Histoplasma capsulatum E
thick walled barrel shaped arthroconidia 2. Sporothrix schenckii
alternating with empty cells best describes 3. Microsporum gypseum
the microscopic morphology of: 4. Trichophyton tonsurans
5. Coccidioides immitis
6.
78 Tinea imbricata is most frequently caused 1. T. mentagrophytes C
by: 2. M. Audouinii
3. T. concentricum
4. E. floccosum
5. M. furfur
6.
79 What is the most likely cause of this 1. T. tonsurans C
infection? 2. T. schoenleinii
3. M. canis
4. T. mentag
5. T. rubrum
6.
80 A patient has a positive Wood's light exam 1. Microsporum ferrugineum A
that is caused by pteridine. What is the 2. Corynebacterium
causative condition? 3. Trichophyton violaceum
4. Pseudomonas
5. Trichophyton tonsurans
6.
81 Which of the following stains is NOT used 1. Mucicarmine D
to stain fungal elements? 2. Periodic Acid Schiff
3. Fontana-Masson
4. Von Kossa

121
5. Gormori methenamine
silver
6.
82 A mass of hyphae is known as a: 1. Sporangia C
2. Conidia
3. Mycelium
4. Chlamydoconidia
5. Pseudohyphae
6.
83 The most sensitive microscopic test for 1. Potassium Hydroxide D
fungal infection is: 2. Potassium Hydroxide with
DMSO
3. Chlorazol Black E
4. Calcofluor white
5. Swartz Lamkins stain
6.
84 Which of the following statements 1. Itraconazole is the A
regarding lobomycosis is FALSE? treatment of choice
2. It resembles a �chain of
coins� on histopathology
3. It is also known as
�keloidal blastomycosis�
4. Lacazia (formerly Loboa)
loboi is the number one
cause
5. The infection also occurs in
dolphins
6.
85 Erythematous to violaceous papules that 1. Trichosporon asahii A
may progress to nodular and necrotizing 2. Trichosporon beigelii
skin lesions in neutropenic patients may 3. Trichosporon ovoides
be caused by which of the following 4. Trichosporon inkin
organisms? 5. Trichophyton rubrum
6.
86 A child presents with several yellowish, 1. M. gypseum A
cup-shaped crusts on the scalp, some with 2. M. canis
single hairs piercing through the center. 3. T. tonsurans
KOH prep reveals arthroconidia and 4. T. verrucosum
airspaces within the hair shaft. A likely 5. T. mentagrophytes
cause is: 6.
87 This organism produces an endothrix tinea 1. Trichophyton E
capitis: mentagrophytes
2. Microsporum gypseum
3. Microsporum nanum
4. Trichophyton verrucosum
5. Trichophyton soudanense
6.
88 Which structure is found in a biopsy of 1. Septate hyphae D
Candida tropicalis, it is branching and 2. Arthroconidia
pinching in at the points of septations: 3. Mosaic fungus
4. Pseudohyphae
5. Pectinate bodies
6.
89 Cryptococcus neoformans resides in: 1. The Mississippi Valley C
region

122
2. Chicken roosts
3. Pigeon droppings
4. Sandy soil
5. None of these answers are
correct
6.
90 A blood culture from a neutropenic patient 1. Scopulariopsis sp C
with onychomycosis grew which of the 2. Aspergillus sp
following organism: 3. Fusarium sp
4. Acremonium sp
5. T. rubrum
6.
91 A 6 year-old Hispanic girl came into the 1. M. audouinii D
clinic with a history of alopecia and scaly 2. M. canis
scalp for 3 weeks. The Wood's lamp 3. M. gypseum
examination was negative and her cervical 4. T. tonsurans
lymph nodes were not enlarged. It was 5. T. rubrum
noted that the hairs were broken off at the 6.
surface of the scalp. The most likely
organism to be isolated would be:
92 This organism does not produce 1. Trichophyton rubrum B
microconidia. The macroconidia are club 2. Epidermophyton floccosum
shaped, and smooth walled, they grow 3. Microsporum gypseum
singly or in clusters. 4. Microsporum canis
5. Trichophyton
mentagrophytes
6.
93 A solitary chancre-like lesion appeared on 1. There is a resistance due A
the arm of a florist who has a pet dog. No to a prior exposure
organisms were seen on biopsy, however 2. There is a decreased
Sporothrix schenckii was cultured from the resistance due to AIDS
tissue. This infection is known as �fixed 3. The infection was acquired
cutaneous sporotrichosis�. It remains from the dog rather than
fixed because: from a plant
4. The infection occurred in
the absence of tissue
injury
5. This species of Sporothrix
has limited infectivity
6.
94 The most common eumycotic organism in 1. White A
the US produces which color grains in its 2. Black
microcolony? 3. Red
4. Yellow
5. Green
6.
95 Which of the following methods of direct 1. Chlorazol black E A
microscopic examination is chitin specific? 2. Calcofluor white
3. Gomori Methenamine
Silver
4. Fontana-Masson
5. KOH
6.
96 The diagnosis is phaeohyphomycosis, 1. 1,2,3 C
what is the organism 1. Alternaria sp. 2. 2. 2,3,4

123
Bipolaris sp. 3. Fusarium sp. 4. Exophiala 3. 1,2,4
sp. 4. 1,3,4
5. All of these answers are
correct
6.
97 A patient with a pulmonary infection and 1. Penicillium marneffei C
cervical adenopathy has vesicles, papules 2. Coccidioides immitis
and ulcerations on the oral and nasal 3. Paracoccidioides
mucosa. Tissue culture reveals yeast with brasiliensis
multiple buds giving a �mariner�s 4. Blastomyces dermatitidis
wheel� appearance. The organism is: 5. Leishmania mexicana
6.
98 Which of the following most accurately 1. Cells produced on the end A
describe conidia? or sides of hypha or
conidiophore
2. Spores that are produced
in a sac
3. Thick-walled round cell
4. Formed by budding
5. Formed by fragmentation
of hyphae
6.
99 Which of the following statements about 1. Are in the mould form in E
dimorphic fungi is INCORRECT? the environment
2. Are in yeast/spherule form
in tissue
3. Are in mould form at 25
degrees C
4. Are in yeast/spherule form
at 37 degrees C
5. Are in mould form in tissue
6.
10 A 45 year-old agricultural worker from 1. Yeast cells in chains and a A
0 Brazil presented with ulcers of the buccal large thick walled round
mucosa and tongue. Cervical lymph nodes central yeast cell
were tender and enlarged. The biopsy surrounded by several
would most likely reveal: thinly attached budding
smaller yeast cells
2. Yeast cells with large
capsules
3. Small budding yeast cells
4. Yeast cells with
pseudohyphae
5. Copper pennies
6.
10 The most common cause of tinea capitis in 1. Trichophyton schoenleinii B
1 the United States today is: 2. T. tonsurans
3. T. mentagrophytes
4. Microsporum audouinii
5. M. canis
6.
10 The etiologic agent responsible for White 1. Candida albicans E
2 Piedra is: 2. Pityrosporum obiculare
3. Corynebacterium tenuis
4. Piedra hortai

124
5. Trichosporon ovoides
6.
10 An elderly lady with moccasin-type tinea 1. Trichophyton verrucosum C
3 pedis has a fungal culture which 2. Microsporum canus
demonstrates smooth, teardrop-shaped 3. Trichophyton rubrum
microconidia which produce a �port- 4. Microsporum gypseum
wine� pigment. The organism is: 5. Trichophyton
mentagrophytes
6.
10 A slimy, mucoid colony growing on 1. Candida albicans B
4 Sabouraud's agar at 37 degrees Celsius is 2. Cryptococcus neoformans
observed to produce urease. The organism 3. Candida glabrata
is: 4. Candida tropicalis
5. Aspergilus flavus
6.
10 Numerous umbilicated nodules resembling 1. 2 E
5 molluscum contagiosum developed in a 2. 3
patient with recently diagnosed HIV 3. 2, 4
infection. Which of the following fungi 4. 4, 5
might be the causative agent(s)? 1. 5. 2, 4, 5
Candida tropicalis 2. Cryptococcus 6.
neoformans 3. Aspergillus fumigatus 4.
Histoplasma capsulatum 5. Penicillium
marneffei
10 Two weeks after a bicycle accident a 25- 1. Aspergillosis C
6 year-old female diabetic patient 2. Actinomycosis
complained of a swollen tender right 3. Mucormycosis
cheek The PAS biopsy demonstrated ring 4. Dermatophytosis
forms and distorted wide hyphae with few 5. Fusariosis
septations and right angle branching. The 6.
diagnosis is:
10 This organism causes a resistant tinea 1. Aspergillus flavus B
7 pedis indistinguishable from 2. Scytalidium dimidiatum
dermatophytosis: 3. Curvularia sp.
4. Scopulariopsis
5. Fonsecaea sp
6.
10 Which of the following is TRUE regarding 1. First line treatment in C
8 coccidiomycosis? pregnancy is itraconazole
2. Droplet transmission is the
most common method of
acquisition of disease
3. Southeast Asians are at a
higher risk of disseminated
disease
4. Erythema nodosum is
associated with a poor
prognosis
5. Approximately 75% of
those contracting the
disease will be
symptomatic
10 A horticulturist of sphagnum moss 1. Gastrointestinal distress A
9 topiaries comes in with a nodular eruption 2. Shortness of breath
with lymphangitic spread and treatment 3. Flushing

125
with oral potassium iodide is initiated. 4. Angioedema
What is the most well recognized side 5. Pruritus
effect of this treatment.
11 A whitish, heaped and convoluted colony 1. Microsporum canis E
0 with growth submerged into the agar and 2. Microsporum ferrugineum
a colorless reverse was isolated from the 3. Trichophyton rubrum
scalp of a 35-year-old male. The organism 4. Trichophyton tonsurans
did not produce any conidia. The diagnosis 5. Trichophyton schoenleinii
most likely is:
11 A patient with scaly feet has a positive 1. Microsporum canis B
1 KOH. Fungal culture reveals smooth, club- 2. Epidermophyton floccosum
shaped macroconidia attached to hyphae 3. Trichophyton rubrum
in groups. No microconidida are seen. The 4. Microsporum gypseum
organism is: 5. Trichophyton tonsurans
11 Yeast: 1. Are filamentous fungi E
2 2. Are characterized by
tubular branching cells
3. Form fuzzy colonies
4. Form smooth colonies
5. Are unicellular oval to
round cells that reproduce
by budding or fission
6.
11 A young girl presented with a scaly 1. 1,2,4 B
3 annular facial rash and alopecia of her 2. 1,2,3,4
lower eyelashes. Which of the following 3. 1,3,4
statements is/are true? 1. A KOH prep and 4. 1,2,5
fungal culture might confirm the diagnosis. 5. 1,2,3,5
2. The etiologic agent might be
Microsporum canis. 3. The family puppy
might be infected. 4. Griseofulvin would be
the drug of choice. 5. A topical azole
cream would be the drug of choice.
11 This dermatophyte is not an anthropophilic 1. Trichophyton rubrum C
4 organism: 2. Epidermophyton floccosum
3. Microsporum gypseum
4. Microsporum audouinii
5. Trichophyton soudanense
11 A 30 year-old male living in the Chicago 1. Blastomycosis A
5 suburbs complained of a slowly growing 2. Cryptococcosis
verrucous plaque with sharp borders on 3. Candidiasis
his left wrist. A biopsy revealed yeast cells 4. South American
10-14 um. Mucicarmine was negative. A Blastomycosis
fluffy white colony grew at room 5. Histoplasmosis
temperature having small round conidia
on thin conidiophores. The diagnosis is:
11 Medlar bodies are diagnostic of infection 1. Blastomycosis B
6 with which organism? 2. Chromomycosis
3. Coccidiomycosis
4. Histoplasmosis
5. Sporotrichosis
11 Which of the following causes "black dot 1. M. canis B
7 ringworm"? 2. T. violaceum
3. T. verrucosum
4. M. gypseum

126
5. M. auddouinii
11 All of the following are common causes of 1. Cladosporium carrionii D
8 chromoblastomycosis EXCEPT: 2. Rhinocladiella aquaspera
3. Phialophora verrucosa
4. Pseudallescheria boydii
5. Fonsecaea pedrosi
11 Which of the following organisms causes 1. Trichophyton schoenleinii A
9 favus? 2. Trichophyton
mentagrophytes
3. Microsporum canis
4. Trichophyton rubrum
5. Microsporum distortum

9. Pediatric Dermatology
# Question Answers Correc
t
Answe
r
1 Which treatment choice would be 1. Advil D
contraindicated in a one-year old 2. Acetaminophen
child who presents with 3. Hydration
monomorphous, nonpruritic flat- 4. Corticosteroids
topped papules on the face, 5. Observation
buttocks, extremities, palms and 6. Show Explaination
soles?
2 What is the most likely diagnosis? 1. Psoriasis D
2. Pityriasis rosea
3. Cutaneous T-cell lymphoma
4. Contact dermatitis
5. Lichen planus
6. Show Explaination
3 An infant who presents with this 1. The lesion will be on the left- A
abnormality will often have the side in a segmental
following: distribution
2. Airway restriction
3. Have multiple liver
hemangiomas
4. Most likely be a male infant
5. An anterior fossa
malformation
6. Show Explaination
4 Late onset subungual keratotic 1. Incontinentia pigmenti A
tumors are associated with: 2. Neurofibromatosis Type 1
3. Carney complex
4. Cowden syndrome
5. Basal cell nevus syndrome
6. Show Explaination
5 Pastia�s lines are characteristic for 1. Measles B
which eruption: 2. Scarlet fever
3. Rubella
4. Kawasaki�s disease
5. Mumps

127
6. Show Explaination
6 All four subtypes of Phakomatosis 1. Nevus spilus C
Pigmentovascularis have which 2. Epidermal nevi
feature in common: 3. Nevus flammeus
4. Nevus anemicus
5. Dermal Melanocytosis.
6. Show Explaination
7 What is the best treatment option? 1. Oral cephalexin D
2. Oral acyclovir
3. Observation
4. Topical tretinoin
5. Topical ketoconazole
6. Show Explaination
8 Variants of xeroderma pigmentosum 1. Nucleotide excision repair D
are due to all of the following defects 2. Helicase
except: 3. Endonuclease
4. Thymidine kinase
5. Postrepliction repair
6. Show Explaination
9 Which of the following can present 1. Ichthyosis vulgaris C
as collodion baby? 2. X-linked ichthyosis
3. Lamellar ichthyosis
4. Bullous congenital
ichthyosiform erythroderma
5. Sjogren-Larsson syndrome
6. Show Explaination
10 A 6 month-old presents with orange- 1. Hematocrit E
brown crusted plaques around the 2. Calcium
mouth and groin. Several bullae are 3. Platelet count
present on the fingers and toes. 4. ALT
Which of the following laboratory 5. Alkaline phosphatase
values is likely to be abnormal? 6. Show Explaination
11 Psammomatous melanotic 1. Bloom syndrome B
schwannomas are associated with: 2. Carney complex
3. Neurofibromatosis Type 1
4. Neurofibromatosis Type 2
5. Tuberous sclerosis
6. Show Explaination
12 What is the best test to confirm a 1. DsDNA antibody D
diagnosis? 2. Anti Ro antibody
3. Biopsy
4. KOH
5. Gram stain
6. Show Explaination
13 Juvenile self-healing papular 1. Arthralgias A
mucinosis is associated with which of 2. Cataracts
the findings? 3. Type I diabetes
4. Calcinosis cutis
5. Xeropthalmia
6. Show Explaination
14 Which of the following clinical signs 1. Clitoral hypertrophy A
is most likely associated? 2. Neurofibromas
3. Axillary freckling
4. Hypertrichosis
5. Alopecia
6. Show Explaination
15 A full term neonate is noted to have 1. Miliaria C
small pustules with no underlying 2. Erythema toxicum
erythema present at delivery. The neonatorum

128
pustules are easily removed with 3. Transient neonatal pustular
clearing of the vernix and a melanosis
collarette appears. A gram stain is 4. Congenital candidiasis
done showing predominately 5. Urticaria pigmentosa
neutrophils without bacteria. What is 6. Show Explaination
the most likely diagnosis?
16 What syndrome is the disorder 1. Turner syndrome C
shown in the photo associated with? 2. Down syndrome
3. Noonan syndrome
4. Griscelli syndrome
5. Bloom syndrome
6. Show Explaination
17 A boy is noted at birth to have 1. Corneal opacities C
coarse scales over his trunk and 2. Cryptorchidism
extremities. The face, palms, soles 3. Ectropion
and flexures are spared. What is the 4. Prolonged maternal labor
least likely association? 5. Neurologic abnormality
6. Show Explaination
18 What is the most likely diagnosis? 1. Psoriasis E
2. Langerhans cell histiocytosis
3. Granuloma gluteale infantum
4. Contact dermatitis
5. Perianal streptococcal
disease
6. Show Explaination
19 What is the most common tumor 1. Trichoblastoma A
associated with this condition? 2. Squamous Cell carcinoma
3. Sebaceous carcinoma
4. Basal cell carcinoma
5. Trichoadenoma
6. Show Explaination
20 �Slapped cheeks� followed by a 1. ssDNA virus A
lacy eruption on extremities: 2. dsDNA virus
3. ssRNA virus
4. dsRNA virus
5. Streptococcus
6. Show Explaination
21 A patient presents with an epidermal 1. Transglutaminase D
nevus of the lower abdomen. You 2. Keratin 6a/16
take a skin biopsy to confirm the 3. Keratin 6b/17
diagnosis. The pathology report 4. Keratin 1/10
indicates that there were findings of 5. Keratin 2e
epidermolytic hyperkeratosis. Which 6. Show Explaination
of the following defects is her
offspring at risk for based on these
findings?
22 Koplik spots typically appear: 1. Before the exanthem A
2. At the same time as the
exanthem
3. 1 week after the exanthem
4. 6 weeks after the exanthem
5. 8 weeks after the exanthem
6. Show Explaination
23 This condition may be associated 1. Hypercalcemia D
with which of the following: 2. Hyperlipidemia
3. Vitiligo
4. Epidermolysis bullosa
dystrophica
5. Perinatal trauma
6. Show Explaination

129
24 An infant presents with the lesion 1. Sternal clefting D
depicted in the photo. Which of the 2. Supraumbilical raphe
following is least likely? 3. Dandy-Walker malformation
4. Seizure disorder
5. Congenital cataracts
6. Show Explaination
25 An 8 year-old boy presents with pink, 1. Paramyxovirus E
flushed cheeks and a low-grade 2. Togavirus
fever. On week later, the following 3. Group A streptococcus
lacy eruption appeared. What is the 4. HHV6
most likely etiology? 5. Parvovirus
6. Show Explaination
26 What is the most likely diagnosis is 1. Psoriasis A
this 16 year old patient who 2. Pityriasis rosea
developed generalized eruption 2 3. Cutaneous T-cell lymphoma
weeks after onset of sore throat due 4. Contact dermatitis
to strep infection? 5. Lichen planus
6. Show Explaination
27 A young girl presents with enlarged 1. Palmoplantar keratoderma D
tongue, exomphalmos, and 2. Lymphatic malformation
organomegaly and has history of 3. Acral edema
Wilm's tumor. What cutaneous 4. Midline capillary
finding is most likely on physical malformation
exam? 5. Angiokeratoma
6. Show Explaination
28 What is the most likely diagnosis? 1. Psoriasis A
2. Langerhans cell histiocytosis
3. Seborrheic dermatitis
4. Contact dermatitis
5. Perianal streptococcal
disease
6. Show Explaination
29 POEMS syndrome is associated with 1. Premature aging D
which of the following: 2. Odontogenic cysts
3. Eye abnormalities
4. M protein
5. Saddle nose deformity
6. Show Explaination
30 Which presentation of psoriasis is 1. Pustular psoriasis E
more common in children: 2. Acrodermatitis continua of
Hallopeau
3. Keratoderma blennorragica
4. Erythrodermic psoriasis
5. Guttate psoriasis
6. Show Explaination
31 What is the most likely diagnosis? 1. Incontinentia pigmenti A
2. Bullous pemphigoid of
infancy
3. Herpes zoster
4. Disseminated herpes simplex
5. Epidermolysis bullosa
simplex
6. Show Explaination
32 A 3 month-old girl with multiple 1. Bleeding complications C
hemangiomas along her right jaw is 2. Underlying bone
at increased risk for: abnormalities
3. Subglottic hemangioma
4. Oral obstruction
5. Hearing defecits
6. Show Explaination

130
33 Which of the following may be 1. Paronychia D
associated? 2. Cleft palate
3. AVM
4. Seizure disorder
5. Atrial septal defect
6. Show Explaination
34 What is the best therapeutic option? 1. Oral cephalexin C
2. Topical immune modulator
3. Oral acyclovir
4. Topical mupirocin
5. Oral prednisone
6. Show Explaination
35 Rapp-Hodgkin is caused by a defect 1. Plakophilin D
in the following gene: 2. Desmoglein 1
3. Ectodysplasin A
4. P63
5. Connexin 30
6. Show Explaination
36 An eight year-old boy presents with 1. Paramyxovirus E
pink, flushed cheeks and a low-grade 2. Togavirus
fever. Eruptions then appeared. 3. Group A streptococcus
What is the most likely etiology? 4. HHV6
5. Parvovirus
6. Show Explaination
37 What deficiency is responsible for 1. Thiamine C
this condition? 2. Niacin
3. Zinc
4. Vitamin C
5. Vitamin A
6. Show Explaination
38 A newborn presents with a pustular 1. Muckle-Wells syndrome B
rash, joint swelling, oral mucosal 2. Deficiency of the interleukin-
lesions, and pain with movement. 1-receptor antagonist (DIRA)
Over time, cutaneous pustulosis, 3. Familial Mediterranean fever
ranging from discrete crops of 4. Hyper-IgD syndrome
pustules to generalized severe 5. PAPA syndrome
pustulosis and ichthyosiform lesions 6. Show Explaination
develop. In addition the newborn
develops sterile multifocal
osteomyelitis and periostitis. What is
this condition?
39 What is the diagnosis? 1. Unilateral laterothoracic A
exanthem
2. Roseola
3. Rubella
4. Urticaria
5. Contact dermatitis
6. Show Explaination
40 Pachyonychia congenita type 2 is 1. Increased risk of malignancy C
most commonly associated with 2. Poikiloderma
which of the following: 3. Natal teeth
4. Aplastic nails
5. Deafness
6. Show Explaination
41 What is the best therapeutic option? 1. Oral cephalexin C
2. Topical immune modulator
3. Oral acyclovir
4. Topical mupirocin
5. Oral prednisone
6. Show Explaination

131
42 Posterior auricular adenopathy is a 1. Measles D
common feature of which exanthem: 2. Mumps
3. Scarlet fever
4. Rubella
5. Erythema infectiosum
6. Show Explaination
43 816 activating mutation in c-kit are 1. Adults with systemic disease A
found most often in which subset of refractory to imantinib
patients with mastocytosis? 2. Adults with systemic disease
associated with eosinophilia
3. Patients with familial history
of mastocytosis
4. Adults with Telangiectasia
Macularis Eruptiva Perstans
5. As a mosaic mutation in
children with solitary
cutaneous mastocytoma
6. Show Explaination
44 A deficiency of the surface 1. Wiskott-Aldrich syndrome A
glycoprotein sialophorin is seen in 2. Chronic granulomatous
which immunedeficient disease? disease
3. Job syndrome
4. Severe combined
immunodeficiency syndrome
5. Leiner�s disease
6. Show Explaination
45 A healthy, full-term infant develops a 1. Gram-positive bacteria D
pustular, erythematous eruption on 2. Predominantly neutrophils
her face and trunk on the third day 3. Multi-nucleated giant cells
of life. A smear taken from one of 4. Predominantly eosinophils
these pustules would show: 5. Hyphae
6. Show Explaination
46 What is the most common tumor 1. Trichoblastoma A
associated with this condition? 2. Syringocystadenoma
papilliferum
3. Sebaceous carcinoma
4. Basal cell carcinoma
5. Trichoadenoma
6. Show Explaination
47 Multiple cylindromas are associated 1. Myotonic dystrophy D
with: 2. Cowden syndrome
3. Carney complex
4. Trichoepitheliomas
5. Pilomatrichomas
6. Show Explaination
48 What is the most likely diagnosis? 1. Papular acrodermatitis of A
childhood
2. Mucocutaneous lymph node
syndrome
3. German measles
4. Letterer-Siwe disease
5. Exanthem subitum
6. Show Explaination
49 In a child with zinc deficiency, yet 1. Magnesium D
normal or near normal zinc levels, 2. Niacin
which test could be a valuable 3. Manganese
adjunctive test? 4. Alkaline phosphatase
5. Iron
6. Show Explaination
50 A 10 year old girl presents with 1. Renal ultrasound B

132
desquamation of the fingertips. 2. ASO titer
Which exam should be ordered? 3. EEG
4. Eye exam
5. Chest x-ray
6. Show Explaination
51 A 6-year-old boy presents with 1. GNAS A
osteoma cutis on his face. Which of 2. NSDHL
the following genes is most likely 3. PTEN
defective? 4. SPINK5
5. SLURP1
6. Show Explaination
52 Osteopathia striata is found in which 1. McCune-Albright syndrome E
disorder? 2. Neurofibromatosis I
3. Buschke-Ollendorff syndrome
4. Gorlin's syndrome
5. Focal dermal hypoplasia
6. Show Explaination
53 This patient had significantly 1. Dermatomyositis A
elevated serum CPK. The likely 2. Lupus erythematosus
diagnosis is: 3. Psoriasis
4. Atopic dermatitis
5. Lichen planus
6. Show Explaination
54 The causative agent of Roseola is: 1. A ssDNA virus B
2. A dsDNA virus
3. A ssRNA virus
4. A dsRNA virus
5. Streptococcus
6. Show Explaination
55 Neonatal acne is associated with 1. Staphylococcus D
species of which organism: 2. Propionibacterium
3. Candida
4. Malassezia
5. Streptococcus
6. Show Explaination
56 A patient presents with multiple 1. AML B
juvenile xanthogranulomas, axillary 2. CML
freckling, multiple caf�-au-lait 3. CLL
macules, three neurofibromas and a 4. Medulloblastoma
family history of NF-1. What other 5. Pancreatic carcinoma
condition is this patient at increased 6. Show Explaination
risk for?
57 An infant presents with the lesion 1. The lesion will go through C
depicted in the photo. What rapid growth followed by
information do you provide the stabilization and regression
parents regarding this condition? 2. The lesion will respond to
laser therapy
3. The lesion will persist and
may grow further
4. The lesion will resolve with
antifungal therapy
5. The lesion is associated with
a XO karyotype
6. Show Explaination
58 The lesion depicted is most often 1. Polyostotic fibrous dysplasia A
associated with: 2. Chondrodysplasia punctata
3. Sphenoid wing hypoplasia
4. Cleft palate
5. Osteopoikilosis
6. Show Explaination

133
59 Which of the following bullous 1. Bullous Dermolysis of the C
disorders is due to a target antigen Newborn
that is a 97 kD protein which is a 2. Neonatal Pemphigus
member of BPAG-2? 3. Linear IgA Bullous
Dermatosis
4. Junctional epidermolysis
bullosa (Herlitz)
5. Epidermolysis bullosa
simplex
6. Show Explaination
60 What is the most common cause of 1. Strep D
neonatal purpura fulminans? 2. Staph
3. Varicella
4. Protein C deficiency
5. Factor V Leiden deficiency
6. Show Explaination
61 A newborn infant presents with ring 1. Alopecia areata B
of long, dark, coarse hair 2. Ectopic brain tissue
surrounding a midline scalp patch of 3. Thyroid disease
alopecia. What is associated with 4. Nevus sebaceus of Jadassohn
this finding? 5. Deafness
6. Show Explaination
62 Schimmelpenning-Feuerstein-Mims 1. Osteopokilosis E
syndrome may be associated with 2. Polyostotic fibrous dysplasia
which of the following: 3. Osteopathia striata
4. Chondrodysplasia punctata
5. Hypophosphatemic rickets
6. Show Explaination
63 The risk of fetal death with 1. First D
intrauterine parvovirus infection may 2. Second
occur with infection in which 3. Third
trimester: 4. First, Second and Third
5. None of these answers are
correct
6. Show Explaination
64 The standard of care of patients with 1. Prednisone C
acute Kawasaki�s disease is: 2. Supportive care
3. Aspirin and IVIG
4. Penicillin
5. Acetaminophen and IVIG
6. Show Explaination
65 A neonate presents with a large 1. Tram-track calcifications C
segmental hemangioma of the V1 2. Anterior fossa defect
distribution. What are features may 3. Posterior fossa defect
be associated with this finding? 4. M-paraproteinemia
5. Muscular dystrophy
6. Show Explaination
66 A child presents with high fever, 1. Two percent D
strawberry tongue, 2. Five percent
lymphadenopathy, and a 3. Ten percent
polymorphous exanthem. 4. Twenty-five percent
Approximately how many of these 5. Sixty percent
patients will have coronary artery 6. Show Explaination
aneurysms if left untreated?
67 What syndrome can accessory tragi 1. Goldenhar syndrome A
be associated with? 2. Turner syndrome
3. Neurofibromatosis
4. Ichthyosis
5. Birt Hogg Dube
6. Show Explaination

134
68 Which of the following disorders is 1. Langerhans cell histiocytosis B
more likely to occur in children with 2. Perianal pseudoverrucous
chronic fecal incontinence? papules and nodules
3. Perianal streptococcal
disease
4. Granuloma gluteale infantum
5. Seborrheic dermatitis
6. Show Explaination
69 What the most likely diagnosis? 1. Atopic dermatitis A
2. Lamellar ichthyosis
3. Rud syndrome
4. Wiskott-Aldrich syndrome
5. Keratosis pilaris
6. Show Explaination
70 Menkes kinky hair syndrome is 1. Trichorrhexsis invaginata D
associated most commonly with: 2. Trichostasis spinulosa
3. Pili multigemini
4. Pili torti
5. Plica neuropathica
6. Show Explaination
71 What is the most likely diagnosis? 1. Traction alopecia C
2. Tinea capitis
3. Aplasia cutis congenita
4. Nevus sebaceous
5. Alopecia areata
6. Show Explaination
72 Which of the following should be the 1. Barium swallow C
next step in the management of this 2. spine x-ray
patient? 3. Cardiac evaluation
4. Administration of oral
antibiotics
5. Pulmonary function studies
6. Show Explaination
73 A 2 day-old full term neonate 1. Erythema toxicum A
develops blotchy erythematous neonatorum
macules with small central pustules 2. Incontinentia pigmenti
over the upper trunk and 3. Urticaria pigmentosa
extremities. A gram stain reveals 4. Transient neonatal pustular
predominantly eosinophils. What is melanosis
the most likely diagnosis? 5. Miliaria
6. Show Explaination
74 A young boy presents with a port- 1. Type I D
wine stain, a Mongolian spot, and a 2. Type II
nevus spilus. Which phakomatosis 3. Type III
pigmentovascularis does he have? 4. Type IV
5. Type V
6. Show Explaination
75 A newborn has a nodule over his 1. Observation D
lumbar spine. Skin biopsy reveals a 2. Excision of the lesion
lipoma. The next appropriate step is: 3. Genetic testing
4. Imaging study
5. Malignancy work up
6. Show Explaination
76 Which finding is associated with 1. Cutis verticis gyrata A
Pachydermoperiostosis: 2. Palmoplantar hyperkeratosis
3. Mucosal keratoses
4. Osteopathia striata
5. Cutis marmorata
6. Show Explaination

135
77 Most common location of cutaneous 1. Cheeks E
lesions in neonatal lupus 2. Nose
erythrematosus is 3. Scalp
4. Perioral
5. Periorbital
6. Show Explaination
78 Which of the following is most likely 1. No additional abnormality A
associated? 2. Deafness
3. Coarctation of the aorta
4. Bifid rib
5. Alopecia
6. Show Explaination
79 What disorder is associated with a 1. Nail patella syndrome A
defect in LMX1B? 2. Steatocystoma
3. Monilithrix
4. Lhermitte-Duclos syndrome
5. Chediak-Higashi syndrome
6. Show Explaination
80 Which of the following is 1. Typically occurs in the 5th E
characteristic of Wooly Hair Nevus: decade
2. Ocular abnormalities may be
associated
3. There are no nevi associated
with this hair abnormality
4. Sponteneous improvement
never occurs
5. It is hereditary
6. Show Explaination
81 Which of followings syndromes 1. Leiner's disease A
characterized by Seborrheic-like or 2. Hyperimmunoglobulinemia E
exfoliative dermatitis syndrome
3. Wiskott-Aldrich syndrome
4. Chronic granulomatous
disease
5. X-linked
agammaglobulinemia
6. Show Explaination
82 Which of the following is not a major 1. Fever >5 days C
criterion for Kawasaki�s disease: 2. Palmoplantar erythema >
desquamation
3. Cardiac aneurysm
4. Strawberry tongue/ red lips
5. Cervical adenopathy
6. Show Explaination
83 Trichothiodystrophy classically 1. Trichostasis B
includes which of the following 2. Trichoschisis
findings: 3. Pili torti
4. Trichorrhexis invaginata
5. Trichorrhexis nodosa
6. Show Explaination
84 The most common age group for 1. Newborns D
papular-purpuric gloves and socks 2. Toddlers
syndrome is: 3. 6-10 year olds
4. Adolescents
5. Elderly
6. Show Explaination
85 Which of the following is the most 1. Seizure disorder E
common complication associated 2. Atrial septal defect
with cutis marmorata telangectatica 3. Systemic lupus
congenita? erythematosus

136
4. Hypercalcemia
5. Limb hypertrophy or atrophy
6. Show Explaination
86 A 4 month-old with diffuse blisters 1. Mupirocin ointment B
and erosions has a skin biopsy 2. Polymyxin B ointment
diagnostic of generalized 3. Petrolatum
mastocytosis. Which topical dressing 4. Neomycin ointment
should be avoided in this patient? 5. Silver sulfadiazine
6. Show Explaination
87 The Carney complex is associated 1. PRKAR1A A
with a defect in: 2. LYST
3. PTEN
4. MASH2
5. MLH1
6. Show Explaination
88 The association of Port-wine stains 1. Bannayan-Riley-Ruvalcaba D
on a limb with soft tissue swelling syndrome
with or without bony over growth is: 2. Goldenhar's syndrome
3. Sturge-Weber syndrome
4. Klippel-Trenaunay syndrome
5. Proteous syndrome
6. Show Explaination
89 A 2 year-old has a high fever for 1. Measles E
three days, as the fever breaks, a 2. Mumps
generalized rash appears on the 3. Rubella
trunk. The most likely diagnosis is: 4. Erythema infectiosum
5. Roseola
6. Show Explaination
90 An infant with failure to thrive has 1. Gaucher�s disease C
multiple xanthomas on skin exam 2. Tay-Sach�s disease
and foamy histiocytes on bone 3. Niemann-Pick disease
marrow biopsy. Your diagnosis is: 4. Fabry�s disease
5. Hunter�s syndrome
6. Show Explaination
91 What is the function of the gene 1. Gap junction protein D
which is defective in ataxia- 2. Cross-linking of structural
telangiectasia? proteins in the protein and
lipid envelope of the upper
epidermis
3. Pathway of cholesterol
biosynthesis
4. DNA repair protein
5. Tumor supressor protein
6. Show Explaination
92 A 24 month-old infant presents with 1. CD1-, S100- cells with C
yellowish-brown, crusted papules reniform nuclei
with petechiae in a seborrheic 2. Foamy histiocytes with
distribution. A biopsy is done to Touton giant cells
confirm a diagnosis. Which histologic 3. CD1+, S100+ cells with
picture is most likely? reniform nuclei
4. Mixed cellular infiltrate in a
�ball and claw� pattern
5. Superficial perivascular
infiltrate with mild spongiosis
and neutrophil containing
scale crust
6. Show Explaination
93 Which of the following is the most 1. Macrocephaly C
common long term sequelae from 2. Saber shins
congenital rubella syndrome? 3. Deafness

137
4. Nystagmus
5. Microcephaly
6. Show Explaination
94 Which of the following is not true 1. It is associated with viral E
about Gianotti-Crosti? infections like enterocirus,
EBV, and CMV
2. It typically affects children
between the age of 3 months
and 15 years
3. It is characterized by
monomorphic symmetric flat
topped papules in acral areas
4. It may have associated
fevers, lymphadenopathy,
and diarrhea
5. Corticosteroids should be
given to alleviate the
pruritus of the lesions
6. Show Explaination
95 GLUT-1 stains placental tissue. What 1. capillary vascular D
other tissue shows positive GLUT-1 malformation
staining? 2. salmon patch
3. glomus tumor
4. infantile hemangioma
5. angel's kiss
6. Show Explaination
96 What is the most likely diagnosis: 1. Epidermolysis bullosa A
simplex
2. Epidermolysis bullosa
dystrophica
3. Pemphigus vulgaris
4. Linear bullous IgA disease
5. Facticial dermatosis
6. Show Explaination
97 An infant presents with yellowish- 1. This presentation is B
brown, crusted papules with consistent with Jacquet's
petechiae in a seborrheic dermatitis
distribution. Which of the following 2. CD1+, S100+ cells with
statements about this entity is likely comma-shaped nuclei should
to be true? be seen on biopsy
3. This presentation occurs
when the infant is weaned
off of breast milk
4. An autosomal recessive
defect in holocarboxylase
synthetase is the cause
5. Maternal-fetal transmission
most likely occured in the
peripartum period
6. Show Explaination
98 Subcutaneous fat necrosis of the 1. Aspirin D
newborn is usually treated with: 2. Topical calcipitriol
3. Excision
4. Observation
5. Retinoids
6. Show Explaination
99 A 2 week-old infant is brought to the 1. 5% C
ER with a rash on her face. She is 2. 10%
found to have a 3rd degree heart 3. 25%
block. What is the risk that a second 4. 50%

138
child born to this mother will have 5. 100%
the same diagnosis? 6. Show Explaination
10 A 3 month old baby girl is brought to 1. Hypertension C
0 your office for consultation. You note 2. Hypercalcemia
an infantile hemangioma on the tip 3. Hypoglycemia
of the nose. Imaging reveals no 4. Tachycardia
underlying structural anomalies in 5. Hypernatremia
the cerebro-vasculature. You discuss 6. Show Explaination
starting propranolol. What side effect
do you need to monitor for?
10 Individuals with which of the 1. Tay Syndrome D
1 following syndromes 2. Rothmund-Thomson
characteristically present with Syndrome
photosensitivity, mental retardation, 3. Hutchinson-Gilford Progeria
a "wizened" appearance, "bird- Syndrome
headed" facies, and "Mickey Mouse" 4. Cockayne Syndrome
ears? 5. Werner Syndrome
6. Show Explaination
10 Which of the following is a potential 1. Cholestrol clefts in fat cells B
2 complication of subcutaneous fat 2. Hypercalcemia
necrosis of the newborn? 3. Acute renal failure
4. Hepatitis
5. Elevated uric acid levels
6. Show Explaination
10 The genetic disorder depicted in this 1. MASH2 B
3 photo is caused by a mutation in: 2. STK11
3. PTEN
4. Patched
5. TSC1
6. Show Explaination
10 Which of the following is a ssDNA 1. Herpesvirus B
4 virus: 2. Parvovirus
3. Picornovirus
4. Adenovirus
5. Parapox
6. Show Explaination
10 What is the diagnosis? 1. Transient neonatal pustular C
5 melanosis
2. Acropustulosis of infancy
3. Neonatal cphalic pustulosis
4. Miliaria
5. Erythema toxicum
neonatorum
6. Show Explaination
10 A full term newborn develops 1. Calcium A
6 erythematous, indurated plaques on 2. Complete blood count
the upper back. Which of the 3. Blood culture
following tests should be performed? 4. Thyroid function test
5. Alkaline phosphatase
6. Show Explaination
10 The differential diagnosis of zinc 1. Granuloma gluteale infantum A
7 deficiency is least likely to include: 2. Biotin deficiency
3. Multiple carboxylase
deficiency
4. Cystic fibrosis
5. Holocarboxylase synthetase
deficiency
6. Show Explaination
10 Which of the following is a sign of 1. Clutton's joints E
8 EARLY postnatal congenital syphilis? 2. Higoumenaki's sign

139
3. Hutchinson's teeth
4. Saber shins
5. Wimberger's sign
6. Show Explaination
10 The most likely etiology of 1. Candida E
9 Jacquet�s diaper dermatitis is: 2. Trichophyton rubrum
3. Group A beta-hemolytic
streptococcus
4. Herpes simplex virus, Type 2
5. Multifactorial
6. Show Explaination
11 Most common malignancy 1. Acute myelogenous leukemia B
0 associated with multiple lesions 2. Chronic myelogenous
similar to the attached image is: leukemia
3. Acute lymphocytic leukemia
4. Chronic lymphocytic
leukemia
5. Melanoma
6. Show Explaination
11 Which of the following diseases with 1. Wiskott-Aldrich syndrome A
1 immunodeficiency has an increased 2. Chronic granulomatous
risk of lymphoreticular malignancy? disease
3. Job syndrome
4. Severe combined
immunodeficiency syndrome
5. Leiner�s disease
6. Show Explaination
11 Ankyloblepharon filiforme adnatum is 1. Plakophilin E
2 seen with a defect in: 2. Plakoglobin
3. Desmoglein
4. C-kit
5. p63
6. Show Explaination
11 An infant presents with red-purple, 1. Granuloma gluteale infantum A
3 granulomatous nodules occurring in 2. Langerhans cell
the diaper area. The etiology is histiocytosisc.
secondary to local irritation, 3. Seborrheic dermatitis
maceration and Candida albicans. 4. Biotin deficiency
What is the most likely diagnosis? 5. Psoriasis
6. Show Explaination
11 A newborn presents with a well- 1. MRI D
4 defined, shiny patch with complete 2. Calcium
alopecia on the vertex of the scalp 3. Skin biopsy
along the suture lines. Which of the 4. Skull x-ray
following is the first step in 5. Fungal culture
diagnosis? 6. Show Explaination
11 Rhinorrhea, condylomata lata, and 1. Rubella D
5 mucous patches are all seen with 2. Toxoplasmosis
which congenital disorder? 3. Herpes simplex virus
4. Syphilis
5. Human papillomavirus
infection
6. Show Explaination
11 Which enanthem is most commonly 1. Koplik spots D
6 seen in association with Exanthem 2. Red strawberry tongue
subitum? 3. Chapped lips; dry, red
mucosa
4. Red macules and streaks on
the soft palate
5. Palatal erosions

140
6. Show Explaination
11 The disorder caused by a defect in 1. At birth B
7 intestinal zinc-specific transporter 2. Upon weaning from breast
SLC39A4 is most likely to present: milk
3. Upon weaning from formula
4. In childhood
5. In adulthood
6. Show Explaination

10. Cutaneous Manifestations of Systemic Disease


# Question Answers Corre
ct
Answ
er
1 The patient demonstrated in figure 6 1. LDL receptor deficiency C

141
would have: 2. Decreased chylomicrons
3. Elevated triglycerides
4. Decreased LDL
5. Decreased triglycerides
6.
2 Which of the following is true 1. It is caused by a mutation in E
regarding piebaldism? the GJB2 gene
2. It is caused by defective
metabolism of phytanic acid
3. It is caused by a defect in a
protein subunit of a kinase that
activates NFkappaB
4. It is caused by a deficiency of
fatty aldehyde dehydrogenase
5. It is caused by a mutation in
the proto-oncogene c-KIT
6.
3 What is the most common internal 1. Hepatitis C D
cause of intractable pruritus? 2. Hypothyroidism
3. Hyperthyroidism
4. Chronic renal failure
5. Internal malignancy
6.
4 Porphyria cutanea tarda may be 1. Hepatitis C virus infection E
associated with all of the following 2. Alcohol
except: 3. Estrogens
4. Polyhalogenated hydrocarbons
5. Inherited deficiency of
uroporphyrinogen III synthase
6.
5 Which of the following statements 1. MEN I is associated with E
about multiple endocrine neoplasia angiofibromas and collagenomas
(MEN) syndromes is FALSE? 2. MEN IIa is related to a defect
in RET proto-oncogene
3. MEN IIa and IIb are both
associated with medullary thyroid
carcinoma
4. MEN I, IIa and IIb are all
inherited in an autosomal
dominant fashion
5. MEN IIa is associated with
multiple mucosal neuromas
6.
6 A 50-year old woman with a history 1. Renal Cell Carcinoma A
of spontaneous pneumothorax 2. Gastric Carcinoma
develops multiple firm, skin colored 3. Breast Carcinoma
lesions on her face and neck over a 4. Ovarian Carcinoma
period of several years. This patient 5. Lung Carcinoma
should have periodic surveillance for 6.
the development of:
7 Which of the following is true 1. Erosive mucosal lichen planus D
regarding cutaneous associations has a weaker association with HCV
with hepatitis C virus (HCV) infection? than does cutaneous lichen planus
2. Pruritus in the setting of
chronic HCV infection is generally
correlated with elevated bile salt
levels in the setting of liver failure
3. Polyarteritis nodosa is
associated with HCV infection but
not hepatitis B virus infection

142
4. HCV-related porphyria cutanea
tarda may be caused by
decompartmentalization of iron
stores and resultant oxidation of
uroporphyrinogen decarboxylase
5. Cutaneous reactions to HCV
treatment are less common with
interferon/ribavirin combination
treatment than with treatment with
interferon alone
6.
8 Which of the following statements 1. Elevated uroporphyrins are B
regarding porphyrias is TRUE? found in the red blood cells of
hepatoerythropoietic porphyria
2. Delta aminolevulenic acid is
the only oxidized porphyrin
3. Acute intermittent porphyria is
the most common form of
porphyria
4. Griseofulvin is safe for those
with variegate porphyria
5. Plasma fluoresces at 410 nm
in patients with variegate
porphyria
6.
9 Which of the following laboratory 1. Neutrophilia D
abnormalities is most common in 2. Elevated amylase
patients with cholesterol emboli? 3. Hypercalcemia
4. Eosinophilia
5. Anemia
6.
10 A patient presents with episodic 1. Change of urine to green color B
flushing of the face and neck, upon addition of nitrosonaphthol
abdominal pain, wheezing, cough, 2. Elevated urinary 5-
and diarrhea. Carcinoid syndrome is hydroxyindolacetic acid
suspected. Which of the following 3. Elevated 24-hour urine
tests would confirm the diagnosis? norepinephrine
4. Decreased 24-hour urine
vanilylmandelic acid
5. Elevated serum tryptase
6.
11 A young African American patient 1. Atherosclerotic disease B
presents with anemia and 2. Sickle cell anemia
spontaneously appearing leg ulcers 3. Factitial dermatitis
over both lateral and medial malleoli. 4. Lupus erythematosus
The most likely diagnosis is: 5. Trauma
6.
12 When metastases to the skin occur 1. Medullary carcinoma B
from a thyroid malignancy, they are 2. Papillary adenocarcinoma
usually due to: 3. Follicular carcinoma
4. Anaplastic carcinoma
5. Cutaneous metastases have
not been reported in association
with thyroid malignancies
6.
13 The organism that causes this 1. T. tonsuran C
infection shown in the figure is: 2. C. immitis
3. C. albicans
4. P. aeruginoas
5. T. verrucosum

143
6.
14 Which of the following is true about 1. typically appears in adulthood C
blue rubber bleb nevus syndrome? 2. the venous malformations
regress with time
3. characterized by compressible
blue nodules 0.1 to 5 cm in size.
4. the nodules are pruritic
5. the gastrointestinal lesions are
typically located in the stomach
6.
15 Hypothyroidism may result in all of 1. Increase in the percentage of E
the following cutaneous findings telogen hairs
except: 2. Yellowish hue
3. Purpura
4. Madarosis
5. All of these answers are
correct
6.
16 Each of the following demonstrates a 1. Granuloma faciale C
vasculitis except: 2. Henoch-Schoenlein purpura
3. Type 1 cryoglobulinemia
4. Wegener�s granulomatosis
5. Leukocytoclastic vasculitis
6.
17 Which of the following statements 1. There is a low incidence of B
about necrolytic migratory erythema metastasis of the offending tumor
is TRUE? at the time of diagnosis
2. Acanthosis and parakeratosis
are found on routine histology
3. Serum glucagon levels are
usually normal
4. Vacuolar changes are normally
found on routine histology
5. The offending tumor originates
from endocrine argentaffin cells
6.
18 Which of the following syndromes 1. Turcot A
necessitates a work-up for colon 2. Birt-Hogg-Dube
cancer? 3. LAMB
4. Muckle Wells
5. Wells
6.
19 A 20-year-old male develops an 1. Montgomery's syndrome A
eruption of 100's of red-brown 2. Benign cephalic histiocytosis
yellowish papules with involvement 3. Rosai-Dorfman disease
of the mucous membranes. He has 4. Necrobiotic xanthogranuloma
no lymphadenopathy. The most likely 5. Multicentric
diagnosis is: reticulohistiocytosis
6.
20 All of the following are true regarding 1. It is an IgG mediated small A
Henoch-Schonlein Purpura EXCEPT: vessel vasculitis
2. Direct immunofluorescence of
lesional and peri-lesional skin will
demonstrate C3 and fibrin deposits
in small vessel wall
3. It is often preceded by an
upper respiratory infection
4. It may be complicated by
intussusception
5. It is self-resolving

144
6.
21 Which of the following is NOT 1. Ovarian cancer in women E
normally associated with 2. Psoriasiform dermatitis of the
dermatomyositis? scalp
3. Elevated aldolase levels
4. Cuticular dystrophy
5. Testicular cancer in men
6.
22 One might see all of the following 1. Elevated liver function tests C
laboratory and clinical abnormalities 2. Positive rheumatoid factor
in cryoglobulinemia associated with 3. Elevated C3 levels
hepatitis C virus infection EXCEPT: 4. Acrocyanosis
5. Urticarial plaques
6.
23 Regarding carcinoid syndrome, which 1. Symptoms are caused by C
of the following is TRUE? metastases to the skin
2. The most common location for
the tumor is the rectum
3. Somatostatin is a preferred
treatment
4. VMA is elevated in the urine
5. The tumors should not be
removed
6.
24 A patient develops cold-exacerbated 1. Lupus Erythematosus A
dusky acral plaques consistent with 2. Dermatomyositis
pernio. The systemic condition most 3. Acquired Immunodeficiency
likely to manifest such lesions is: Syndrome
4. Hepatitis C
5. Diabetes Mellitus
6.
25 All of the following disorders have an 1. Rothmund Thompson B
increased risk of systemic syndrome
malignancy except 2. Cockayne syndrome
3. Bloom syndrome
4. Werner syndrome
5. Xeroderma pigmentosum
6.
26 An uncommon complication of 1. Acute generalized C
treatment with potassium iodide is: exanthematous pustulosis
2. Erythema nodosum
3. Wolff-Chiakoff effect
4. Exacerbation of lichen planus
5. Hyperhidrosis
6.
27 All of the following are true regarding 1. Perilesional direct B
the condition pictured EXCEPT: immunofluroescence shows
granular IgA in the dermal papillae
and at the dermoepidermal
junction
2. Only 20% of patients have a
gluten-sensitive enteropathy
3. It is associated with HLA-DQ2,
HLA-DR3, and HLA-B8
4. It is associated with
Hashimoto's thyroiditis
5. Cutaneous findings are due to
autoantibodies to epidermal
transglutaminase
6.

145
28 What is the most common primary 1. Liver C
site of a carcinoid tumor? 2. Stomach
3. Appendix
4. Duodenum
5. Ileum
6.
29 All of the following statements are 1. Diabetes or glucose E
true regarding this condition EXCEPT: intolerance is found in 20% of
these patients
2. This condition may be
associated with cutaneous
anesthesia, hypohidrosis, and
partial alopecia
3. There is no impact of tight
glucose control on the likelihood of
developing this condition
4. 0.3-3% of diabetics have this
skin condition
5. This condition is associated
with increased dermal mucin
6.
30 A patient with gastric cancer 1. Numerous wart-like lesions on A
develops acanthosis nigricans and a the dorsal hands and wrists
sudden eruption of numerous warty 2. Follicular spicules on the nose
stuck-on papules on the trunk. What 3. Carpal tunnel syndrome
other finding may be seen? 4. Periorbital pupura
5. Migratory thrombophlebitis
6.
31 A patient presents with a complaint 1. Chest and abdominal/pelvic CT D
of facial flushing that spreads to the scanning should be the next step
neck and upper trunk. Review of in this patient's evaluation
systems reveals that the patient has 2. The patient may have an
occassional bouts of diarrhea, and associated sclerodermoid-like
intermittent bronchospasm. eruption on examination
Laboratory testing reveals an 3. The patient likely has a
elevated urine 5-hydroxyindole-acetic neoplasm originating in the
acid level. Which of the statements endocrine argentaffin cells
regarding this condition is NOT 4. Treatment with
correct? cyproheptadine would be
contraindicated
5. The patient likely has a
neoplasm located in the GI tract
6.
32 A patient presents with plane/palmar 1. familial lipoprotein lipase D
xanthomas. The most likely genetic deficiency
disorder would be: 2. Familial hypertriglyceridemia
3. Familial hypercholesterolemia
4. Familial
dysbetalipoproteinemia
5. Cerebrotendinous
xantomatosis
6.
33 Mutations in the STK11 gene 1. Muir-Torre Syndrome E
encoding a serine threonine kinase 2. Bannayan-Riley-Ruvalcaba
are seen in: Syndrome
3. Birt-Hogg-Dubbe Syndrome
4. Cronkhite-Canada Syndrome
5. Peutz-Jeghers Syndrome
6.
34 A patient presents with mild mental 1. This syndrome is inherited in E

146
retardation, infertility, joint an autosomal recessive manner
contractures, short stature, 2. If photosensitivity is a feature,
ichthyosis, and sparse hair with gonad size may be normal
trichoschisis. All of the following are 3. Patients may have associated
true regarding this patient's condition cataracts
EXCEPT: 4. The syndrome is caused by
impaired nucleotide excision repair
5. Perifoveal glistening white
dots are a feature
6.
35 Which of the following is NOT 1. Lentigines C
associated with Cronkhite-Canada 2. Adenomatous gastrointestinal
syndrome? polyps
3. Fibrocystic breast disease
4. Onycholysis
5. Weight loss
6.
36 Which of the following cutaneous 1. Honeycombed diffuse A
findings are characteristic for palmoplantar keratoderma,
Vohwinkles Syndrome? pseudoainhum with
autoamputation, star-shaped
keratosis over knuckles, nail
dystrophy, and alopecia
2. Dense depigmented lusterless
hair, pili torti, doughy skin, diffuse
cutaneous hypopigmentation
3. Transient erythroderma at
birth, palmoplantar keratoderma,
follicular hyperkeratosis, scarring
alopecia, dystrophic nails
4. Rapidly progressive alopecia
of all hair-bearing areas,
onycholysis, onychoschizia,
onychomadesis, hyperpigmented
macules on extremities
5. Oral papillomatosis,
palmoplantar keratoses, acral
keratoses, lipomas, hemangiomas,
scrotal tongue
6.
37 A 2 year old child is evaluated for 1. neurofibromas A
suspected diagnosis of 2. macrocephaly
neurofibromatosis. Which of the 3. pigmented iris hamartomas
following diagnostic findings is 4. seizures
typically absent on exam in this age 5. cafe au lait macules
group? 6.
38 Hyperkeratotic follicular nasal 1. Multiple myeloma A
papules have been described as a 2. Castleman's tumor
paraneoplastic sign in the setting of 3. AML
which neoplasm? 4. Adenocarcinoma of the lung
5. Renal cell carcinoma
6.
39 This syndrome is cause by defects in 1. Bannayan-Riley-Rubalcaba D
the genes that code for tumor Syndrome
suppressor proteins hamartin and 2. Blue Rubber Bleb Nevus
tuberin. Syndrome
3. Cronkhite-Canada Syndrome
4. Bourneville's Disease
5. Cowden's Syndrome
6.

147
40 Which of the following is an 1. Waardenburg Syndrome D
autosomal recessive disorder 2. Sjorgen-Larsson Syndrome
characterized by pili torti and 3. Refsum Disease
deafness? 4. Bjornstad's Syndrome
5. Cockayne Syndrome
6.
41 Which type of porphyria is the 1. Erythropoietic protoporphyria C
autosomal recessive form of (EPP)
porphyria cutanea tarda? 2. Congenital erythropoietic
porphyria (CEP)
3. Hepatoerythropoietic
porphyria (HEP)
4. Variegate Porphyria (VP)
5. Acute Intermitent Porphyria
(AIP)
6.
42 All of the following are true regarding 1. Inheritance is autosomal E
Cockayne Syndrome EXCEPT: recessive
2. It is caused by an inability to
repair cyclobutane dimers
3. It is associated with basal
ganglia calcifications
4. It is associated with retinal
pigment degeneration with a "salt
and pepper" appearance
5. None of these answers are
incorrect
6.
43 Which of the following is more 1. Pyoderma gangrenosum A
commonly associated with ulcerative 2. Oral lesions
colitis as compared to Crohn�s 3. Polyarteritis nodosa
disease? 4. Fistulae
5. Metastatic lesions
6.
44 A 6 month-old has a verrucous 1. An ophthalmologist C
plaque on the mucosal surface of the 2. A gastroenterologist
lower lip. Skin biopsy is consistent 3. A neurologist
with Riga-Fede disease. You should 4. A hematologist
refer the patient to: 5. An otolarngologist
6.
45 Carotenemia can be a manifestation 1. Porphyria B
of: 2. Hypothyroidism
3. Pretibial myxedema
4. Graves disease
5. Amyloidosis
6.
46 Patients with plexiform neuroma and 1. Juvenile chronic myelogenous A
NF I who also have JXG are at leukemia
increased risk for developing: 2. Non-Hodgkin's Lymphoma
3. Esophageal cancer
4. Breast cancer
5. Medullary carcinoma of the
thyroid gland
6.
47 The most common autoimmune 1. Autoimmune thyroiditis A
disease associated with hepatitis C 2. Myasthenia gravis
is: 3. Aplastic anemia
4. Lymphocytic sialadenitis
5. Rheumatoid arthritis
6.

148
48 Patients with Werner�s syndrome 1. Hypertrophic cardiomyopathy C
typically experience which of the 2. Aortic aneurysms
following types of cardiac disease? 3. Premature atherosclerosis
4. Cardiomegaly
5. Mitral valve prolapse
6.
49 Regarding paraneoplastic 1. Desmoplakin, one of the C
pemphigus, which of the following is molecular antigens, has a
TRUE? molecular weight of 190kd
2. Metastatic squamous cell
carcinoma of the skin is a common
cause
3. Granular C3 deposition is
found at the dermoepidermal
junction on direct
immunofluorescence
4. Monkey esophagus is the
preferred substrate for indirect
immunofluorescence
5. Non-Hodgkins lymphoma is
rarely associated
6.
50 Regarding eruptive xanthomata, 1. They are associated with type D
which of the following is TRUE? II and III hyperlipidemias
2. They are associated with
calcium channel blockers
3. There is no association with
ethanol consumption
4. They are associated with type
I, IV, and V hyperlipidemias
5. They favor the flexor surfaces
of the extremities
6.
51 All of the following are seen more 1. Oral cobblestoning C
commonly in Crohn's disease than in 2. Polyarteritis nodosa
ulcerative colitis EXCEPT: 3. Pyostomatitis vegetans
4. Perineal fistulas
5. Perineal fissures
6.
52 In patients with mixed 1. Elevated rheumatoid factor A
cryoglobulinema associated with 2. + ANA
hepatitis C, the most likely laboratory 3. + p-ANCA
abnormality is: 4. Decreased cryoglobulins
5. Elevated hematocrit
6.
53 What is the most common 1. Breast cancer B
malignancy associated with this 2. Gastric cancer
condition at this location? 3. Thyroid cancer
4. Pancreatic cancer
5. Melanoma
6.
54 All of the following are true regarding 1. Men with this condition may E
the condition pictured EXCEPT: be at increased risk for lymphoma
2. Associated calcinosis cutis
may be seen in pediatric patients
3. Age-appropriate cancer
screening is recommended
4. May be associated with a
psoriasiform scalp dermatitis
5. Elevated aldolase is more

149
specific than CPK
6.
55 A 64-year old man develops yellowish 1. Monoclonal gammopathy A
periorbital plaques that occasionally 2. Positive antinuclear antibodies
ulcerate and heal with scarring. What 3. Elevated creatinine
is the most likely associated lab 4. Elevated triglycerides
finding? 5. Increased thyroid stimulating
hormone
6.
56 Which of the following is true 1. The Wolff-Chaikoff effect must E
regarding treatment of inflammatory be considered
dermatoses with potassium iodide? 2. Binding of excess organic
iodide in the thryoid gland may
occur
3. Thyroid hormone synthesis
may be inhibited
4. None of the answers are
correct
5. All of these answers are
correct
6.
57 Which of the following is 1. Approximately 20% of E
characteristic of diabetic skin? diabetics have necrobiosis lipoidica
diabeticorum (NLD)
2. The level of cleavage in
bullous diabeticorum is subcorneal
3. Candida tropicalis is the most
common cause of angular cheilitis
4. There is a well-established
association between deep
granuloma annulare (GA) and
diabetes
5. Yellow skin may occur in up to
10% of diabetics
6.
58 Which of the following diseases is 1. Porphyria Cutanea Tarda D
caused by an enzymatic defect that 2. Acute Intermittent Porphyria
occurs in the mitochondria? 3. Congenital Erythropoeitic
Porphyria
4. Erythropoeitic Protoporphyria
5. Hepatoerythropoetic Porphyria
6.
59 Squamous cell carcinoma is seen in 1. Gorlin syndrome E
which syndrome? 2. Rombo syndrome
3. Nicolau-Balus syndrome
4. Rasmussen syndrome
5. Bazex syndrome
(Acrokeratosis paraneoplastica)
6.
60 Tripe palms are a cutaneous 1. Renal carcinoma B
manifestation associated with which 2. Lung carcinoma
of the following malignancies? 3. Prostate carcinoma
4. Colon carcinoma
5. Pancreatic carcinoma
6.
61 Features of Cushing disease include 1. Facial plethora D
all of the following EXCEPT? 2. Striae
3. Hypertension
4. Suppression of corticotropin
occurs with administration of

150
dexamethasone
5. Hirsutism
6.
62 A 70-year old male develops 1. Gastric Carcinoma A
hyperpigmented velvety plaques on 2. Lung Carcinoma
his lips, dorsal hands and feet, and in 3. Ovarian Carcinoma
his axilla. What underlying 4. Lymphoma
malignancy is most commonly 5. Breast Carcinoma
associated with this finding? 6.
63 Which of the following is a 1. Hypertrichosis lanuginosa A
paraneoplastic disease most often acquisita
associated with lung carcinoma? 2. Dermatomyositis
3. Acanthosis nigricans
4. Paraneoplastic pemphigus
5. Erythroderma
6.
64 A 63-year-old male develops small, 1. 20-25% A
non-tender, violaceous papules on 2. 5-10%
his dorsal hands, face, ears, and 3. 65-70%
trunk. The patient also develops a 4. 50-55%
destructive arthropathy with finger 5. 85-90%
deformities. What percentage of 6.
patients with this condition has an
associated malignancy?
65 Hepatitis C infection is associated 1. Gianotti-Crosti syndrome B
with: 2. Mixed cryoglobulinemia
3. Pityriasis rosea
4. Kapsi�s sarcoma
5. Oral hairy leukoplakia
6.
66 A patient presents with diffuse waxy 1. This condition may be B
keratoderma of the palms and soles associated with esophageal
as well as oral hairy leukoplakia. carcinoma
Which of the following statements 2. This condition is autosomal
regarding this condition is NOT true? recessive
3. Family members should be
advised to undergo cancer
screening
4. Features may include
squamous cell carcinomas arising
from keratodermic skin
5. None of these answers are
correct (all are true)
6.
67 Which of the following may be 1. Dermatitis herpetiformis A
associated with Graves� disease? 2. Geographic tongue
3. Hypohidrosis
4. Madarosis
5. Thick, pale lips
6.
68 This autosomal dominant condition is 1. POEMS syndrome C
characterized by trichodiscomas, 2. Rombo syndrome
fibroepithelial polyps, and 3. Birt-Hogg-Dube syndrome
fibrofolliculomas: 4. Proteus syndrome
5. Cowden's disease
6.
69 Which of the following skin findings is 1. Pityriasis lichenoides D
most closely linked to hepatocellular 2. Pityriasis alba
carcinoma as a paraneoplastic 3. Pityriasis amiantacea
syndrome? 4. Pityriasis rotunda

151
5. Pityriasis rosea
6.
70 A patient with gluten-sensitive 1. Granular IgA at the A
enteropathy presents with vesicles dermoepidermal junction on direct
on the extensor surfaces of the immunofluorescence
extremities. What findings are most 2. Linear C3 and IgG at the
likely on a perilesional biopsy? dermoepidermal junction on direct
immunofluorescence
3. Linear IgA surrounding vessels
on direct immunofluorescence
4. Pautrier�s micro-abscesses in
the epidermis on H & E
5. Leukocytoclastic vasculitis
6.
71 An end stage AIDS patient with 1. Argyria E
tuberculosis presents with diffuse 2. Lymphoma
hyperpigmentation of both sun- 3. Tinea versicolor
exposed and unexposed areas. The 4. Kaposi's sarcoma
palmer creases are markedly 5. Addison disease
hyperpigmented and the patient is 6.
very ill. The most likely diagnosis is:
72 Terry's nails can most likely be seen 1. Cirrhosis A
in which of the following conditions? 2. Crohn's disease
3. End-stage renal disease
4. Hidradenitis suppurativa
5. Hyperthyroidism
6.
73 The peak sensitivity to prophyrins 1. 220-290nm D
occurs at which wavelengths? 2. 290-320nm
3. 320-400nm
4. 400-410nm
5. 410-450nm
6.
74 Patients with this syndrome are at 1. Bourneville's Disease E
increased risk for developing 2. Nail-Patella Syndrome
Lhermite-Duclos disease: 3. MEN Type IIA
4. Fabry's Disease
5. Cowden's Syndrome
6.
75 Which of the following statements 1. MEN Type IIB is also known as D
regarding multiple endocrine Sipple's Syndrome
neoplasia syndromes is true? 2. MEN Type IIA is also known as
Wermer's Syndrome
3. Multiple mucosal neuromas
are seen in association with MEN
Type IIA
4. Lichen or macular amyloidosis
is seen in association with MEN
Type IIA
5. Patients with MEN Type IIB are
at increased risk for developing
follicular thyroid carcinoma
6.
76 Which of the following is NOT true 1. Urine cortisol levels are B
regarding Cushing's Syndrome? elevated
2. Corticotropin is suppressed
with the administration of
dexamethasone
3. It may be associated with an
underlying oat cell lung carcinoma

152
4. It may be associated with
hypertension and hypokalemia
5. May present with
hyperpigmentation and facial
plethora
6.
77 Which of the following is not a 1. Diarrhea C
feature of Cronkhite-Canada 2. Alopecia
syndrome? 3. Lung carcinoma
4. Dystrophic nails
5. Hyperpigmented macules
6.
78 Which of the following is NOT true 1. Patients with proximally- A
regarding calciphylaxis? located lesions have a better
prognosis than those with acral
lesions
2. May be treated with
parathyroidectomy
3. May be present with retiform
purpura
4. Histologic findings include
medial calcification and intimal
hyperplasia of small arteries and
arterioles
5. Has an associated mortality of
60-80%
6.
79 With regards to patients with 1. Diabetic dermopathy is most D
diabetes mellitus and skin, which of commonly expressed on the
the following is most accurate? forearms and feet
2. Scleredema most often
presents on the tibial surfaces
3. Scleredema is the most
common skin manifestation of
diabetes mellitus
4. Less than 5% of patients with
diabetes mellitus have necrobiosis
lipoidica
5. Less than 5% of patients with
necrobiosis lipoidica have diabetes
mellitus
6.
80 Which of the following is FALSE 1. It is inherited in an X-linked C
regarding Fabry�s disease? recessive fashion
2. It is associated with acral
parasthesias
3. It is inherited in an X-linked
dominant fashion
4. It may be associated with
renal failure
5. The etiology is a defect in
alpha-galactosidase A
6.
81 Which one of the following clinical 1. Sudden eruption of seborrheic B
findings suggests the diagnosis of keratoses
multiple myeloma? 2. Hyperkeratotic follicular
papules on the nose
3. Erythema, vesicles, and
erosions in periorifical and acral
areas

153
4. Concentric erythematous rings
with trailing scale on trunk and
extremities
5. Tripe palms
6.
82 A complication seen in women of 1. Atrial septal defect B
childbearing age with the condition in 2. Pulmonary
the figure is: lymphangioleiomyoma
3. Gastrointestinal bleeding
4. Lisch nodules
5. Pulmonary emboli
6.
83 Which of the following is (are) 1. Autosomal recessive D
characteristic of basal cell nevus inheritance
syndrome? 2. NEMO gene mutation
3. Tram-track calcifications
4. Colobomas
5. Telangiectasias
6.
84 What is the unique laboratory finding 1. elevated coproporphyrins B
in stool in porphyria cutanea tarda 2. elevated isocoproporphyrins
(PCT) and hepatoerythropoietic 3. decreased isocoproporphyrins
porphyria (HEP)? 4. elevated protoporphyrins
5. elevated uroporphyrins
6.
85 A 70-year old female develops 1. Squamous cell carcinoma of A
erythema with fine adherent scale on the larynx
acral skin that progresses to 2. Adenocarcinoma of the colon
keratoderma and eventually a more 3. Adenocarcinoma of the breast
generalized psoriasiform dermatitis. 4. Squamous cell carcinoma of
What is the most likely underlying the vagina
malignancy? 5. Medullary thyroid carcinoma
6.
86 Which of the following is TRUE about 1. Bullae are common on the E
diabetic-related skin disease? thighs
2. Yellow skin affects the majority
of diabetic patients
3. Diabetic dermopathy affects
the upper back
4. Necrobiosis lipoidica affects
approximately 20% of diabetics
5. Direct immunofluorenscence is
usually negative in bullous
diabeticorum
6.
87 The shoulder pad sign has been 1. Dermatomyositis C
described in which disease? 2. Cushings disease
3. Systemic amyloidosis
4. Nephrogenic fibosing
dermopathy
5. Systemic lupus erythematosis
6.
88 Regarding eruptive xanthomas, 1. They occur in the setting of A
which of the following is true? familial hyperlipidemia types I, IV,
and V
2. They occur in the setting of
familial hyperlipidemia types II and
III
3. Triglyceride levels are usually
below 500mg/dl

154
4. They are most commonly
found on the eyelids
5. They are not related to alcohol
consumption
6.
89 The presence of antibodies to c-ANCA 1. Ulverative colitis D
is characteristically seen in patients 2. Churg-Strauss disease
with which disease? 3. Polyarteritis nodosa
4. Wegener�s granulornatosis
5. Nodular vasculitis
6.
90 All of the following are true regarding 1. It is caused by a mutation in B
incontinentia pigmenti EXCEPT: the NEMO gene
2. It is an X-linked recessive
disorder
3. Inflammation and blistering
may be followed by hyperkeratotic,
verrucous lesions
4. It is associated with cerebellar
ataxia
5. It is associated with coloboma
and retinal detachment
6.
91 Paraneoplastic pemphigus is 1. Non-Hodgkins lymphoma B
associated with all of the following 2. Lung carcinoma
underlying malignancies EXCEPT: 3. Chronic lymphocytic leukemia
4. Thymoma
5. Castleman's tumor
6.
92 A patient infected with the hepatitis 1. Increased hyaluronic acid in E
C virus is at highest risk for which of dermis of lower and upper
the following cutaneous extremities
manifestations? 2. Lindsay�s nails
3. Congenital hyperpigmentation
of the retinal pigment (CHRPE)
4. Periorificial lentigines
5. Positive rheumatoid factor and
decreased complement (C3)
6.
93 A patient on hemodialysis presents 1. The diagnostic B
with indurated plaques having a peau histopathological findings include
d�orange texture on the bilateral acanthosis and hyperkeratosis
lower legs. Which of the following 2. Serum protein electrophoresis
statements is TRUE? should be performed
3. Dapsone will likely be effective
treatment
4. The face is usually is affected
5. The palms and soles are
usually affected
6.
94 The most common location of the 1. Face C
lesions in nephrogenic fibrosing 2. Palms and soles
dermopathy is: 3. Lower extremities
4. Back
5. Chest
6.
95 A 55 year old patient presents with 1. Peutz-Jeghers syndrome D
new onset brown macules on arms, 2. Cowden disease
legs, face and palms. She gives a 3 3. Ulcerative colitis
month history of diarrhea, abdominal 4. Cronkhite-Canada syndrome

155
cramps, weight loss and protein- 5. Plummer-Vinson syndrome
losing enteropathy. The most likely 6.
diagnosis is:
96 All of the following are true regarding 1. Seen in end-stage renal B
calciphylaxis except: disease
2. Vascular mural calcification
occurs late in the process
3. High morality rate is due to
sepsis
4. Association with
hyperparathyroidism
5. Association with an elevated
calcium: phosphate product
6.
97 All of the following statements 1. Type I is composed of A
regarding cryoglobulinemia are true monoclonal IgG and polyclonal IgM
EXCEPT: 2. Type II is composed of
polyclonal IgG and monoclonal IgM
3. Type III is composed of
polyclonal IgG and polyclonal IgM
4. 80% of cases of mixed
cryoglobulinemia are associated
with Hepatitis C infection
5. None of these answers are
correct (all statements are true)
6.
98 Which of the following porphyria 1. Dystrophic calcifications C
cutanea tarda associations has a 2. Estrogen levels
direct relationship to the level of 3. Sclerodermoid changes
urine uroporphyrins? 4. RBC fluorescence
5. Hypertrichosis
6.
99 Which of the following is NOT 1. Mixed cryoglobulinemia E
associated with Hepatitis C disease? 2. Uroporphyrinogen
decarboxylase deficiency
3. Single-stranded RNA viridae
4. Leukocytoclastic vasculitis
5. Single-stranded DNA viridae
6.
10 What is the treatment of choice for 1. Phlebotomy A
0 porphyria cutanea tarda? 2. Antimalarials
3. Erythropoietin
4. Oral iron supplementation
5. Naproxen
6.
10 A patient with congenital 1. An autosomal dominant C
1 hypertrophy of retinal epithelium is mutation in the MSH2 gene
most likely to have: 2. Pheochromocytoma
3. Adenomatous polyposis
4. Tram-track calcifications on
head radiograph
5. Peg-shaped teeth
6.
10 The expected histology of a biopsy 1. Increased mucin E
2 take from the lesion shown in the 2. Schumann bodies
image would 3. Granulomatous infiltrate
localized to the papillary dermis
4. Elastic fiber degeneration
5. Collagen degeneration
6.

156
10 Erythema gyratum repens is known 1. Lung carcinoma E
3 to be associated with all of the 2. Breast carcinoma
following malignancies except: 3. Cervical carcinoma
4. Bladder carcinoma
5. Gastric carcinoma
6.
10 Which of the following is true 1. Has a rapidly progressive but D
4 regarding nephrogenic fibrosing reversible course
dermopathy? 2. Is associated with a
paraproteinemia
3. Is associated with peripheral
eosinophilia
4. May be associated with
antiphospholipid antibodies
5. Is associated with a dramatic
increase in dermal mucin
6.
10 Which of the following statements 1. Fecal coproporphyrinogen is D
5 regarding porphyrias is FALSE? increased in variegate porphyria
2. Hepatoerythropoietic
porphyria is the homozygous form
of porphyria cutanea tarda
3. Acute intermittent porphyria
has no skin findings
4. Coproporphyrinogen is
elevated more than
uroporphyrinogen in 24 hour urine
samples in porphyria cutanea
tarda
5. In erythropoietic
protoporphyria, protoporphyrin IX
absorbs in the Soret band
6.
10 The clinical findings in figure 3 are 1. Graves disease A
6 associated with which disorder? 2. Myasthenia gravis
3. Dermatomyositis
4. Sarcoidosis
5. Rheumatoid Arthritis
6.
10 Which step is rate-limiting in the 1. Aminolevulinic acid synthase A
7 synthesis of protoporphyrinogen? 2. Aminolevulinic acid
dehydratase
3. Porphobillinogen deaminase
4. Coproporphyrinogen oxidase
5. Ferrochelatase
6.

157
11 . Disorders of the Hair and Nails
# Question Answers Corre
ct
Answ
er
1 Koilonychia is most commonly seen in 1. Renal disease D
association with: 2. Progressive systemic sclerosis
3. Alopecia areata
4. Impaired iron metabolism
5. Pulmonary disease
6. Show Explaination
2 The syndrome including mental and physical 1. Bazex Follicular Atrophoderma D
retardation, convulsions, episodic 2. Crandall's syndrome
unconsciousness, liver enlargement, skin 3. Citrullinemia
lesions, and dry and brittle hair showing 4. Argininosuccinic aciduria
trichorrhexis nodosa microscopically and 5. Bjornstad's syndrome
fluorescing red. is: 6. Show Explaination
3 Bj�rnstad syndrome is the combination of pili 1. Lentigines D
torti and which of the following features? 2. White forelock
3. Hypogonadism
4. Deafness
5. Seizures
6. Show Explaination
4 A 4 year-old boy has a single circular alopecic 1. Aplasia cutis congenita A
atrophic patch on the vertex of the scalp. This 2. Congenital syphilis
most likely is due to which of the following 3. Keratosis Follicularis Spinulosa
conditions? Decalvans
4. Morphea en coup de sabre
5. Triangular Alopecia
6. Show Explaination
5 Highest graft survival in hair transplantation is 1. 4-5 mm plugs D
achieved through the use of: 2. Minigrafts
3. Single hairs
4. Follicular units
5. Micrografts
6. Show Explaination
6 Argininosuccinic aciduria is characteristically 1. Pili trianguli et canaliculi D
associated with this hair finding �name the 2. Trichoschisis
condition pictured below: 3. Monilethrix
4. Trichorrhexis nodosa
5. Trichorrhexis invaginata
6. Show Explaination
7 Acute paronychia is most commonly caused by 1. Candida albicans D
which of the following organisms? 2. Candida tropicalis
3. Pseudomonas aeruginosa
4. Staphylococcus aureus
5. Trichophyton rubrum
6. Show Explaination
8 A patient with diffuse severe sudden hair loss 1. Anagen effluvium C
developing 3 months after hospitalization for 2. Catagen effluvium
septicemia likely has which of the following 3. Telogen effluvium
conditions? 4. Loose anagen syndrome
5. Uncombable hair syndrome
6. Show Explaination
9 Which genodermatosis characterized by broad 1. Rothmund-Thompson E
thumbs and this nail disorder pictured below? 2. Tuberous sclerosis
3. Pachyonychia congenita
4. Proteus syndrome
5. Rubenstein-Taybi syndrome
6. Show Explaination
10 The best diagnosis for this scarring form of 1. Chronic cutaneous lupus D
alopecia is erythrematosus
2. Lichen planopilaris

158
3. Peseudopelade of Broq
4. Perifolliculitis capitis abscedens et
suffodiens
5. Central centrifugal cicatricial
alopecia
6. Show Explaination
11 Endonyx onychomycosis is due to: 1. Trichophyton rubrum C
2. Trichophyton mentagrophytes
3. Trichophyton soudanense
4. Scytalidium dimidiatum
5. Scopuliaropsis
6. Show Explaination
12 A patient with diffuse hair loss developing after 1. Anagen effluvium A
a thallium scan likely has which of the 2. Catagen effluvium
following conditions? 3. Telogen effluvium
4. Loose anagen syndrome
5. Uncombable hair syndrome
6. Show Explaination
13 Air spaces in the hair shaft lead to this 1. Pili trianguli et canaliculi B
condition pictured below � name the 2. Pili annulati
condition: 3. Trichoschisis
4. Pohl Pinkus constriction
5. Wooly hair
6. Show Explaination
14 The defect associated with the hair finding 1. hHB6/hHB1 A
shown in the image is: 2. hHB7/hHB2
3. hHB8/hHB3
4. hHB9/hHB4
5. hHB10/hHB5
6. Show Explaination
15 Terry's nails are seen in which of the following 1. Renal failure C
conditions? 2. Pulmonary fibrosis
3. Diabetes
4. Bladder carcinoma
5. Aerodigestive carcinoma
6. Show Explaination
16 A patient with curly hair that straightened 1. Distal-less homeobox-3 gene (DLX3) A
after puberty, enamel hypoplasia, dental pits, 2. Vascular-endothelial growth factor
and increased bone density likely has a receptor 3
mutation in which of the following genes? 3. Bone morphogenetic protein type 2
4. SLURP 1
5. SPINK 5
6. Show Explaination
17 Proximal white subungual onychomycosis with 1. Blood count E
polydactylous involvement needs at least one 2. Sedimentation rate
laboratory test: 3. T-helper CD4 lymphocytes
4. Liver enzymes
5. HIV
6. Show Explaination
18 A patient has hyperkeratotic tissue on the 1. Onychophosis A
lateral and proximal nails folds. What is the 2. Onychomadesis
diagnosis? 3. Hapalonychia
4. Onychocryptosis
5. Onychophagia
6. Show Explaination
19 Black nail is caused by: 1. Trichophyton mentagrophytes B
2. Proteus mirabilis
3. Pseudomonas spp.
4. Staphylococcus aureus
5. Hortaea werneckii
6. Show Explaination
20 A child presents with sparse, fine hair, thin 1. P63 D
nails and loose skin in infancy. He has a pear- 2. DLX3
shaped, broad nose and on X-ray has cone- 3. RMRP
shaped epiphyses. Which of the following 4. TRPS1

159
genes is mutated in this syndrome? 5. ABCC6
6. Show Explaination
21 Which of the following syndromes does NOT 1. Crandall A
have the finding shown in the image as part of 2. Argininosuccinic aciduria
the spectrum? 3. Citrullinemia
4. Menke's Kinky Hair
5. Netherton
6. Show Explaination
22 Which part of the nail complex results in nail 1. Proximal nail fold B
pits? 2. Proximal nail matrix
3. Dorsal nail matrix
4. Nail plate
5. Nail bed
6. Show Explaination
23 What is the gene defect that causes red hair? 1. Tyrosinase C
2. P protein
3. Melanocortin 1 receptor
4. Tyrosine aminotransferase
5. Tyrosinase-related protein 1
6. Show Explaination
24 Signs of virilization include all of the following 1. Centripetal obesity A
EXCEPT: 2. Temporal balding
3. Clitoromegaly
4. Increased muscle mass in the limb
girdles
5. Acne
6. Show Explaination
25 Triangular lunulae are: 1. Part of the syndrome caused by a A
LMX1b mutation
2. Associated with hyperplastic patellae
3. Associated with a renal cell
carcinoma
4. Associated with a mutation of
ATP2C1
5. Associated with a mutation of
SERCA2
6. Show Explaination
26 A patient with wooly hair, diffuse palmoplantar 1. HHB6 B
keratoderma, and right ventricular 2. Plakoglobin
arrhythmogenic cardiomyopathy likely has a 3. Plakophilin
mutation in which of the following genes? 4. SPINK 5
5. SLURP 1
6. Show Explaination
27 A commonly known cause of splinter 1. Trichinosis E
hemorrhages in the nail is endocarditis. Which 2. Trauma
of the following would be unlikely to cause 3. Psoriasis
splinter hemorrhages? 4. Vasculitis
5. Rheumatoid arthritis
6. Show Explaination
28 A significantly increased number of "club 1. Anagen effluvium B
hairs" on a pull test is indicative of: 2. Telogen effluvium
3. Normal hair anatomy
4. Alopecia areata
5. Angrogenetic alopecia
6. Show Explaination
29 Which of the following is an example of an 1. Terry's lines C
apparent leukonychia? 2. Mee's lines
3. Muehrcke's lines
4. Hutchinson's lines
5. Onychoschizia
6. Show Explaination
30 A six year old girl presents after being sent 1. Pediculosis B
home from school for "lice". On examination, 2. Hair casts
there are structures on the hair which are 3. Trichoptilosis
freely slide along the hair shaft. What is the 4. Monilithrix

160
most likely diagnosis? 5. Pili annulati
6. Show Explaination
31 The best test for Cushing syndrome is: 1. Plasma LH E
2. Plasma testosterone
3. Plasma prolactin
4. Plasma DHEA
5. Overnight dexamethasone
suppresion test
6. Show Explaination
32 Yellow nails can be seen in each of the 1. Bronchiectasis B
following syndromes except: 2. Hyperlipidemia
3. Tobacco use
4. Thyroid disease
5. Penicillamine
6. Show Explaination
33 A patient with synophrys, dystopia canthorum, 1. Diffuse thinning E
and heterochromia iridum irides likely has 2. Scarring alopecia
which of the following hair abnormalities? 3. Trichorrhexis invaginata
4. Trichorrhexis nodosa
5. White forelock
6. Show Explaination
34 This hair finding is caused by mutations in 1. SPINK 5 A
what gene? 2. NEMO
3. SERCA2
4. Dyskerin
5. Keratin 16
6. Show Explaination
35 A subungual hematoma covering 50% of the 1. No treatment B
nailbed should be managed by which of the 2. Removal of the nail plate
following? 3. Trephination of the nail plate
4. Needle aspiration of the hematoma
5. Pressure dressing
6. Show Explaination
36 Acquired progressive kinking of hair is a 1. Antimetabolites C
complication of which of the following 2. Antimalarials
medications? 3. Retinoids
4. Slfonamides
5. Dapsone
6. Show Explaination
37 Which of the following syndromes does not 1. Lichen planus E
have Dorsal pterygium as a feature: 2. Lesch-Nyhan syndrome
3. Cicatricial pemphigoid
4. Chronic GVHD
5. Lamellar ichthyosis
6. Show Explaination
38 Muehrcke's nails are most often associated 1. Hyperthyroidism C
with: 2. Hypothyroidism
3. Hypoalbuminemia
4. Hyperalbuminemia
5. None of these answers are correct
6. Show Explaination
39 Choose the correct statement regarding white 1. Commonly occurs in the tropics E
piedra. 2. Consists of cream-colored colonies
attached to hairs
3. Treatment consists of cutting the hair
4. Is caused by piedra hortae
5. Both Consists of cream-colored
colonies attached to hairs and
Treatment consists of cutting the hair
6. Show Explaination
40 The most common enzyme abnormality in 1. 3-beta-hydroxysteroid C
congenital adrenal hyperplasia is: dehydrogenase isomerase
2. 11-beta-hydroxylase
3. 21-hydroxylase
4. 15-hydroxylase

161
5. 17-hydroxylase
6. Show Explaination
41 Primary total dystrophic onychomycosis is due 1. Candida spp A
to: 2. Fusarium
3. Trichophyton rubrum
4. Scytalidium dimidiatum
5. Scopulariopsis brevicaulis
6. Show Explaination
42 Spotted red lunulae are absent in which of the 1. Alopecia areata B
following conditions? 2. Keratosis follicularis
3. Systemic lupus erythematosus
4. Rheumatoid arthritis
5. Lichen planus
6. Show Explaination
43 Frontal bossing, saddle nose, hypoplastic 1. Christ Siemens Touraine syndrome A
midface, peg shaped or conical teeth and 2. Hidrotic ectodermal dysplasia
hypopigmented short sparse scalp and body 3. Arginosuccinic aciduria
hair are prominent features of which of the 4. Monilethrix
following 5. Pachyonychia congenita
6. Show Explaination
44 The "ragged cuticle" seen in dermatomyositis 1. Samitz sign A
is also known as: 2. Ventral pterygium
3. Plummers sign
4. Candy-cane nails
5. Emperipolesis
6. Show Explaination
45 The combination of poliosis, uveitis, deafness, 1. Piebaldism D
and vitiligo are characteristic of which of the 2. Waardernburg syndrome
following conditions? 3. Cronkhite-Canada syndrome
4. Vogt-Koyanagi-Harada syndrome
5. Cornelia de Lange Syndrome
6. Show Explaination
46 Deposition of mucin in the hair follicle can be 1. Keratosis Follicularis Spinulosa C
associated with which disease? Decalvans
2. Adams-Oliver Syndrome
3. Mycosis fungoides
4. Acne Keloidalis Nuchae
5. Perifolliculitis Capitis Abscedens et
Suffodiens
6. Show Explaination
47 Congenital generalized hypertrichosis is 1. Androgen-secreting ovarian tumors E
associated with which of the following 2. Pituitary insufficiency
symptoms? 3. Distichiasis
4. Methimazole teratogenicity
5. Gingival fibromatosis
6. Show Explaination
48 Which of the following is NOT a cause of red 1. Endocarditis A
lunulae? 2. Psoriasis
3. Carbon monoxide poisoning
4. Imuran
5. Rheumatoid arthritis
6. Show Explaination
49 Pili trianguli et canaliculi is the characteristic 1. Monilethrix E
hair fiding in which of the following disoders? 2. Pili annulati
3. Naxos syndrome
4. Anhidrotic ectodermal dysplasia
5. Uncombable hair syndrome
6. Show Explaination
50 Mee's lines are: 1. Double white transverse lines from D
abnormal vascular bed
2. Brownish macules beneath the nail
plate
3. Local or diffuse hyperkeratotic tissue
that develops on the lateral or
proximal nail folds

162
4. Transverse white lines that affect all
nails, grow out with nail growth
5. Vertical black lines on a single or
multiple nails
6. Show Explaination
51 Distal subungual onycholysis associated with 1. Candida spp C
paronychia is due to: 2. Fusarium spp
3. Scytalidium dimidiatum
4. Trichophyton rubrum
5. Trichophyton mentagrophytes
6. Show Explaination
52 Psoriatic onycholysis is caused by psoriasis in 1. Nail matrix C
which of the following nail subunits? 2. Nail plate
3. Nail bed
4. Proximal nail fold
5. Hyponychium
6. Show Explaination
53 Which of the following medication is a cause of 1. Dapsone B
hirsutism without virilization? 2. Diazoxide
3. Diazepam
4. Dinitrochlorobenzene
5. Dantrolene
6. Show Explaination
54 The visible portion of the nail matrix is called 1. Eponychium B
the: 2. Lunula
3. Cuticle
4. Hyponychium
5. Nail plate
6. Show Explaination
55 The combination of a low-set hairline and 1. Werner syndrome D
synophrys is seen in which of the following 2. Turner Syndrome
conditions? 3. Noonan Syndrome
4. Cornelia de Lange Syndrome
5. Waardenburg's syndrome
6. Show Explaination
56 The distal nail matrix is visible at the following 1. Onychodermal band C
portion of the nail unit: 2. Hyponychium
3. Lunula
4. Cuticle
5. Lateral nailfold
6. Show Explaination
57 Which of the following are findings associated 1. Glomerulonephritis A
with Nail-Patella Syndrome? 2. Alopecia
3. Bladder diverticuli
4. Photosensitivity
5. Hip dysplasia
6. Show Explaination
58 Which autoantibody is associated with these 1. Mi-2 A
cuticular changes, shawl sign, and overall 2. Jo-1
favorable prognosis? 3. High titered ANA with speckled
pattern
4. Scl-70
5. Ro
6. Show Explaination
59 Trichobezoar is is a potentially life-threatening 1. Alopecia areata C
complication of: 2. Anhidrotic ectodermal dysplasia
3. Trichotillomania
4. Discoid lupus
5. Lichen planopilaris
6. Show Explaination
60 Plummer's nails are associated with which of 1. Dermatomyositis D
the following disorders? 2. Systemic sclerosis
3. Lupus erythematosus
4. Hyperthyroidism
5. Lead poisoning

163
6. Show Explaination
61 Tay�s syndrome is characterized by this hair 1. Trichorrhexis nodosa C
condition pictured below � identify the 2. Trichorrhexis invaginata
condition: 3. Trichoschisis
4. Monilethrix
5. Pili trianguli et canaliculi
6. Show Explaination
62 Regarding androgens in women, which of the 1. Testosterone binds the androgen B
following statements is NOT correct? receptor
2. Eyebrows, eyelashes and vellus hairs
are androgen-dependent
3. The hair follicle requires conversion
of testosterone to
dihydrotestosterone for expression of
androgen action
4. Dihydrotestosterone binds the
androgen receptor
5. There are no differences in
eyelashes, eyebrows and vellus hair-
bearing areas in men and women
6. Show Explaination
63 A 24 year-old woman presents with the 1. Pheochromocytoma B
complaint of hirsutism and asks about therapy. 2. Polycystic ovarian disease
She has a virilization pattern of hair growth. 3. Congenital adrenal hyperplasia
On questioning, she relates having irregular 4. Ovarian carcinoma
menstrual periods and acne. Which of the 5. Normal variant
following is the most likely diagnosis? 6. Show Explaination
64 Lindsay's nails (distal nail normal, proximal nail 1. Chronic renal failure A
white) is characteristic of: 2. Plummer-Vinson syndrome
3. Hemochromatosis
4. Ectodermal dysplasia
5. LEOPARD syndrome
6. Show Explaination
65 Yellow nail syndrome has been associated with 1. D-penicillamine A
use of which of the following treatments for 2. Methotrexate
Rheumatoid Arthritis? 3. Infliximab
4. Etanercept
5. Adalimumab
6. Show Explaination
66 Mutations in c-kit are associated with which of 1. Waardenburg syndrome B
the following conditions? 2. Piebaldism
3. Tuberous sclerosis
4. Vogt-koyanagi-harada
5. Cornelia de Lange Syndrome
6. Show Explaination
67 An 18 year-old girl who was hospitalized last 1. Twenty-nail dystrophy B
month after a serious car accident is noted to 2. Beau�s lines
have white transverse grooves on her 3. Half-and-half nails
fingernails and toenails. The most likely 4. Mees� lines
diagnosis is: 5. Terry�s nails
6. Show Explaination
68 Triangular lunulae are seen in what disorder? 1. Dyskeratosis congentia D
2. Neurofibromatosis II
3. Gorlin�s syndrome
4. Nail-patella syndrome
5. Papillon-Lefevre syndrome
6. Show Explaination
69 Ventral pterygium pictured below is 1. Lichen planus B
characteristically associated with what 2. Systemic sclerosis
disorder? 3. Cicatricial pemphigoid
4. SJS/TEN
5. Chronic GVHD
6. Show Explaination
70 Transverse white bands on one or two nails is 1. Trauma to the matrix A
caused by: 2. Alopecia areata
3. Psoriasis

164
4. Systemic lupus erythematosis
5. Lichen planus
6. Show Explaination
71 What condition most likely describes a hair 1. Pili torti D
defect with ruffled cuticle in 4-year old female 2. Pili annulati
with blond hair? 3. Wooly hair
4. Loose anagen hair syndrome
5. Monilethrix
6. Show Explaination
72 Mutations in hair keratins hHB6 and hHB1 1. Trichoptilosis E
cause this characteristic finding pictured below 2. Hair casts
� name the condition: 3. Tiger-tail hair
4. Trichoschisis
5. Monilethrix
6. Show Explaination
73 Blue lunulae are characteristic of which 1. Yellow nail syndrome B
disease? 2. Wilson's disease
3. Rubenstein-Taybi syndrome
4. Hypertrophic pulmonary
osteoarthropathy
5. Clubbing
6. Show Explaination
74 The most common cultured agent in chronic 1. Trichophyton rubrum C
paronychia associated with a black hue of the 2. gram-negative bacteria
lateral edge of the nail plate is: 3. Candida spp
4. Scytalidium dimidiatum
5. Scopulariopsis brevicaulis
6. Show Explaination
75 This nail finding is characteristically associated 1. Congenital heart failure B
with what disorder? 2. Chronic renal failure
3. Anemia
4. Hyperthyroidism
5. Cirrhosis
6. Show Explaination
76 Papular atrichia is caused by mutations in 1. Wingless C
which gene? 2. Patched
3. Hairless
4. Fox forehead
5. Distal-less homebobox
6. Show Explaination
77 Yellow nail syndrome can be associated with: 1. D-penicillamine use in patients with A
rheumatoid arthritis
2. Wilson�s disease
3. Alopecia areata
4. Doxorubicin
5. Minocycline use in acne patients
6. Show Explaination
78 The hair finding characterized by an 1. Trichoschisis E
invagination of the distal hair shaft into the 2. Pili torti
cup formed by the proximal hair shaft is: 3. Pili triangulati
4. Trichorrhexis nodosa
5. Trichorrhexis invaginata
6. Show Explaination
79 Thin hair with premature graying is 1. Werner syndrome A
characteristic of which of the following 2. Turner Syndrome
syndromes? 3. Noonan Syndrome
4. Cornelia de Lange Syndrome
5. Waardenburg's syndrome
6. Show Explaination
80 Which of the following is the predominant 1. Diffuse white knots and a brush- C
dermatoscopic finding seen in alopecia areata? pattern
2. Perifollicular arborizing vessels
3. Yellow dots
4. Reduction of follicular ostia
5. Small oval nodes

165
6. Show Explaination
81 Fraying of the cuticles is a sign of which of the 1. Dermatomyositis A
following conditions? 2. Systemic sclerosis
3. Lupus erythematosus
4. Hypothyroidism
5. Lead poisoning
6. Show Explaination
82 Chronic paronychia is most commonly caused 1. Candida albicans A
by which of the following organisms? 2. Candida tropicalis
3. Pseudomonas aeruginosa
4. Staphylococcus aureus
5. Trichophyton rubrum
6. Show Explaination
83 A woman with hypopigmented in lines of 1. Chondrodysplasis punctata E
Blaschko and scarring alopecia likely suffers 2. Anhidrotic Ectodermal Dysplasia
which of the following conditions? 3. Focal Dermal Hypoplasia
4. Rothmund-Thomson Syndrome
5. Bloch-Sulzberger Syndrome
6. Show Explaination
84 How long should the last wash-out period 1. One month C
before performing mycologic investigations in 2. Two months
a clinically recurrent onychomycosis treated by 3. Three months
terbinafine, itraconazole or fluconazole: 4. Four months
5. Six months
6. Show Explaination
85 Blue lunulae are associated with each of the 1. Argyria B
following except: 2. Cardiac failure
3. Quinacrine
4. Wilson�s disease
5. Phenolphthalein
6. Show Explaination
86 Which part of the matrix gives rise to the 1. Distal matrix B
dorsal nail plate? 2. Proximal matrix
3. Ventral matrix
4. Dorsal matrix
5. Lateral matrix
6. Show Explaination
87 A patient with sparse hair, a pear-shaped 1. Focal Dermal Hypoplasia (Goltz B
broad nose and cone-shaped epiphyses likely syndrome)
suffers which of the following conditions? 2. Tricho-rhino-phalangeal Syndrome
3. Hay-Wells Syndrome
4. Ectrodactyly-Ectodermal dysplasia-
Clefting (EEC)
5. Trichodentoosseous Syndrome
6. Show Explaination
88 Atrophy of the isthmus is a characteristic 1. Discoid lupus erythematosus B
histologic finding of which of the following 2. Lichen planopilaris
conditions? 3. Parry-Romberg syndrome
4. Folliculitis Decalvans
5. Acne Keloidalis Nuchae
6. Show Explaination
89 A patient with cirrhosis has noticed that the 1. Half-and-half nails D
proximal aspect of his nails have become 2. Mees' lines
white. This characteristic finding is called: 3. Beau's lines
4. Terry's lines
5. Muehrcke's lines
6. Show Explaination
90 Ferritin levels need to be, at minimum, above 1. 20ng/dL B
which of the serum levels to treat iron- 2. 40ng/dL
deficiency related telogen effluvium? 3. 60ng/dL
4. 30ng/dL
5. 10ng/dL
6. Show Explaination
91 The most common cause for superficial type 1. Trichophyton rubrum B
onychomycosis is: 2. Trichophyton mentagrophytes

166
3. Epidermophyton floccosum
4. Fusarium oxysporum
5. Scopulariopsis brevicalis
6. Show Explaination
92 The common culprit of proximal white 1. Candida albicans B
subungual onychomycosis is : 2. Trichophyton rubrum
3. Trichophyton interdigitale
4. Scydalidium dimidiatum
5. Fusarium spp
6. Show Explaination
93 What determines the thickness of the nail 1. Hyponychium B
plate? 2. Nail matrix
3. Nail bed
4. Proximal nail fold
5. Nail plate
6. Show Explaination
94 Which of the following syndromes can present 1. Bjornstad syndrome D
with woolly or curly hair? 2. Citrullinemia
3. Menkes syndrome
4. Noonan syndrome
5. Netherton syndrome
6. Show Explaination
95 Follicular atrophoderma is a feature of which of 1. Bazex syndrome A
the following conditions? 2. Menkes kinky hair syndrome
3. Papular atrichia
4. Tricho-dento-osseous syndrome
5. Tricho-rhino-phalangeal syndrome
6. Show Explaination
96 A rapid onset of hair growth with or without 1. Adrenal adenoma E
accompanying virilization can occur in all of 2. Adrenal carcinoma
the following conditions EXCEPT: 3. Arrhenoblastoma
4. Krukenburg tumor of the ovary
5. Ovarian cyst
6. Show Explaination
97 A patient with psoriasis has pitting of the nails. 1. Proximal matrix A
This finding is due to involvement of which 2. Nail bed
part of the nail unit? 3. Hyponychium
4. Proximal nail fold
5. Distal matrix
6. Show Explaination
98 A 42 year-old woman presents with the 1. Check plasma levels of A
complaint of excess hair growth on her face. androstenedione and testosterone
She has normal menses and has recently had 2. Send a 21-hydroxylase enzyme
her "annual" exam and the note relates normal deficiency test
sized ovaries. What is the most logical next 3. Biopsy from the most affected area
step? 4. Refer her to endocrinology
5. Order a CT of the abdomen
6. Show Explaination
99 This characteristic nail finding is seen in what 1. Lichen planus C
disorder? 2. Pachyonychia congenita
3. Darier's disease
4. COIF syndrome
5. Coffin-Siris syndrome
6. Show Explaination
10 This hair disorder and mutations in ATP7A 1. Bjornstad syndrome B
0 (MNK) gene are found in what 2. Menke's kinky hair syndrome
genodermatosis? 3. Crandall's syndrome
4. Bazex's follicular atrophoderma
5. Trichothiodystrophy
6. Show Explaination
10 Distal subungual onychomycosis is most often 1. Trichophyton mentagrophytes D
1 caused by: 2. Epidermophyton floccosum
3. Trichophyton schoenleinii
4. Trichophyton rubrum
5. Trichophyton megninii

167
6. Show Explaination
10 What is the mode of inheritance of this 1. Autosomal dominant A
2 disorder of the hair shaft? 2. Autosomal recessive
3. X-linked dominant
4. X-linked recessive
5. Sporadic
6. Show Explaination
10 A patient with a history of gastrointestinal 1. Peutz-Jegher syndrome C
3 polyposis presents with alopecia, generalized 2. Gardner syndrome
pigmentation, and nail dystrophy. Which of the 3. Cronkhite-Canada Syndrome
following is the most likely diagnosis? 4. Cowden syndrome
5. Dyskeratosis congenita
6. Show Explaination
10 What is the common cause of the �one hand- 1. Trichophyton rubrum A
4 two-foot� syndrome? 2. Trichophyton mentagrophytes ( var.
Interdigitale)
3. Candida albicans
4. Scytalidium dimidiatum
5. Scopulariopsis brevicaulis
6. Show Explaination
10 What nutritional deficiency can lead to this nail 1. Vitamin B12 C
5 finding? 2. Vitamin C
3. Iron
4. Magnesium
5. Thiamine
6. Show Explaination
10 Arsenical poisoning is associated with what 1. Muehrcke's lines B
6 nail finding? 2. Mee's lines
3. Hapalonychia
4. Onycholysis
5. Oil spots
6. Show Explaination
10 Anonychia is not a feature of which of the 1. Nail-Patella syndrome E
7 following syndromes? 2. Coffin-Siris syndrome
3. DOOR syndrome
4. COIF syndrome
5. Hidrotic ectodermal dysplasia
6. Show Explaination
10 The epithelium that lies on the volar surface of 1. Nail bed D
8 the digit is the: 2. Nail matrix
3. Nail plate
4. Hyponychium
5. Eponychium
6. Show Explaination
10 Best diagnosis of this type of cicatricial 1. Chronic cutaneous lupus C
9 alopecia shown in attached image is erythematosus
2. Lichen planopilaris
3. Central centrifugal cicatricial
alopecia
4. Pseudopelade of Brocq
5. Alopecia mucinosa
6. Show Explaination
11 Fungal finger onycholysis usually results from: 1. Trichophyton rubrum C
0 2. Trichophyton interdigitale
3. Candida albicans
4. Scopulariopsis brevicaulis
5. Fusarium spp
6. Show Explaination
11 Cutaneous signs of cortisol excess include all 1. Plethora E
1 of the following EXCEPT: 2. Atrophic/fragile skin
3. Striae distensae
4. Dorsocervical/Supraclavicular fat
pads
5. Peripheral obesity
6. Show Explaination

168
11 Steatocystoma multiplex and natal teeth are 1. Jadassohn-Lewandowsky B
2 associated with which of the following pachyonychia congenita (Type 1)
conditions? 2. Jackson-Sertole pachyonychia
congenita (Type 2)
3. Nail-patella syndrome
4. Coffin-Siris syndrome
5. Rubenstein-Taybi syndrome
6. Show Explaination
11 Pili Annulati is associated with which of the 1. Netherton E
3 following syndromes? 2. PIBIDS
3. Citrullinemia
4. Hidrotic ectodermal dysplasia
5. Alopecia areata
6. Show Explaination
11 The combination of ankyloblepharon, 1. Focal Dermal Hypoplasia (Goltz C
4 ectodermal dysplasia, and cleft palate with syndrome)
wiry sparse hair is characteristic of which of 2. Tricho-rhino-phalangeal Syndrome
the following syndromes? 3. Hay-Wells Syndrome
4. Ectrodactyly-Ectodermal dysplasia-
Clefting (EEC)
5. Trichodentoosseous Syndrome
6. Show Explaination
11 A defect in which part of the nail cause a true 1. Cuticle E
5 leukonychia? 2. Hyponychium
3. Lunula
4. Nail bed
5. Nail plate
6. Show Explaination
11 Which of the following conditions with hair 1. Anhidrotic Ectodermal Dysplasia D
6 abnormalities is caused by mutations in a DNA 2. Hidrotic Ectodermal Dysplasia
helicases? 3. KID Syndrome
4. Rothmund-Thomson Syndrome
5. Incontinentia Pigmenti
6. Show Explaination
11 Yellow nail syndrome is associated with which 1. Multiple myeloma E
7 of the following? 2. Panhypopituitarism
3. Diabetes mellitus
4. Dermatophyte infection
5. Lymphedema and bronchiectasis
6. Show Explaination
11 Pathognomonic nails changes in HOOD 1. Red and white longitudinal banding E
8 syndrome or Hereditary osteoonychodysplasia 2. Pterygium inversum
are: 3. Yellow nails
4. Pincer nails
5. Triangular lunulae
6. Show Explaination
11 Which of the following are characteristics of 1. Kinky hair, frontal bossing, small C
9 Trichorhinophalangeal syndrome? widely spaced teeth with poor
enamel
2. Palmoplantar keratoderma,
trichorrhexis nodosa, sinus infections
3. Shortened phalanges, sparse hair,
bulbous nose
4. Brittle hair, short stature, cerebellar
ataxia
5. Sparse fine hair, Short extremities,
Immunodeficiency
6. Show Explaination
12 Which of the following is caused by mutations 1. Rothmund-Thompson syndrome B
0 in gap junction proteins? 2. Hidrotic ectodermal dysplasia
3. Anhidrotic ectodermal dysplasia
4. Netherton syndrome
5. Naxos syndrome
6. Show Explaination
12 A patient presents with 20 nails with absent 1. Upper extremity edema C

169
1 cuticles and lunulae, slow growth, dystrophic 2. Cirrhosis
shape and a yellow hue. Which of the following 3. Pleural effusions
findings is/are associated? 4. Chest pain
5. Pulmonary fibrosis
6. Show Explaination
12 Splinter hemorrhage of the nail can be seen 1. Scabies B
2 with which of the following parasitic infections? 2. Trichinosis
3. Sparganosis
4. Dracunculiasis
5. Gnathostomiasis
6. Show Explaination
12 A patient with yellow nail syndrome should be 1. Hepatic C
3 evaluated for systemic disease involving which 2. Renal
organ system? 3. Pulmonary
4. Cardiovascular
5. Hematologic
6. Show Explaination

12. Bullous and Vesicular Dermatoses


# Question Answers Corre
ct
Answ
er
1 Nickel can be detected in jewelry by applying: 1. Dimethylglyoxime A
2. Methylchloroisothiazolinone
3. Dowicil
4. Quinone
5. Thiocyanate
6. Show Explaination
2 Which pair is not associated with allergic cross- 1. Thimerosal : piroxicam E
reactivity? 2. Ethylenediamine : hydroxyzine
3. PABA : procaine
4. Balsam of Peru : cinnamon
5. Thiuram : sulfa drugs
6. Show Explaination
3 What is the most common site of involvement of 1. Oral mucosa A
this autoimmune blistering disease? 2. Eyes
3. Skin
4. Genitalia
5. Nasopharyngeal mucosa
6. Show Explaination
4 A child has a defect in type VII collagen and 1. Albopapuloid/Pasini variant of C
presents with acral blisters which heal with milia dystrophic epidermolysis bullosa
and scarring. What is the diagnosis? 2. Bart's syndrome
3. Cockayne-Touraine disease
4. Epidermolysis bullosa acquisita
5. Weber-Cockayne
6. Show Explaination
5 Patients with Senear-Usher syndrome are most 1. Erythematous papules and E
likely to present with: plaques around the umbilicus
2. Transient vesicles on the oral
mucosa

170
3. Severely pruritic grouped vesicles
symmetrically distributed
primarily on extensor surfaces
4. Erythema multiforme-like oral
ulcerations which are severe
5. Erythematous crusts and
hyperkeratotic lesions on the
nose, ears, cheeks, scalp, and
chest
6. Show Explaination
6 Which is incorrect? 1. Tulipalin : alstroemeria E
2. Urushiol : anacardia
3. Sesquiterpene lactones :
artichokes
4. Usnic acid : lichen
5. Diallyl disulfide : fig
6. Show Explaination
7 Antibodies against type VII collagen are seen in: 1. Epidermolysis bullosa simplex D
2. Pemphigus erythematosus
3. Cicatricial pemphigoid
4. Epidermolysis bullosa acquisita
5. Bullous pemphigoid
6. Show Explaination
8 Atopic dermatitis is associated with all except: 1. Lchthyosis hystrix A
2. Central facial pallor
3. Pityriasis alba
4. Nipple eczema
5. Hyperlinear palms
6. Show Explaination
9 Perioral exuberant granulation tissue is sometimes 1. Cicatricial pemphigoid B
seen in: 2. Herlitz
3. Duhring's
4. Weber-Cockayne
5. Stevens-Johnson
6. Show Explaination
10 Common causes of drug-induced linear IgA: 1. Vancomycin E
2. Penicillin
3. Cephalosporins
4. Captopril
5. All of these answers are correct
6. Show Explaination
11 Which of the following concerning Degos' Disease 1. After undergoing multiple stages, C
is true? it resolves without scarring
2. It affects women more than men
3. Gatrointestinal involvement is a
poor prognosis
4. Glucocorticoids are standard of
treatment
5. Lab results indicate a low plasma
fibrinogen level and decreased
platelet aggregation
6. Show Explaination
12 The ocular form of cicatricial pemphigoid is most 1. Beta-4-integrin A
likely to be associated with antibodies to: 2. Laminin 5
3. BPAg1
4. BPAg2-NC16A
5. Plectin
6. Show Explaination
13 This is found in both desmosomes and adherens 1. Alpha-catenin D
junctions: 2. Desmoglein 3
3. Desmoglein 1
4. Plakoglobin
5. Lamin
6. Show Explaination
14 Antibodies to which antigen are most likely 1. Bullous pemphigoid antigen II A

171
responsible for this disease? 2. Desmoglein I
3. Desmocollin
4. Keratin 5
5. Plakoglobin
6. Show Explaination
15 An elderly gentleman with a history of 1. Cicatricial pemphigoid D
hypertension and a thymoma, presents to the 2. Sweet�s syndrome
Emergency Room with oral and conjunctival 3. Toxic epidermal necrolysis
erosions and hemorrhagic bullae on his hands and 4. Paraneoplastic pemphigus
feet. These finds are seen in: 5. Erythema multiforme
6. Show Explaination
16 Which of the following is true of herpes 1. Demonstrates antibodies to the C- C
gestationis? terminal domain of BPAg2
2. Has higher frequency in females
with HLA-DQ2
3. Is associated with Grave's disease
4. Usually occurs in the first
trimester of pregnancy
5. Shows subepidermal bullae
mostly with neutrophils
6. Show Explaination
17 Characteristic eosinophilic abscesses are seen in: 1. Bullous drug B
2. Pemphigus vegetans
3. Herpes gestationis
4. Incontinentia pigmenti
5. Paraneoplastic pemphigus
6. Show Explaination
18 At what level is the blister separation plane in 1. Granular layer D
linear IgA dermatosis? 2. Suprabasal layer
3. Basal layer
4. Lamina lucida
5. Lamina densa
6. Show Explaination
19 A 60 year old gentleman presents with a recurrent 1. Desmoglein I B
pustular eruption. Biopsy demonstrates 2. Desmocollin I
subcorneal pustules with abundant neutrophils 3. Desmoplakin
and a serum protein electrophoresis shows a 4. Plakoglobin
monoclonal IgA gammopathy. The autoantigen 5. Beta 4 integrin
responsible for this condition: 6. Show Explaination
20 Antibodies in chronic disease of childhood disease, 1. Basal layer E
map to the: 2. Lamina lucida
3. Lamina densa
4. Sublamina densa
5. Lamina lucida and sublamina
densa
6. Show Explaination
21 A 35 year-old dentist presents with tingling in his 1. Methyl methacrylate A
fingertips. What allergen is most likely to cause 2. Paraphenylenediamine
this dermatitis? 3. Colophony
4. Thiuram mix
5. Ethylenediamine dihydrochloride
6. Show Explaination
22 Which of the following proteins is a 160kDa 1. Desmoglein 3 B
cadherin? 2. Desmoglein 1
3. Desmoplakin I
4. Envoplakin
5. Plectin
6. Show Explaination
23 Which of the following immunosuppressive agents 1. Cyclophosphamide A
has been most effective in cicatricial pemphigoid? 2. Azathioprine
3. Mycophenolate mofetil
4. Cyclosporine
5. Methotrexate
6. Show Explaination
24 Herpes gestationis is exacerbated by: 1. Oral contraceptives E

172
2. Menstruation
3. Third trimester
4. Postpartum state
5. All of these answers are correct
6. Show Explaination
25 Dermatitis herpetiformis is associated with: 1. HLA-Cw6 E
2. HLA-B27
3. HLA-DR4
4. HLA-DQ1
5. HLA-DR3
6. Show Explaination
26 What is the antigen found in the variant of 1. BPAg1 D
cicatrical pemphigoid that is associated with 2. BPAg2
internal malignancies? 3. Type VII collagen
4. Epiligrin
5. Keratins 5 & 14
6. Show Explaination
27 The rate limiting step in the porphyria pathway is 1. Ferrochetalase C
mediated by the enzyme: 2. ALA dehydratase
3. ALA synthase
4. Uroporphyrinogen decarboxylase
5. Porphobilinogen deaminase
6. Show Explaination
28 Common drugs causing SJS/TEN include all except: 1. Sulfonamides E
2. Carbamazepime
3. Allopurinol
4. Oxicam
5. Colchicine
6. Show Explaination
29 Which association is incorrect? 1. Epidermolysis bullosa acquisita : D
inflammatory bowel disease
2. Dermatitis herpetiformis : small
bowel lymphoma
3. Paraneoplastic pemphigus :
Castleman’s
4. Herpes gestationis : menopause
5. Porphyria cutanea tarda :
hemochromatosis
6. Show Explaination
30 Which of the following is true of a phototoxic 1. Requires prior sensitization of E
reaction? photoallergen in susceptible
individuals
2. Photoallergen must bind to carrier
protein
3. Develops after repeated
exposures
4. Produces an eczematous reaction
5. Results in direct tissue injury
6. Show Explaination
31 Which of the following is the most common 1. octyl dimethyl PABA D
photoallergen? 2. Dihydroxyacetone
3. Benzophenone
4. Oxybenzone
5. glyceryl thioglycolate
6. Show Explaination
32 Bullous diabeticorum typically presents on the: 1. Face D
2. Chest
3. Arms
4. Legs
5. Groin
6. Show Explaination
33 Epidermolysis bullosa simplex is caused by 1. Keratins 1&10 C
mutations in: 2. Keratins 1&9
3. Keratins 5&14
4. Keratins 6a &16

173
5. Keratins K2e&10
6. Show Explaination
34 Formaldehyde releasers include all except: 1. Methylchoroisothiazolinone A
2. Germall
3. Quaternium � 15
4. Dowicil
5. DMDM hydantoin
6. Show Explaination
35 Fiberglass dermatitis can be prevented by: 1. Water E
2. Acetic acid 5%
3. Sodium chloride
4. Alkali
5. Talcum powder
6. Show Explaination
36 Which type of porphyria disease is most likely to 1. Acute intermittent porphyria D
be associated with cholelithiasis? 2. Congenital erythropoeitic
porphyria
3. Hereditary coproporphyria
4. Erythropoetic protoporphyria
5. Variegate porphyria
6. Show Explaination
37 The lamina densa is primarily composed of? 1. Connexins E
2. Plectin
3. Collagen XVII
4. Collagen VII
5. Collagen IV
6. Show Explaination
38 Etyhlenediamine dihydrochloride is known to 1. Neomycin A
cross-react with all the following substances 2. Aminophylline
except? 3. Promethazine
4. Meclizine
5. Hydroxyzine
6. Show Explaination
39 The Absoe-Hansen sign as applies to blister 1. Lateral slipping of the epidermis B
formation refers to when perilesional is rubbed
2. Lateral dissection of the blister
when pressure is applied directly
to a blister
3. Blister formation following
stroking of the lesion
4. Blister induction with incised
trauma to skin
5. Blister formation after
cryosurgery
6. Show Explaination
40 The C-terminal domain of BPAg2 is targeted in: 1. Bullous pemphigoid B
2. Cicatricial pemphigoid
3. IgA pemphigus
4. Duhring�s disease
5. Pemphigoid gestationis
6. Show Explaination
41 The most common cause of contact dermatitis in 1. Hair dyes D
the United States is: 2. Nickel
3. Nail lacquers
4. Toxicodendrons
5. Rubber
6. Show Explaination
42 A patient who works in a photography developing 1. Mercaptobenzothiazole D
lab complains of eczematous dermatitis on his 2. Lanolin
hands. Which one of the following chemicals is he 3. Epoxy resin
most likely allergic to? 4. Parapheylenediamine
5. Colophony
6. Show Explaination
43 Bullous lupus erythematosus is most commonly 1. Type XII collagen B
associated with antibodies to: 2. Type VII collagen

174
3. Plectin
4. Desmoglein I
5. Laminin 5
6. Show Explaination
44 Pinus palustris is the source of: 1. Urea E
2. D-limonene
3. Quinones
4. Lactones
5. Abietic acid
6. Show Explaination
45 Which of the following forms of epidermolysis 1. Weber-Cockayne C
bullosa is autosomal recessive? 2. Dowling-Meara
3. Junctional EB with pyloric atresia
4. EB simplex with muscular
dystrophy
5. Hyperplastic cockayne-touraine
6. Show Explaination
46 This medicine causes degranulation of mast cells 1. NSAIDs E
2. ASA
3. opiates
4. polymixin B
5. All of these answers are correct
6. Show Explaination
47 Latex allergy can cross-react with all except: 1. Avocado E
2. Banana
3. Kiwi
4. Chestnut
5. Artichoke
6. Show Explaination
48 Leiner's disease (erythroderma desquamativum) is 1. C5-9 deficiency D
associated with: 2. Numerous infections
3. Diarrhea
4. Numerous infections and diarrhea
5. All of these answers are correct
6. Show Explaination
49 The croton plant is irritating secondary to: 1. Calcium oxalate D
2. Thiocyanate
3. Protoanemonin
4. Phorbol esters
5. Capsaicin
6. Show Explaination
50 Herpes gestationis is most commonly associated 1. Grave's A
with: 2. Hashimoto's
3. Diabetes
4. Lupus
5. Rheumatoid arthritis
6. Show Explaination
51 Clumped tonofilaments are seen on electron 1. Dowling-Meara A
microscopy in: 2. Weber-Cockayne
3. Gunther�s
4. Naxos disease
5. Vohwinkel�s syndrome
6. Show Explaination
52 What is the most common cause of this entity? 1. Drugs A
2. Infections
3. Malignancies
4. Idiopathic
5. UV exposure
6. Show Explaination
53 Gap junctions consist of: 1. Laminin E
2. Plectin
3. Uncein
4. Filagrin
5. Connexins
6. Show Explaination

175
54 This is derived from Myroxylon pereirae: 1. Thimerosol C
2. Aniline dyes
3. Balsam of Peru
4. Colophony
5. Thiuram
6. Show Explaination
55 Which of the following agent(s) has been most 1. Cyclophosphamide C
effective in treating severe ocular cicatricial 2. Mycophenolate mofetil
pemphigoid? 3. Cyclophosphamide +
corticosteroids
4. Cyclosporin
5. Azathioprine
6. Show Explaination
56 Chloracne may be secondary to exposure to: 1. Chloroacetophenone D
2. Hydrochloric acid
3. Hydrocarbons
4. Dioxin
5. Methylchloroisothiazolinone
6. Show Explaination
57 Herpes gestationis is most commonly associated 1. Lymphoma C
with: 2. Multiparity
3. Grave's disease
4. Inflammatory bowel disease
5. Lupus
6. Show Explaination
58 Which form of epidermolysis bullosa presents with 1. Weber-cockayne B
generalized bullae, absent nails, dysplastic teeth 2. Herlitz
(due to enamel defects), nonhealing granulation 3. Non-Herlitz
tissue periorally, and is often fatal by age 3-4? 4. Hyperplastic cockayne-touraine
5. Albapapuloid Pasini variant
6. Show Explaination
59 Which of the following is not a formaldehyde- 1. Bronopol B
releasing preservative? 2. Methylchloroisothiazinolone
3. Quaternium-15
4. Imidazolidinyl urea
5. DMDM hydantoin
6. Show Explaination
60 The epidermolysis bullosa acquisita antigen is: 1. A transmembrane protein E
2. An intracellular protein
3. A desmosomal protein
4. Located in the lamina lucida
5. An anchoring fibril
6. Show Explaination
61 Which type of porphyria manifests with peripheral 1. Acute Intermittent Porphyria A
neuropathy, colicky abdominal pain, but NO 2. Gunther's disease
cutaneous symptoms? 3. Variegate porphyria
4. Hereditary Copropophyria
5. Erythropoietic protopophyria
6. Show Explaination
62 Direct immunofluorescent studies in chronic 1. Iga depostion in the superficial C
bullous disease of childhood is most likely to show: blood vessels
2. Linear IgG at the basement
membrane
3. Linear IgA at the basement
membrane
4. Granular IgG
5. Linear C3 at the basement
membrane
6. Show Explaination
63 Circulating autoantibodies to type XVII collagen 1. Epidermolysis bullosa accquisita B
are most characteristic of which disease? 2. Herpes gestationis
3. Pemphigus vulgaris
4. Pemphigus foliacious
5. Paraneoplastic pemphigus
6. Show Explaination

176
64 A woman in her 2nd trimester of pregnancy 1. Lymphoma C
presents to clinic with urticarial plaques and 2. Multiparity
papules around her umbilicus, chest and 3. Grave’s Disease
extremities. Tense vesicles are present within a 4. Inflammatory Bowel Disease
few of the erythematous plaques. This 5. Lupus
woman’s condition is most commonly 6. Show Explaination
associated with:
65 The picture shown is considered to be diagnostic 1. Junctional epidermolysis bullosa- E
for what type of epidermolysis bullosa Non Herlitz type
2. Junctional epidermolysis bullosa-
Herlitz type
3. Dowling-Meara form of
Epidermolysis Bullous simplex
4. Dominant dystrophic
epidermolysis bullosa
5. Recessive dystrophic
epidermolysis bullosa
6. Show Explaination
66 Common cause(s) of drug-induced pemphigus 1. Captopril C
foliaceus: 2. Penicillamine
3. Captopril and penicillamine
4. Methotrexate
5. Dilantin
6. Show Explaination
67 Which autoantibodies would most likely be found 1. Alpha 6 beta 4 integrin A
in an individual with eye findings without any 2. Bullous pemphigoid antigen 1
cutaneous involvement? 3. Bullous pemphigoid antigen 2
4. Desmocollin
5. Laminin 5
6. Show Explaination
68 Which pair is incorrect? 1. Desmoglein 1 : 160 kDa E
2. Desmoplakin I : 250 kDa
3. Type VII collagen : 290 kDa
4. Type XVII collagen: 180 kDa
5. Envoplakin : 190 kDa
6. Show Explaination
69 Patients with epidermolysis bullosa acquisita have: 1. A defect in plectin D
2. A defect in collagen XVII
3. Antibodies to collagen XVII
4. Antibodies to collagen VII
5. A defect in laminin 5
6. Show Explaination
70 A one-day old infant presents with papules and 1. Acropustulosis of infancy C
pustules on the face, trunk, and proximal 2. Herpes infection
extremities. Palms and soles are spared. The 3. Erythema toxicum neanotorum
patient is otherwise doing well. A biopsy shows 4. Scabies
numerous eosinophils. The diagnosis is: 5. Transient neonatal pustular
melanosis
6. Show Explaination
71 Which pair is incorrect? 1. Wrinkle-resistance : formaldehyde C
2. Chewing gum : colophony
3. Parabens : artichokes
4. Glyceryl thioglycolate : acid
permanent wave
5. Permethrin : chrysanthemum
6. Show Explaination
72 Pemphigus erythematosus: 1. Is also called Hallopeau syndrome C
2. May be an abortive form of
subcorneal pustulosis
3. Is often in a malar/seborrheic
distribution
4. Does not have linear IgG and C3
at the basement membrane zone
5. All of these answers are correct
6. Show Explaination

177
73 The vector of fogo selvagem may be: 1. Triatoma B
2. Simulium
3. Cimex
4. Ornithodorus
5. Mus
6. Show Explaination
74 The most common malignancy associated with 1. Non-Hodgkin's lymphoma A
paraneoplastic pemphigus is: 2. Chronic lymphocytic leukemia
3. Multiple myeloma
4. Acute myelocytic leukemia
5. Hodgkin's lymphoma
6. Show Explaination
75 The defect in junctional epidermolysis bullosa 1. Granular layer C
occurs at the: 2. Spinous layer
3. Lamina lucida
4. Lamina densa
5. Sublamina densa
6. Show Explaination
76 A 20-year-old female presents with an allergic 1. Cinnamic acid C
contact dermatitis to a perfume containing Lily of 2. Vanillin
the valley. What is the causative allergen? 3. Hydroxycitronellal
4. Atranorin
5. Evernic acid
6. Show Explaination
77 Which neoplasm is the most common cause of 1. Thymoma E
paraneoplastic pemphigus? 2. CLL
3. Castleman's disease
4. Retroperitoneal sarcoma
5. Non-Hodgkin's lymphoma
6. Show Explaination
78 Which of the following allergens is a non- 1. Methylisothiazolone B
formadelhyde releasing preservative with a low 2. Paraben
sensitization potential? 3. Bronopol
4. Quternium-15
5. DMDM hydantoin
6. Show Explaination
79 Patients with this disorder may develop 1. Chronic lymphocytic leukemia A
exaggerated reactions to insect bites: 2. Bullous pemphigoid
3. Lupus erythematosus
4. Atopic dermatitis
5. Incontinentia pigmenti
6. Show Explaination
80 What is the antigen associated with dermatitis 1. Tissue transglutaminase A
herpetiformis? 2. Epilegrin
3. Desmocollin
4. Desmoglein 3
5. Periplakin
6. Show Explaination
81 What percentage of dermatitis herpetiformis 1. 5% E
patients are asymptomatic but have findings 2. 15%
consistent with celiac sprue on gastrointestinal 3. 40%
biopsy? 4. 70%
5. 95%
6. Show Explaination
82 Which of the following is LEAST LIKELY to cross 1. Japanese lacquer tree E
react with Toxicodendron plant dermatitis? 2. Cashew nut
3. Mango rind
4. Gingko tree
5. Kiwi
6. Show Explaination
83 The dimethylglyoxime test is used to detect the 1. Gold D
presence of: 2. Silver
3. Mercury
4. Nickel
5. Latex

178
6. Show Explaination
84 Which of the following proteins is typically NOT 1. BPAg1 B
found to be antigenic in paraneoplastic 2. BPAg2
pemphigus? 3. Desmoplakin
4. Envoplakin
5. Periplakin
6. Show Explaination
85 Which pair is incorrect? 1. Dermatitis herpetiformis : B
granular IgA
2. Lichen planus : linear C3
3. Bullous pemphigoid : linear C3
4. Systemic lupus erythematosus :
linear IgG
5. IgA pemphigus: intercellular IgA
6. Show Explaination
86 Paraneoplastic pemphigus: 1. Is characterized by a D
pathognomonic 250 kDa antigen
2. Is most often seen in association
with lung cancer
3. Does not remit even if the cancer
is excised completely
4. May be caused by a benign
neoplasm
5. All of these answers are correct
6. Show Explaination
87 Patients with latex allergy are LEAST LIKELY to 1. Cashew A
develop a reaction to: 2. Avocado
3. Chestnut
4. Banana
5. Kiwi
6. Show Explaination
88 A patient develops erythema and vesiculation on 1. Thiuram mix D
his dorsal foot that spares the toewebs and soles 2. Carbamates
after buying a new pair of shoes. What is the most 3. Latex
likely allergen? 4. Mercaptobenzothiazole
5. Formaldehydge
6. Show Explaination
89 Irritation of the hand produced by capsaicin can 1. Water B
be relieved by application of 2. Acetic acid 5%
3. Sodium chloride
4. Alkali
5. Talcum powder
6. Show Explaination
90 Which of the following substances is known to 1. Gold A
cause a delayed positive patch test reaction? 2. Nickel
3. Bacitracin
4. Fragrance mix
5. Rosin
6. Show Explaination
91 Which of the following can be responsible for 1. Lanolin alcohol D
contact dermatitis to K-Y Jelly? 2. Budesonide
3. Alpha tocopherol
4. Propylene glycol
5. Triclosan
6. Show Explaination
92 The antigen in linear IgA is: 1. BPAg1 C
2. Plectin
3. 97kDa part of BPAg2
4. Laminin 5
5. Alpha-6, beta integrin
6. Show Explaination
93 Which pair is incorrect? 1. Eyedrops : thimerosal E
2. Ear drops : neomycin
3. Theophylline : ethylenediamine
4. Antabuse : thiuram

179
5. Cobalamin : balsam
6. Show Explaination
94 The primary autoantigen in pemphigoid 1. Desmoplakin C
gestationis is 2. BPAG1
3. BPAG2
4. Plakoglobin
5. Anchoring fibrils
6. Show Explaination
95 Which of the following is known as "prickly heat"? 1. Miliaria crystallina B
2. Miliaria rubra
3. Miliaria profunda
4. Grover's disease
5. Fox-Fordyce disease
6. Show Explaination
96 Patients with Duhring's disease are most likely to 1. Mutations in plectin D
have: 2. Mutations in laminin 5
3. Mutations in transglutaminase I
4. Antibodies to transglutaminase 3
5. Antibodies to BPAg2
6. Show Explaination
97 In staphylococcal scalded skin syndrome, the 1. Desmoglein 1 A
exfoliative toxin cleaves: 2. Desmoglein 3
3. Desmocollin 1
4. Desmocollin 3
5. BpAg2
6. Show Explaination
98 Which type of epidermolysis bullosa has the 1. Weber-Cockayne E
greatest risk of developing squamous cell 2. Dowling-Meara
carcinomas? 3. Herlitz
4. EB Simplex with muscular
dystrophy
5. Hallopeau-Siemens type of
recessive dystrophic EB
6. Show Explaination
99 Fecal isocoproporphyrin is seen in: 1. Porphyria cutanea tarda A
2. Harderoporphyria
3. Variegate porphyria
4. Acute intermittent porphyria
5. Coproporphyria
6. Show Explaination
10 In Brunsting-Perry pemphigoid, the recrurrent 1. Genitals D
0 crops of blisters are most likely to appear on: 2. Palms and soles
3. Umbilicus
4. Head and neck
5. Buttocks
6. Show Explaination
10 Bullous pemphigoid antigen 1 (BPAg1) is a 1. Cadherin D
1 member of this family: 2. Integrin
3. Intermediate filament
4. Plakin
5. Selectin
6. Show Explaination
10 Bullous lesions are seen in: 1. Congenital syphilis A
2 2. Primary syphilis
3. Secondary syphilis
4. Tertiary syphilis
5. All of these answers are correct
6. Show Explaination
10 Transient bullous disorder of childhood has a 1. Alpha6-integrin E
3 defect in: 2. Plectin
3. BPAg2
4. Epiligrin
5. Collagen VII
6. Show Explaination
10 The most important group of disease linked with 1. Lymphoproliferative disorders A

180
4 the pathogenesis of paraneoplastic pemphigus in 2. Colerectal adenocarcinoma
adults is 3. Breast Cancer
4. Sarcomas
5. Melanoma
6. Show Explaination
10 You are consulted on this patient who has just 1. She should avoid bananas A
5 undergone an orthopedic spinal procedure. What 2. She is allergic to betadine
is true of this patient? 3. She is most likely having an
allergic response to an antibiotic
4. She has miliaria crystallina
5. This is not related to her
procedure
6. Show Explaination
10 What is the enzyme defect in Gunther's disease? 1. Porphobilinogen deaminase B
6 2. Uroporphyrinogen synthetase III
3. Uroporphyrinogen decarboxylase
4. Ferrochetalase
5. Protoporphyrinogen oxidase
6. Show Explaination
10 Cicatricial pemphigoid can be induced by: 1. Aminoglycosides C
7 2. Benzene
3. Clonidine
4. Vancomycin
5. Ciprofloxacin
6. Show Explaination
10 The best first line treatment for dermatitis 1. Methotrexate B
8 herpetiformis is: 2. Dapsone
3. Colchicine
4. Prednisone
5. IVIG
6. Show Explaination
10 "Oriental" tiger balm cross-reacts with: 1. Balsam of Peru A
9 2. Ylang-Ylang oil
3. Neomycin
4. Lanolin
5. Rosin
6. Show Explaination
11 Which steriod screening agent should be used 1. Glutaral B
0 when an allergic contact dermaititis is suspected 2. Tixocortol-21-pivalate
to hydrocortisone? 3. Budesonide
4. Hydrocortisone-17-butyrate
5. Benzalkonium chloride
6. Show Explaination
11 Gallstones are associated with: 1. Erythropoietic protoporphyria A
1 2. Hepatoerythropoietic porphyria
3. Variegate porphyria
4. Coproporphyria
5. Harderoporphyria
6. Show Explaination
11 Mutations in beta-catenin are most commonly 1. Naxos disease C
2 associated with: 2. Bullous pemphigoid
3. Pilomatricomas
4. Ectodermal dysplasia with skin
fragility
5. Ocular cicatricial pemphigoid
6. Show Explaination
11 Which of the following is the most common cause 1. Paraben mix C
3 of contact dermatitis due to a formaldehyde 2. Thimerosal
releasing preservative? 3. Quaternium 15
4. Imidazolidinyl urea
5. Paraphenylenediamine
6. Show Explaination
11 Cleavage of this adhesion molecule is seen in 1. Desmoplakin 1 C
4 Staphylococcal Scalded Skin Syndrome: 2. Desmoplakin 2
3. Desmoglein 1

181
4. Desmoglein 3
5. Desmoplakin 1 and desmoplakin
2
6. Show Explaination
11 This is associated with deafness: 1. Claudin C
5 2. Cadherin
3. Connexin
4. Desmin
5. Desmoplakin
6. Show Explaination
11 Each of the following is true about anti-p200 1. Responsive to dapsone E
6 pemphigoid except: 2. Subepidermal bullae
3. 200-kd antigen
4. features of linear IgA disease
5. mucosal involvement
6. Show Explaination
11 Which of the following chemicals is the most 1. Thiuram mix B
7 common cause of shoe contact dermatitis? 2. 2-Mercaptobenzothiazole
3. Carba mix
4. Mixed dialkyl thioureas
5. Formaldehyde
6. Show Explaination
11 Papillary dermal deposits of IgA and a papillary 1. Sweet's syndrome C
8 dermal infiltrate of neutrophils is diagnostic of: 2. Leukocytoclastic vasculitis
3. Dermatitis herpetiformis
4. Linear IgA dermatosis
5. Bullous pemphigoid
6. Show Explaination
11 Antibodies in some forms of Stevens-Johnson bind 1. Desmoplakin I A
9 to: 2. Desmoglein 1
3. Desmoglein 3
4. Plakoglobin
5. Desmoplakin I and desmoglein 3
6. Show Explaination
12 This disease is inherited in an X-linked recessive 1. Chronic granulomatous disorder A
0 manner: 2. Job�s syndrome
3. Dowling-Meara
4. Mongomery�s syndrome
5. Treacher-Collins
6. Show Explaination
12 A 59-year old female has erythema and 1. Ammonium persulfate C
1 vesiculation in an area of application of tincture of 2. Black rubber mix
benzoin. Which allergen is she also most likely 3. Balsam of Peru
allergic to? 4. Eugenol
5. Epoxy resin
6. Show Explaination
12 Which of the associated antigens for this condition 1. Anti-epiligrin A
2 has been associated with colon cancer? 2. 97 kD linear IgA dermatosis
antigen
3. Desmoplakin
4. Bullous pemphigoid antigen 2
5. Beta-4 integrin
6. Show Explaination
12 Which of the following adhesion molecules are 1. Selectins C
3 important for maintaining adhesion between 2. Integrins
keratinocytes? 3. Cadherins
4. Glycans
5. Immunoglobulin superfamily
6. Show Explaination
12 Th2-related cytokines include: 1. IL10 A
4 2. IFNgamma
3. IFNalpha
4. IL10 and IFNgamma
5. All of these answers are correct
6. Show Explaination

182
12 Allergic contact dermatitis is characterized 1. Psoriasiform dermatitis C
5 histologically by: 2. Lichenoid infiltrate
3. Spongiosis
4. Parakeratosis
5. Granuloma
6. Show Explaination
12 Ocular cicatricial pemphigoid has antibodies 1. Gamma-catenin C
6 against: 2. Peripherin
3. Beta4-integrin
4. Kalinin
5. Vinculin
6. Show Explaination
12 In penicillamine-induced pemphigus, the split is 1. Subcorneal A
7 most often: 2. Intraspinous
3. Suprabasal
4. Intraepidermal and subepidermal
5. Subepidermal
6. Show Explaination
12 Epidermolysis bullosa simplex with muscular 1. Laminin 5 B
8 dystrophy is associated with a mutation in: 2. Plectin
3. collagen VII
4. uncein
5. alpha 6 beta integrin
6. Show Explaination
12 Which of the following contains the same allergen 1. Mango fruit B
9 as poison sumac? 2. Brazilian Pepper Tree
3. Balsam of Peru
4. Ragweed
5. Artichoke
6. Show Explaination
13 Direct immunofluorescent studies in a patient with 1. Linear IgA at the basement B
0 bullous pemphigoid is most likely to show: membrane
2. Linear C3 at the basemement
membrane
3. Granular IgA in dermal papillae
4. Intercellular IgG4 throughout the
epidermis
5. C3 in the dermal papillae
6. Show Explaination
13 What is the most common allergen causing 1. Nickel A
1 allergic contact dermatitis? 2. Bacitracin
3. Formaldehyde
4. Quaternium-15
5. Neomycin
6. Show Explaination
13 A child develops an allergic reaction at the site of 1. Lanolin B
2 an influenza vaccine. To which of the following 2. Thimerosol
substances may she be allergic to? 3. Ethylenediamine dichloride
4. Triclosan
5. Gluteraldehyde
6. Show Explaination
13 The agent most likely to cause pseudoporphyria 1. NSAID's A
3 is: 2. Penicillin
3. Beta-blockers
4. Vancomycin
5. Ace-inhibitors
6. Show Explaination

183

You might also like